Problemas para la 18 a Olimpiada Mexicana de Matemáticas (Avanzado)

Problemas para la 18a Olimpiada Mexicana de Matem´ aticas (Avanzado) Editado por: Anne Alberro Semerena Radmila Bulajich Manfrino Jos´ e Antonio G´ omez Ortega 2004 Anne Alberro Semerena Facultad de Ciencias, Universidad Aut´onoma del Estado de Morelos. Radmila Bulajich Manfrino Facultad de Ciencias, Universidad Aut´onoma del Estado de Morelos. Jos´ e Antonio G´ omez Ortega Facultad de Ciencias, Universidad Nacional Aut´onoma de M´exico. Presentaci´ on La Sociedad Matem´atica Mexicana organiza la 18a Olimpiada Mexicana de Matem´aticas. Los ganadores en ella formar´an la preselecci´on que se preparar´a para las las distintas olimpiadas internacionales del a˜no 2005: la XLVI Olimpiada Internacional, que se llevar´a a cabo en M´exico durante el mes de julio, la XX Olimpiada Iberoamericana a celebrarse en septiembre en Ecuador y la VII Olimpiada de Centroam´erica y el Caribe que se llevar´a a cabo en Honduras en el mes de julio. En la 18a Olimpiada Mexicana de Matem´aticas pueden participar los estudiantes de M´exico nacidos despu´es del 1o de agosto de 1985. Los concursantes deber´an estar inscritos en una instituci´on preuniversitaria durante el primer semestre del c´ıclo escolar 2004 - 2005 y para el 1o de julio del a˜no 2005 no deber´an haber iniciado estudios de nivel universitario. Los problemas que se presentan en este folleto aparecieron en las primeras etapas de las Olimpiadas Estatales de Matem´aticas en M´exico. La intenci´on de este folleto es que sirva como orientaci´on a los alumnos que desean participar en estas Olimpiadas. Como se puede ver, los problemas que aparecen aqu´ı, no son ejercicios rutinarios o problemas en los que se apliquen directamente los conocimientos que se adquieren en la escuela. M´as bien son problemas que requieren de una buena dosis de ingenio y de esfuerzo para ser resueltos. Como en todos los aspectos del aprendizaje de las matem´aticas, el esfuerzo individual y el enfrentamiento solitario con los problemas son importantes, pero tambi´en es muy importante la discusi´on con los compa˜neros y los profesores. Una forma de manifestar creatividad en matem´aticas es resolviendo problemas. Otra forma, que a veces requiere de m´as madurez, es invent´andolos. Invitamos a todos los lectores de este folleto: profesores, estudiantes, ol´ımpicos y exol´ımpicos a que nos env´ıen problemas con soluci´on. Las aportaciones ser´an consideradas iv Etapas de la Olimpiada para su inclusi´on en ex´amenes o en futuros folletos. Esta publicaci´on incluye una seleccci´on de los problemas que formaron parte de los folletos avanzados elaborados como gu´ıas para la 13a y 14a olimpiada mexicana de matam´aticas. El primero editado por: Omar Antol´ın, Radmila Bulajich, Carlos Cabrera y Jos´e Antonio G´omez; el segundo por: Omar Antol´ın, Radmila Bulajich, Jos´e Antonio G´omez y Rita V´azquez. Etapas de la Olimpiada Como ya es tradici´on, la Olimpiada Mexicana de Matem´aticas consta de tres etapas: Ex´ amenes Estatales. Estos ex´amenes servir´an para formar las selecciones estatales que asistir´an al Concurso Nacional. Concurso Nacional. Este concurso se llevar´a a cabo en el Estado de M´exico en noviembre de 2004 y en ´el se elegir´a a la preselecci´on mexicana. Entrenamientos. A los alumnos de la preselecci´on que surjan del Concurso Nacional se les entrenar´a intensivamente durante el primer semestre del a˜no 2005. Tambi´en se les aplicar´an ex´amenes para determinar a los que representar´an a M´exico en las Olimpiadas Internacionales. La participaci´on en las tres etapas mencionadas es individual. Resumen de Resultados v Resumen de Resultados En el a˜no de 1987 la Sociedad Matem´atica Mexicana organiz´o la Primera Olimpiada Mexicana de Matem´aticas. A partir de esa fecha, los concursos nacionales se han celebrado anualmente en las ciudades de Xalapa, Hermosillo, Metepec, Guanajuato, Oaxtepec, La Trinidad, Acapulco, Guadalajara, Colima, M´erida, Monterrey, Quer´etaro, Oaxaca, Morelia, Oaxtepec, Colima y Guanajuato. Resultados de M´ exico en las Internacionales Los resultados de las Delegaciones Mexicanas en Olimpiadas Internacionales, Iberoamericanas y de Centroam´erica y el Caribe han sido los siguientes: Olimpiada Interncional de Matem´ aticas A˜no 1988 1989 1990 1991 1992 1993 1994 1995 1996 1997 1998 1999 2000 2001 2002 2003 Pa´ıs sede Australia Rep. Fed. de Alemania Rep. Popular de China Suecia Rusia Turqu´ıa Hong Kong Canad´a India Argentina Taiwan Rumania Corea Estados Unidos Escocia Jap´on No. de pa´ıses 49 50 54 55 56 73 69 74 75 82 75 81 82 83 84 86 Lugar de M´exico 37 31 36 35 49 63 65 59 53 32 44 52 30 46 46 42 En 2003, la delegaci´on que represent´o a M´exico en la Olimpiada Internacional estuvo integrada por los alumnos: Octavio Arizmendi Echegaray (Morelos), Ana Paula Estrada Vargas (Jalisco), Marco Antonio Figueroa Ibarra (Sonora), Yoalli Mabel Hidalgo Pontet (Jalisco), Antonio Olivas Mart´ınez (Sonora) y Carlos Vargas Obieta (Jalisco). Se obtuvieron 3 medallas de bronce (Marco Antonio Figueroa Ibarra, Antonio Olivas Mart´ınez y Ana Paula Estrada Vargas) y una Resumen de Resultados vi menci´on honor´ıfica (Yoalli Mabel Hidalgo Pontet). En total, en las Olimpiadas Internacionales se han obtenido 3 medallas de plata, 21 medallas de bronce y 17 menciones honor´ıficas. Olimpiada Iberoamericana de Matem´ aticas A˜no 1989 1990 1991 1992 1993 1994 1995 1996 1997 1998 1999 2000 2001 2002 2003 Pa´ıs sede Cuba Espa˜na Argentina Venezuela M´exico Brasil Chile Costa Rica M´exico Rep´ublica Dominicna Cuba Venezuela Uruguay El Salvador Argentina No. de pa´ıses 13 15 16 16 16 16 18 17 17 18 20 21 21 22 19 Lugar de M´exico 3 3 5 6 9 6 9 2 3 5 3 2 3 3 4 Los cuatro integrantes de la delegaci´on mexicana que participaron en la Olimpiada Iberoamerica de 2003 obtuvieron medalla: una de oro (Marco Antonio Figueroa Ibarra de Sonora), medalla de plata (Antonio Olivas Mart´ınez de Sonora) y dos medallas de bronce (Carlos Vargas Obieta de Jalisco y Adri´an Enrique Chi Centeno de Yucat´an). En total, en las Olimpiadas Iberoamericanas. M´exico ha obtenido 10 medallas de oro, 22 medallas de plata, 21 medallas de bronce y 3 menciones honor´ıficas. Resumen de Resultados vii Olimpiada Matem´ atica de Centroam´ erica y el Caribe A˜no 1999 2000 2001 2002 2003 Pa´ıs sede No. de pa´ıses Costa Rica 10 El Salvador 9 Colombia 10 M´exico 8 Costa Rica 11 Lugar de M´exico 2 2 2 1 1 Los 3 alumnos mexicanos en la IV Olimpiada Matem´atica de Centroam´erica y el Caribe, ganaron 2 medallas de oro, ellos fueron Iv´an Josh´ua Hern´andez M´aynez de Coahuila y Gonzalo Arturo Montalv´an G´amez de Puebla y una medalla de plata para el alumno Rosemberg Toala Enr´ıquez de Chiapas. En total en la Olimpiada Matem´atica de Centroam´erica y el Caribe, M´exico ha obtenido 7 medallas de oro, 6 de plata y 2 de bronce. Resultados del Concurso Nacional de la 17a Olimpiada Mexicana de Matem´ aticas En noviembre de 2003 se llev´o a cabo en Guanajuato, Gto. el 17a Concurso Nacional, con la participaci´on de todos los estados de la Rep´ublica. Los 16 alumnos ganadores del primer lugar fueron: Marco Antonio Figueroa Ibarra (Sonora) H´ector Daniel Garc´ıa Lara (Chihuahua) Iv´an Joshua Hern´andez M´aynez (Coahuila) Rosemberg Toal´a Enr´ıquez (Chiapas) Gonzalo Arturo Montalv´an G´amez (Puebla) Carlos Vargas Obieta (Jalisco) Federico Bribiesca Argomedo (Michoac´an) Rafael D´ıaz Cruz (Distrito Federal) Guillermo Enrique Carro Prado (Nuevo Le´on) Jos´e Miguel Cisneros Franco (Veracruz) Francisco Javier Ibarra Goycoolea (Baja California) Cristos Alberto Ruiz Toscano (Jalisco) Guevara Manuel Angel Guevara L´opez (Zacatecas) Luis Alberto Mart´ınez Chigo (Veracruz) Arturo Aguirre Escobar (Distrito Federal) David Guadalupe Torres Flores (Guanajuato) Resumen de Resultados viii Los 5 alumnos preseleccionados para la Olimpiada Matem´atica de Centroam´erica y el Caribe fueron: Isaac Buenrostro Morales (Jalisco) Iv´an Joshua Hern´andez M´aynez (Coahuila) Jonathan Eliud Rinc´on Galv´an (Nuevo Le´on) Pablo Sober´on Bravo (Morelos) David Guadalupe Torres Flores (Guanajuato) Aunque la participaci´on en el Concurso Nacional es individual, es importante destacar la labor que han llevado a cabo los estados de la Rep´ublica apoyando a sus concursantes. Con el prop´osito de reconocer este trabajo, presentamos el registro de los estados que ocuparon los primeros 10 lugares en el 17a Concurso Nacional. 1. Jalisco 2. Puebla 3. Chihuahua 4. Distrito Federal 5. Sonora 6. Morelos 7. Nuevo Le´on 8. Guanajuato 9. Quer´etaro 10. Veracruz En esta ocasi´on, el premio a la Superaci´on Acad´emica se llam´o Quanaxhuato y fue ganado por el Distrito Federal. El segundo y tercer lugar de este premio lo ocuparon, respectivamente, Veracruz y Tlaxcala. Informaci´ on sobre la Olimpiada. Para obtener m´as informaci´on sobre los eventos de la Olimpiada Mexicana de Matem´atica o para consultar m´as material de estudio, visita nuestro sitio de Internet: http://erdos.fciencias.unam.mx/omm COMITE ORGANIZADOR DE LA OLIMPIADA MEXICANA DE MATEMATICAS Mayo 2004 Contenido Presentaci´ on III 1. Enunciados de los Problemas 1 1.1. Problemas de Pr´actica . . . . . . . . . . . . . . . . . . . . . . . 1 1.2. Problemas de los ´ultimos tres Concursos Nacionales de la OMM . . . . . . . . . . . . . . . . . . . . . . . 10 2. Olimpiadas Internacionales en las que participa M´ exico 15 2.1. V Olimpiada Matem´atica de Centroam´erica y el Caribe . . . . . . 15 2.2. XVIII Olimpiada Iberoamericana de Matem´aticas . . . . . . . . . . . . . . . . . . . . . . . . . . . . 16 2.3. XLIV Olimpiada Internacional de Matem´aticas . . . . . . . . . . . . . . . . . . . . . . . . . . . . 18 3. Soluciones de los Problemas 21 3.1. Soluciones de los Problemas de Pr´actica . . . . . . . . . . . . . 21 3.2. Soluciones de los ´ultimos tres Concursos Nacionales de la OMM . . . . . . . . . . . . . . . . . . . . . . . 49 Bibliograf´ıa 68 x Contenido Cap´ıtulo 1 Enunciados de los Problemas Presentamos aqu´ı algunos problemas para mostrar el tipo de matem´aticas que se manejan en la fase estatal de la Olimpiada Mexicana de Matem´aticas. Al final encontrar´as las soluciones. 1.1. Problemas de Pr´ actica Problema 1. Considere 109 enteros con 0 < a1 < ... < a109 < 1999. Muestre que entre los valores di = ai+1 − ai , i = 1, ..., 108 hay un valor que se repite 4 o m´as veces. Encuentre un ejemplo de enteros 0 < a1 < ... < a109 ≤ 1999 donde ninguna diferencia di = ai+1 − ai se repita m´as de 3 veces. Problema 2. Sean a1 , a2 , ..., an enteros con las propiedades de que: (i) a1 · a2 · ... · an = n (ii) a1 + a2 + · · · + an = 0 Muestre que 4 divide a n. Problema 3. Un n´umero de tres cifras es “equilibrado” si una de sus cifras es . el promedio de las otras dos, por ejemplo el 258 es equilibrado pues 5 = 2+8 2 ¿Cu´antos n´umeros equilibrados de tres cifras hay? 2 Problemas de Pr´ actica Problema 4. Si intentamos cubrir una cuadr´ıcula de 5 × 5 con piezas de tama˜no 2×1, siempre nos quedar´a un hueco. ¿En qu´e sitios de la cuadr´ıcula puede quedar el hueco? Problema 5. ¿Para qu´e enteros positivos n es posible dividir un tri´angulo equil´atero de lado n en trapecios iguales cuyos lados midan 1, 1, 1 y 2? Problema 6. Un icosaedro es un s´olido regular de 20 caras, cada una de las cuales es un tri´angulo equil´atero. ¿Cu´antas diagonales tiene un icosaedro? Problema 7. Utilizando exclusivamente los d´ıgitos 2 y a se forma el siguiente n´umero de 90 cifras. 2a22a222a2222a...,22..,2a. Si el n´umero es m´ultiplo de 9, ¿qu´e valores son posibles para el d´ıgito a? Problema 8. Encuentre todos los enteros positivos a, b tales que: a + b−1 = 13 a−1 + b y a + b ≤ 80. Problema 9. Considerando la figura siguiente muestre que: AM CP + =1 AN CQ A Q M P θ θ B θ N C 1.1 Problemas de Pr´ actica 3 Problema 10. Sea ABCD un cuadril´atero inscrito en una circunferencia, las l´ıneas BD y AC se cortan en el punto P . Si O es el circuncentro del tri´angulo AP B y H es el ortocentro del tri´angulo CP D, demuestre que O, P y H est´an alineados. Problema 11. ¿De cu´antas formas distintas se puede llenar una cuadr´ıcula de 4 × 4 con fichas de 2 × 1? Problema 12. En una c´arcel hay 10 reos condenados a muerte a los que se les va a dar una ´ultima oportunidad para salvarse: se pondr´an los 10 en una fila y a cada uno le pondr´an un sombrero, ya sea blanco o negro. Cada reo s´olo podr´a ver el color de los sombreros de sus compa˜neros de adelante (no podra ver el suyo ni ninguno de los de atr´as). Se les ir´a preguntando, de uno en uno, empezando por el u ´ltimo de la fila y en orden hasta terminar con el primero: “¿Cu´al cree que es el color de su sombrero?”. Si un reo atina a su color le salvan la vida, si no lo matan. ¿C´omo le pueden hacer los reos para ponerse de acuerdo de tal forma que se salven al menos 9 reos? Problema 13. El conjunto {1, 2, ..., n} se colorea de rojo y negro, de manera que 1 y n reciban diferente color. Muestre que el n´umero de parejas de enteros consecutivos con diferente color es impar. Problema 14. Una cuadr´ıcula de 8 × 8 se cubre con 32 domin´os de 2 × 1. Demuestre que al menos existen dos domin´os que forman un cuadrado de 2 × 2. Problema 15. ¿Cu´antos paralelep´ıpedos rectangulares distintos se pueden construir, para los cuales la longitud de cada arista es un entero del 1 al 10?. Problema 16. ¿Ser´a posible numerar las 4 caras, los 4 v´ertices y las 6 aristas de un tetraedro, es decir, asignarles un n´umero entero del 1 al 14, de tal manera que el n´umero asignado a una arista sea igual al promedio de los n´umeros asignados a los v´ertices que la determinan, e igual al promedio de los n´umeros asignados a las caras que la comparten? Problema 17. Encuentre una lista de cinco primos diferentes donde la diferencia entre cualesquiera dos t´erminos consecutivos de la lista sea siempre seis. Pruebe que esta lista es ´unica. 4 Problemas de Pr´ actica Problema 18. Sea ABC un tri´angulo escaleno de ´area 1999. Sea A1 un punto del lado BC y sean B1 y C1 puntos sobre las rectas AC y AB respectivamente, tales que AA1 , BB1 y CC1 son paralelas. Encuentre el ´area del tri´angulo A1 B1 C1 . Problema 19. (i) Pruebe que para todo entero positivo n existen tres enteros a, b, c tales que n = a2 + b2 − c2 . (ii) Pruebe que a lo m´as para 1699 enteros positivos n, n < 1999, existen enteros a, b, c tales que n = a2 + b2 + c2 . Problema 20. En el plano se marcan a + b puntos; a de ellos se designan con la letra A y los otros b puntos se designan con la letra B. Al unir los puntos consecutivos se forma un pol´ıgono de a + b lados. Sobre cada uno de los lados hacemos lo siguiente: si los dos v´ertices del lado est´an denotados por letras A, escribimos el n´umero 2; si los dos v´ertices del lado est´an denotados por letras B, escribimos el n´umero 12 ; si los dos v´ertices del lado est´an denotados por letras diferentes, escribimos el n´umero 1. ¿Cu´al es el producto de todos los n´umeros escritos? Problema 21. Sea ABC un tri´angulo, D y E los pies de las alturas desde A y B repectivamente. Sean M en la prolongaci´on de BE tal que EM = AD y N la intersecci´on de la prolongaci´on BC con la perpendicular a BM por M. Demuestre que el tri´angulo NCA es is´osceles. Problema 22. Sea AL la bisectriz del ´angulo A de un tri´angulo acut´angulo ABC. Sean M y N sobre los lados AB y AC respectivamente, de manera que ∠MLA = ∠B y ∠NLA = ∠C. Si D es el punto de intersecci´on de AL y MN, muestre que AL3 = AB · AC · AD. Problema 23. Sean ABC un tri´angulo y L, M, N los puntos medios de los lados BC, CA, AB respectivamente. Muestre que ∠LAC = ∠MBA si y s´olo si ∠CNA = ∠ALB. Problema 24. Si a, b y c son n´umeros positivos con a + b + c = 2, muestre que: a2 b2 c2 + + ≥ 1. a+b b+c c+a Problema 25. Considere un tri´angulo rect´angulo ABC y llame P , Q, R, a las reflexiones de A, B, C sobre BC, CA, AB respectivamente. Calcule la raz´on del ´area del tri´angulo ABC entre el ´area del tri´angulo P QR. 1.1 Problemas de Pr´ actica 5 Problema 26. Dentro de un hex´agono regular ABCDEF se coloca un punto G arbitrario y se trazan segmentos que lo unen con los v´ertices, formando 6 tri´angulos ABG, BCG, CDG, DEG, EF G, F AG los cuales se colorean en forma alternada de negro y blanco. Demuestre que el ´area de la regi´on negra es igual al ´area blanca. Problema 27. Este es un juego para dos jugadores que se juega en un tablero cuadrado cuadriculado que tiene un n´umero impar de filas y columnas. El juego empieza en la esquina inferior izquierda, donde el primer jugador pone su marca. Los jugadores alternan su jugada. En su turno cada jugador puede poner su marca en una de las casillas contiguas ya sea directamente encima, directamente a la derecha o diagonalmente encima y a la derecha de la ´ultima marca puesta por su oponente. El juego contin´ua de esta forma y gana el jugador que consiga poner su marca en la casilla ubicada en la esquina superior derecha del tablero. Encuentre una estrategia que permita al primer jugador ganar siempre el juego. Problema 28. ¿Por qu´e cada vez que tomamos diez n´umeros consecutivos existe uno que es primo relativo con cada uno de los otros nueve? Problema 29. En el tri´angulo equil´atero ABC, una recta paralela al lado BC intersecta a AC en M y a AB en P . Si D es el incentro del tri´angulo AP M y E es el punto medio de P C, determine los ´angulos del tri´angulo BDE. Problema 30. En cada subconjunto de 7 elementos del conjunto {1,2,...,10} se toma el mayor. ¿Cu´al es la suma de todos esos elementos mayores? Problema 31. Sea A un subconjunto de N = {1, 2, ..., n} y arreglemos sus elementos en orden decreciente de magnitud. Formemos las sumas S sumando y restando alternadamente elementos consecutivos del subconjunto A (el primer elemento de la suma siempre es positivo). ¿Cu´al es la suma de todas las sumas S generadas por los distintos subconjuntos de N? Problema 32. Si ABCDEF G es un hept´agono regular, muestre que 1 1 1 = + . AB AC AD Problema 33. Sea ABC un tri´angulo y sean E, F puntos sobre los lados CA y AB. Sea D la intersecci´on de BE con CF y suponga que las ´areas de los tri´angulos BDF , CDE y BCD son 3, 3 y 9 respectivamente. Encuentre el ´area del cuadril´atero AF DE. 6 Problemas de Pr´ actica Problema 34. Una cuadr´ıcula de 3 × 3 est´a inicialmente llena con ceros y unos de la siguiente manera: 1 0 1 0 1 0 1 0 1 La cuadr´ıcula se ir´a modificando con la siguiente regla: Cada que se coloque una cuadr´ıcula de 2 × 2 sobre la cuadr´ıcula original, se aumentan en 1 los n´umeros de los cuadros comunes. Por ejemplo si la cuadr´ıcula de 2 × 2 se coloca arriba y a la izquierda del tablero de 3 × 3, ´este queda modificado as´ı: 2 1 1 1 2 0 1 0 1 ¿Es posible, repitiendo la regla, llegar a un tablero de 3 × 3 donde todos los n´umeros sean m´ultiplos de 2? Si lo es, d´e una forma de hacerlo, en caso contrario argumente por qu´e no es posible. Problema 35. ¿Es posible que alguna potencia de 2 termine en 1982? Problema 36. Sea E un punto sobre una circunferencia de di´ametro AB. Por los puntos B y E se trazan las rectas tangentes t1 y t2 , respectivamente, que se cortan en el punto M. Sea C el punto de intersecci´on de las rectas AE y t1 . Demuestre que M es el punto medio de BC. Problema 37. En la siguiente cuadr´ıcula: 2 1 2 1 2 1 2 1 2 1 2 1 2 1 2 1 2 1 2 1 2 1 2 1 2 una “sustituci´on”consiste en tomar el n´umero en un cuadro y sumarle o restarle un m´ultiplo par de alguno de sus vecinos. ¿Es posible que despu´es de varias sustituciones sucesivas queden los n´umeros del 1 al 25 en la cuadr´ıcula? 1.1 Problemas de Pr´ actica 7 Problema 38. ¿Ser´a posible dividir la superficie de una esfera en un n´umero impar de regiones, todas triangulares? Nota: Cada dos regiones triangulares son ajenas, o comparten un v´ertice, o bien tienen un lado completo en com´un. Problema 39. Demuestre que no es posible construir un tri´angulo cuyos lados sean n´umeros primos y que su ´area sea entera. Problema 40. Se tienen los n´umeros del 1 al 100 ({1, 2, 3, 4, ..., 100}) y cada uno es pintado de rojo, azul, amarillo o verde. Demuestre que hay dos n´umeros del mismo color cuya diferencia tambi´en es del mismo color. Problema 41. Sea C1 una circunferencia fija y AB una cuerda en ella. Sean P un punto en uno de los arcos AB y C2 una circunferencia tangente a C1 en P y tangente a la cuerda AB. Si Q es el punto de tangencia de AB a C2 , demuestre que el ´angulo AP Q es constante, para todo punto P sobre el arco AB. Problema 42. Consideremos un c´ırculo con centro O, A y B puntos en la circunferencia tales que ∠AOB = 60◦ . Sean M un punto cualquiera en el arco AB y P , Q, R, S, los puntos medios de los segmentos AM, OB, OA, BM, respectivamente. Demuestre que P R es perpendicular a P S. Problema 43. Sean x, y, z n´umeros enteros positivos tales que 7 divide a x3 + y 3 − z 3 . Pruebe que alguno de ellos es m´ultiplo de 7. Problema 44. Pruebe que si un n´umero primo p puede escribirse de la forma p = p21 + p22 + p23 con p1 < p2 < p3 n´umeros primos entonces p1 = 3. Problema 45. Un rect´angulo grande est´a dividido en 9 rect´angulos m´as peque˜nos, como muestra la figura. En la parte interior de algunos rect´angulos peque˜nos est´a escrito su per´ımetro. ¿Cu´al es el per´ımetro del rect´angulo grande? 6 12 4 8 6 8 Problemas de Pr´ actica Problema 46. Se quieren pintar las caras de un dodecaedro de tal forma que si dos caras tienen una arista en com´un entonces quedan pintadas con colores diferentes. (i) Muestre que no es posible hacer esto con s´olo tres colores, (ii) Muestre que es posible lograr esto con cuatro colores. Problema 47. Si x1 · x2 · x3 · x4 + x2 · x3 · x4 · x5 + ... + xn · x1 · x2 · x3 = 0 donde cada xi es 1 ´o −1, demuestre que n es m´ultiplo de 4. Problema 48. Encuentre el entero positivo m´as peque˜no que se puede expresar como suma de 9 enteros positivos consecutivos, como suma de 10 enteros positivos consecutivos y como suma de 11 enteros positivos consecutivos (las tres cosas a la vez). Problema 49. ¿Es posible agrupar los n´umeros 1, 2, ..., 100 en 3 grupos A1 , A2 y A3 tales que 102 divida a la suma de los elementos de A1 , 203 divida a la suma de los elementos de A2 y 304 divida a la suma de los elementos de A3 ? Problema 50. Paula numera un cuaderno con 96 hojas y coloca en las p´aginas la secuencia del 1 al 192. Juan quita 24 hojas al azar y suma los 48 n´umeros escritos en las p´aginas. ¿Puede ser esta suma igual a 1998? Problema 51. Dada una cuadr´ıcula de 5×5 se escribe en cada uno de los cuadros un 1 ´o un −1. El producto de los n´umeros en cada columna es escrito debajo de cada una de ellas. El producto de los n´umeros en cada rengl´on es escrito a la derecha de cada uno de estos. Pruebe que la suma de esos 10 productos no puede ser cero. Problema 52. Demuestre que si p y q son n´umeros primos tales que entero entonces p = q. p2 +q 2 p+q es Problema 53. Pruebe que el n´umero de cuatro cifras N = abcd es divisible entre 3 si (a − 2b + c + 4d) es divisible entre 3. Problema 54. Pablo ha dibujado un cuadrado ABCD con tinta negra y debe colorear con rojo todos los puntos P del interior del cuadrado tales que el ´area del cuadril´atero BCP A es igual al triple del ´area del cuadril´atero AP CD. Describir cu´al es la parte roja del dibujo y justificar. 1.1 Problemas de Pr´ actica 9 Problema 55. Supongamos que queremos formar 5 pilas de cajas con las siguientes condiciones: cada pila debe tener entre una y cinco cajas, adem´as, cada pila no puede tener m´as cajas que la pila de su izquierda. ¿De cu´antas formas podemos hacer esto? Problema 56. Tenemos 16 focos acomodados en un tablero de 4 × 4, todos apagados. Cada vez que alguien toca un foco, este cambia de estado junto con todos los focos de su fila y su columna (Las filas son horizontales y las columnas verticales). (i) Demuestre que es posible, despu´es de tocar los focos adecuados, que todos los focos queden encendidos. (ii) Si el tama˜no del tablero fuera de 5 × 5. ¿Ser´ıa posible terminar con todos los focos prendidos? Problema 57. Los n´umeros del 1 al 12 se colocan (sin repetir) en los c´ırculos del siguiente arreglo triangular: (i) Demuestre que no existe una forma de acomodarlos que cumpla que las sumas de los n´umeros que est´an en cada uno de los lados del tri´angulo sea 27. (ii) Muestre que si existe un acomodo en que la suma en cada uno de los lados del tri´angulo sea 28. Problema 58. Sea ABCDEF GH un oct´agono regular de radio 1. Demuestra que AB · AD = AC. Problema 59. Demuestre que para 4 puntos no alineados, los cuatro tri´angulos que estos puntos determinan no pueden ser todos acut´angulos. 10 Problemas Concursos Nacionales de la OMM Problema 60. Sea ABCD un cuadril´atero convexo, donde M es el punto medio de BC, N es el punto medio de CD y O es la intersecci´on de las diagonales AC y BD. Demuestre que O es el gravicentro del tri´angulo AMN si y s´olo si ABCD es un paralelogramo. Problema 61. La colecci´on infinita de n´umeros 1, 2, 4, 5, 7, 9, 10, 12,14,16, ... se ha formado de la manera siguiente: se coloca el primer impar (1), luego se colocan los siguientes dos pares (2, 4), despu´es los tres impares siguientes al u ´ltimo par colocado (5, 7, 9), luego los cuatro pares siguientes al ´ultimo impar que se coloc´o y as´ı sucesivamente. ¿Cu´al es el n´umero par m´as cercano a 1998 que aparece en esta colecci´on? Problema 62. (i) Demuestre que el producto de tres enteros positivos consecutivos no puede ser un cubo. (ii) Demuestre que s´olo existe un primo p tal que el n´umero 2p + 1 es un cubo. Problema 63. Sean a, b y c tres enteros positivos tales que a > b > c. Pruebe que si a + b es m´ultiplo de c, b + c es m´ultiplo de a y a + c es m´ultiplo de b, abc entonces el cociente a+b+c es un cuadrado. Problema 64. ¿Cu´ales son las posibles ´areas de un hex´agono con todos los ´angulos iguales y cuyos lados miden 1, 2, 3, 4, 5, 6, en alg´un orden? Problema 65. En una cuadr´ıcula de 3 × 3 se acomodan los d´ıgitos del 1 al 9 sin repetir. Considere cada rengl´on como un n´umero de 3 cifras, y ll´amele A a la suma de estos tres n´umeros. Ahora, considere cada columna como un n´umero de tres cifras, y ll´amele B a la suma de estos tres n´umeros. ¿Puede encontrar alguna forma de acomodar los d´ıgitos del 1 al 9 de manera que A + B = 1997? 1.2. Problemas de los u ´ltimos tres Concursos Nacionales de la OMM Problema 1. (15a OMM) Encuentra todos los n´umeros de 7 d´ıgitos que son m´ultiplos de 3 y de 7, y cada uno de cuyos d´ıgitos es 3 o´ 7. Enunciados de los Problemas 11 Problema 2. (15a OMM) Se tienen algunas pelotas de colores (son por lo menos tres colores), y por lo menos tres cajas. Las pelotas se ponen en las cajas de manera que no quede vac´ıa ninguna caja y que no haya tres pelotas de colores distintos que est´en en tres cajas distintas. Prueba que hay una caja tal que todas las pelotas que est´an fuera de ella son del mismo color. Problema 3. (15a OMM) En un cuadril´atero ABCD inscrito en una circunferencia llamemos P al punto de intersecci´on de las diagonales AC y BD, y sea M el punto medio de CD. La circunferencia que pasa por P y que es tangente a CD en M corta a BD y a AC en los puntos Q y R, respectivamente. Se toma un punto S sobre el segmento BD de tal manera que BS = DQ. Por S se traza una paralela a AB que corta a AC en un punto T . Prueba que AT = RC. Problema 4. (15a OMM) Dados dos enteros positivos n y a se forma una lista de 2001 n´umeros como sigue: El primer n´umero es a; a partir del segundo, cada n´umero es el residuo que se obtiene al dividir el cuadrado del anterior entre n. A los n´umeros de la lista se les ponen los signos + y − alternadamente empezando con +. Los n´umeros con signo as´ı obtenidos se suman y a esa suma se le llama suma final para n y a. ¿Para qu´e enteros n ≥ 5 existe alguna a tal que 2 ≤ a < n2 y la suma final para n y a es positiva? Problema 5. (15a OMM) Sea ABC un tri´angulo tal que AB < AC y el ´angulo BAC es el doble del ´angulo BCA. Sobre el lado AC se toma un punto D tal que CD = AB. Por el punto B se traza una recta l paralela a AC. La bisectriz exterior del ´angulo en A intersecta a l en el punto M, y la paralela a AB por el punto C intersecta a l en el punto N. Prueba que MD = ND. Problema 6. (15a OMM) Un coleccionista de monedas raras tiene monedas de denominaciones 1, 2, ..., n (tiene muchas monedas de cada denominaci´on). Desea poner algunas de sus monedas en 5 cajas de manera que se cumplan las siguientes condiciones: (a) En cada caja hay a lo m´as una moneda de cada denominaci´on. (b) Todas las cajas tienen el mismo n´umero de monedas y la misma cantidad de dinero. (c) Para cualesquiera dos cajas sucede que entre los dos tienen por lo menos una moneda de cada denominaci´on. (d) No existe una denominaci´on tal que todas las cajas tengan una moneda de esa denominaci´on. ¿Para qu´e valores de n puede el coleccionista hacer lo que se propone? 12 Problemas Concursos Nacionales de la OMM Problema 7. (16a OMM) En una cuadr´ıcula de 32 × 32 se escriben los n´umeros del 1 al 1024 de izquierda a derecha, con los n´umeros del 1 al 32 en el primer rengl´on, los del 33 al 64 en el segundo, etc. La cuadr´ıcula se divide en cuatro cuadr´ıculas de 16 × 16 que se cambian de lugar entre ellas como sigue: Despu´es, cada cuadr´ıcula de 16 × 16 se divide en cuatro cuadr´ıculas de 8 × 8 que se cambian de lugar del mismo modo; a su vez cada una de esas se divide y as´ı sucesivamente hasta llegar a cuadr´ıculas de 2 × 2 que se dividen en cuadros de 1 × 1, los cuales se cambian de lugar del mismo modo. Al terminar estas operaciones, ¿qu´e n´umeros quedan en la diagonal que va de la esquina superior izquierda a la inferior derecha en la cuadr´ıcula de 32 × 32? Problema 8. (16a OMM) Sean ABCD un paralelogramo y K la circunferencia circunscrita al tri´angulo ABD. Sean E y F las intersecciones de K con los lados (o sus prolongaciones) BC y CD respectivamente (E distinto de B y F distinto de D). Demuestra que el circuncentro del tri´angulo CEF est´a sobre K. Problema 9. (16a OMM) Sean n un entero positivo. ¿Tiene n2 m´as divisores positivos de la forma 4k + 1 o de la forma 4k − 1? Problema 10. (16a OMM) Una ficha de domin´o tiene dos n´umeros (no necesariamente diferentes) entre 0 y 6. Las fichas se pueden voltear, es decir, 4 5 es la misma ficha que 5 4 . Se quiere formar una hilera de fichas de domin´o distintas de manera que en cada momento de la construcci´on de la hilera, la suma de todos los n´umeros de las fichas puestas hasta ese momento sea impar. Las fichas se pueden agregar de la manera usual a ambos extremos de la hilera, es decir, de manera que en cualesquiera dos fichas consecutivas aparezca el mismo n´umero en los extremos que se juntan. Por ejemplo, se podr´ıa hacer la hilera: 1 3 3 4 4 4 , en la que se coloc´o primero la ficha del centro y luego la de la izquierda. Despu´es de poner la primera ficha, la suma de todos los n´umeros es 7; despu´es de poner la segunda, 11; despu´es de la tercera, 19. ¿Cu´al es la mayor cantidad de fichas que se pueden colocar en una hilera? ¿Cu´antas hileras de esa longitud m´axima se pueden construir? Problema 11. (16a OMM) Tres enteros distintos forman una terna compatible si alguno de ellos, digamos n, cumple que cada uno de los otros dos es, o bien divisor, o bien m´ultiplo de n. Para cada terna compatible de n´umeros entre 1 y 2002 se calcula la suma de los tres n´umeros de la terna. ¿Cu´al es la mayor suma obtenida? ¿Cu´ales son las ternas en las que se obtiene la suma m´axima? Enunciados de los Problemas 13 Problema 12. (16a OMM) Sea ABCD un cuadril´atero con AD paralelo a BC, los ´angulos en A y B rectos y tal que el ´angulo CMD es recto, donde M es el punto medio de AB. Sean K el pie de la perpendicular a CD que pasa por M, P el punto de intersecci´on de AK con BD y Q el punto de intersecci´on de BK con AC. Demuestra que el ´angulo AKB es recto y que KQ KP + = 1. PA QB Problema 13. (17a OMM) Dado un n´umero k de dos o m´as cifras, se forma otro entero m insertando un cero entre la cifra de las unidades y la de las decenas de k. Encuentra todos los n´umeros k para los cuales m resulta ser un m´ultiplo de k. Problema 14. (17a OMM) Sean A, B y C tres puntos colineales con B entre A y C. Sea Y una circunferencia tangente a AC en B, y sean X y Z las circunferencias de di´ametros AB y BC, respectivamente. Sea P el otro punto (adem´as de B) en el que se cortan las circunferencias X y Y; sea Q el otro punto (adem´as de B) en el que se cortan las circunferencias Y y Z. Sup´on que la recta P Q corta a X en un punto R distinto de P , y que esa misma recta P Q corta a Z en un punto S ditinto de Q. Demuestra que concurren AR, CS y la tangente com´un a X y Z por B. Problema 15. (17a OMM) En una fiesta hay el mismo n´umero n de muchachos que de muchachas. Sup´on que a cada muchacha le gustan a muchachos y que a cada muchacho le gustan b muchachas. ¿Para qu´e valores de a y b es correcto afirmar que forzosamente hay un muchacho y una muchacha que se gustan mutuamente? Problema 16. (17a OMM) Sea ABCD un trapecio con AB paralelo a DC. Se toman puntos P y Q sobre AB y CD respectivamente, tales que PAPB = DQ . QC Sea M la intersecci´on de AQ con DP y sea N la intersecci´on de P C con QB. Pruebe que la longitud de MN depende s´olo de las longitudes de AB y DC, y calcula su valor. 14 Problemas Concursos Nacionales de la OMM Problema 17. (17a OMM) Se escriben en tarjetas todas las parejas de enteros (a, b) con 1 ≤ a < b ≤ 2003. Dos personas juegan con las tarjetas como sigue: cada jugador en su turno elige (a, b) (que se retira del juego) y escribe el producto a · b en un pizarr´on (ambos jugadores usan el mismo pizarr´on). Pierde el jugador que ocasione que el m´aximo com´un divisor de los n´umeros escritos hasta ese momento sea 1. ¿Qui´en tiene estrategia ganadora? (Es decir, ¿cu´al de los dos jugadores puede inventar un m´etodo con el cual asegura su triunfo?) Problema 18. (17a OMM) Dado un entero n un cambio sensato consiste en sustituir n por 2n + 1 ´o 3n + 2. Dos enteros positivos a y b se llaman compatible si existe un entero que se puede obtener haciendo uno o m´as cambios sensatos, tanto a partir de a, como a partir de b. Encuentra todos los enteros positivos compatibles con 2003 menores que 2003. Cap´ıtulo 2 Olimpiadas Internacionales en las que participa M´ exico 2.1. V Olimpiada Matem´ atica de Centroam´ erica y el Caribe Problema 1. Dos jugadores A y B, juegan por turnos el siguiente juego: Se tiene un mont´on de 2003 piedras. En su primer turno, A escoge un divisor de 2003, y retira ese n´umero de piedras del mont´on inicial. Posteriormente, B escoge un divisor del n´umero de piedras restantes, y retira ese n´umero de piedras del nuevo mont´on, y siguen as´ı sucesivamente. Pierde el jugador que retira la u ´ltima piedra. Demostrar que uno de los dos jugadores tiene una estrategia ganadora y describir dicha estrategia. Nota: Se entiende por estrategia ganadora un m´etodo de juego que le garantiza la victoria al que lo aplica sin importar lo que haga su oponente. Problema 2. Sea S una circunferencia y AB un di´ametro de ella. Sea t la recta tangente a S en B y considere dos puntos C, D en t tales que B est´e entre C y D. Sean E y F las intersecciones de S en AC y AD, y sean G y H las intersecciones de S con CF y DE. Demostrar que AH = AG. 16 Problemas de las Olimpiadas Internacionales Problema 3. Sean a, b enteros positivos, con a > 1 y b > 2. Demostrar que ab + 1 ≥ b(a + 1) y determinar cu´ando se tiene la igualdad. Problema 4. Sean S1 y S2 dos circunferencias que se intersectan en dos puntos distintos P y Q. Sean l1 y l2 dos rectas paralelas, tales que: (i) l1 pasa por el punto P e intersecta a S1 en un punto A1 distinto de P y a S1 en un punto A2 distinto de P . (ii) l2 pasa por el punto Q e intersecta a S1 en un punto B1 distinto de Q y a S2 en un punto B2 distinto de Q. Demostrar que los tri´angulos A1 QA2 y B1 QB2 tienen igual per´ımetro. Problema 5. Un tablero cuadrado de 8 cm de lado se divide en 64 casillas cuadradas de 1 cm de lado cada una. Cada casilla se puede pintar de blanco o de negro. Encontrar el n´umero total de maneras de colorear el tablero de modo tal que cada cuadrado de 2 cm de lado formado por cuatro casillas con un v´ertice com´un, contenga dos casillas blancas y dos negras. Problema 6. Digamos que un entero positivo es tico si la suma de sus d´ıgitos (en base 10) es m´ultiplo de 2003. paralelas, tales que: (i) Demostrar que existe un entero positivo N tal que sus primeros 2003 m´ultiplos N, 2N, 3N, . . . , 2003N, son todos ticos. (ii) ¿Existe alg´un entero positivo N tal que todos sus m´ultiplos sean ticos? 2.2. XVIII Olimpiada Iberoamericana de Matem´ aticas Problema 1. (a) Se tienen dos sucesiones, cada una de 2003 enteros consecutivos, y un tablero de 2 filas y 2003 columnas ... ... ... ... Decida si siempre es posible distribuir los n´umeros de la primera sucesi´on en la primera fila y los de la segunda sucesi´on en la segunda fila, de tal manera que los resultados obtenidos al sumar los dos n´umeros de cada columna formen una nueva sucesi´on de 2003 n´umeros consecutivos. Enunciados de los Problemas 17 (b) ¿Y si se reemplaza 2003 por 2004? Tanto en a) como en b), si la respuesta es afirmativa, explique c´omo ditribuir´ıa los n´umeros, y si es negativa, justifique el porqu´e. Problema 2. Sean C y D dos puntos de la semicircunferencia de di´ametro AB tales que B y C est´an en semiplanos distintos respecto de la recta AD. Denotemos M, N y P los puntos medios de AC, DB y CD, respectivamente. Sean OA y OB los circuncentros de los tri´angulos ACP y BDP . Demuestre que las rectas OA OB y MN son papalelas. Problema 3. Pablo estaba copiando el siguiente problema: Considere todas las sucesiones de 2004 n´umeros reales (x0 , x1 , x2 , . . . , x2003 ), tales que x0 0≤ 0≤ = x1 x2 .. . 1, ≤ 2x0 , ≤ 2x1 , 0 ≤ x2003 ≤ 2x2002 . Entre todas estas sucesiones, determine aquella para la cual la siguiente expresi´on toma su mayor valor: S = . . . . Cuando Pablo iba a copiar la expresi´on de S le borraron la pizarra. Lo u ´nico que pudo recordar es que S era de la forma S = ±x1 ± x2 ± . . . ± x2002 + x2003 , donde el ´ultimo t´ermino, x2003 , ten´ıa coeficiente +1, y los anteriores ten´ıan coeficientes +1 ´o −1. Demuestre que Pablo, a pesar de no tener el enunciado completo, puede determinar con certeza la soluci´on del problema. Problema 4. Sea M = {1, 2, . . . , 49} el conjunto de los primeros 49 enteros positivos. Determine el m´aximo entero k tal que el conjunto M tiene un subconjunto de k elementos en el que no hay 6 n´umeros consecutivos. Para ese valor m´aximo de k, halle la cantidad de subconjuntos de M, de k elementos, que tienen la propiedad mencionada. 18 Problemas de las Olimpiadas Internacionales Problema 5. En el cuadrado ABCD, sean P y Q puntos pertenecientes a los lados BC y CD respectivamente, distintos de los extremos, tales que BP = CQ. Se consideran puntos X e Y , X 6= Y , pertenecientes a los segmentos AP y AQ, respectivamente. Demuestre que, cualesquiera sean X e Y , existe un tri´angulo cuyos lados tienen las longitudes de los segmentos BX, XY y DY . Problema 6. Se definen las suceisones (an )n≥0 , (bn )n≥0 por: an+1 a0 = 1 = a2001 + bn , n b0 = 4 y bn+1 = b2001 + an , n para n ≥ 0. Demuestre que 2003 no divide a ninguno de los t´erminos de estas sucesiones. 2.3. XLIV Olimpiada Internacional de Matem´ aticas Problema 1. Sea A un subconjunto del conjunto S = {1, 2, . . . , 1000000} que contiene exactamente 101 elementos. Demuestra que existen n´umeros t1 , t2 , . . . , t100 en S tal que los conjuntos Ai = {x + tj / x ∈ A} para j = 1, 2, . . . , 100 son ajenos dos a dos. Problema 2. Encuentra todas las parejas de enteros positivos (a, b) tales que sea un entero positivo. a2 2ab2 − b2 + 1 Problema 3. Considera un hex´agono convexo en el cual cualesquiera dos lados √ opuestos tienen la siguiente propiedad: la distancia entre sus puntos medios es 23 veces la suma de sus longitudes. Demuestra que todos los ´angulos del hex´agono son iguales. Un hex´agono convexo ABCDEF tiene tres pares de lados opuestos: AB y DE, BC y EF , CD y F A. Problema 4. Sea ABCD un cuadril´atero c´ıclico. Sean P , Q y R los pies de las perpendiculares desde D a las rectas BC, CA y AB, respectivamente. Demuestra que P Q = QR si y s´olo si la bisectriz del ∠ABC y ∠ADC se intersectan en AC. Enunciados de los Problemas 19 Problema 5. Sea n un entero positivo y x1 , x2 , . . . , xn n´umeros reales tales que x1 ≤ x2 ≤ · · · ≤ xn . (a) Demuestra que n X n X i=1 j=1 |xi − xj | !2 n ≤ n 2(n2 − 1) X X (xi − xj )2 . 3 i=1 j=1 (b) Demuestra que la igualdad es v´alida si y s´olo si x1 , x2 , . . . , xn es una progresi´on aritm´etica. Problema 6. Sea p un n´umero primo. Demuestra que existe un n´umero primo q tal que para todo entero n, el n´umero np − p no es divisible entre q. 20 Problemas de las Olimpiadas Internacionales Cap´ıtulo 3 Soluciones de los Problemas 3.1. Soluciones de los Problemas de Pr´ actica Soluci´ on del problema 1. (i) Sean di = ai+1 −ai las diferencias, i = 1, ..., 108. Por un lado tenemos que: d1 + d2 + · · · + d108 = a109 − a1 ≤ 1998 − 1 = 1997. Ahora, si d′1 ≤ d′2 ≤ · · · ≤ d′108 son las diferencias ordenadas, como d′1 , d′2 , d′3 son al menos 1; d′4 , d′5 , d′6 son al menos 2;...; d′106 , d′107 , d′108 son al menos 36, tenemos que: d1 + d2 + · · · + d108 = d′1 + d′2 + · · · + d′108 ≥ 3 (1 + 2 + · · · + 36) = 1998, lo que da una contradicci´on. (ii) Considere: a1 = 1 y ai = a1 + d1 + · · · + di−1 , donde d1 = d2 = d3 = 1, d4 = d5 = d6 = 2, ..., d106 = d107 = d108 = 36. Soluci´ on del problema 2. Primero veamos que n es par. Si n fuera impar entonces cada ai deber´a ser impar, pero la suma de un n´umero impar de impares no puede ser igual a cero. Luego n es par. Ahora veamos que n es m´ultiplo de 4. Si n fuera de la forma n = 4m + 2 = 2(2m + 1), entonces alguna ai es 2 y las restantes n − 1 son impares, luego, la suma de las ai impares, es impar (pues n − 1 es impar) y si agregamos el 2 tenemos que a1 + a2 + · · · + an es impar, luego, no podr´a ser 0. Por tanto n debe ser de la forma 4m. 22 Soluciones Problemas de Pr´ actica Soluci´ on del problema 3. Hay 105. Primero observemos que: 111, 222, 333, ..., 999 son equilibrados. Notemos tambi´en que si abc es un n´umero equilibrado entonces tambi´en lo son: acb, bac, bca, cab y cba, adem´as si dos cifras de un n´umero equilibrado son iguales la tercera cifra es igual a estas, por lo que los n´umeros equilibrados de cifras diferentes aparecen de 6 en 6. Finalmente, hay que tener en cuenta que si abc es equilibrado con c = a+b entonces a y b deben 2 tener la misma paridad. Construimos ahora los n´umeros equilibrados siguiendo un orden, primero los que tienen la primera cifra igual a 1, despu´es cuando es 2, etc. 132, 153, 174, 195 243, 264, 285, 354, 375, 396, 465, 486, 576, 597, 687, 798. Por la observaci´on anterior estos 16 n´umeros equilibrados, generan a 16 · 6 = 96 n´umeros y con los 9 primeros, dan los 105 n´umeros equilibrados. Soluci´ on del problema 4. Coloreamos el tablero con dos colores: blanco y negro en forma alternada, de manera que queden 13 negros y 12 blancos. Como una pieza de 2 × 1 cubre un cuadro blanco y uno negro, al cubrir el tablero con 12 de estas piezas nos quedar´a sin cubrir un cuadro negro. Veamos que cualquiera de ellos puede quedar sin cubrir. Dos renglones contiguos siempre se pueden cubrir, as´ı el cuadrado de 4 renglones y 5 columnas que se obtiene al quitar el primer rengl´on se puede cubrir y en el primer rengl´on se pueden dejar de cubrir cualquiera de los 3 cuadros negros. Este m´etodo funciona tambi´en para el tercer y quinto rengl´on. Fij´emonos en el cuadro de 3 × 3 que est´a en la esquina superior izquierda, ´este se puede cubrir dejando el centro descubierto de la siguiente manera: 3.1 Soluciones de los Problemas de Pr´ actica 23 Como el resto del cuadrado de 5×5 se cubre, entonces tenemos manera de cubrir el de 5 × 5 dejando el cuadrado negro central del subcuadrado de 3 × 3 que seleccionamos. En forma an´aloga, se procede para los cuadros negros restantes. Soluci´ on del problema 5. Los trapecios de lados 1, 1, 1 y 2 son el resultado de “pegar”3 tri´angulos equil´ateros de lado 1 formando la mitad de un hex´agono regular. Un tri´angulo equil´atero de lado n se divide en n2 triangulitos de lado 1. Por lo tanto, para que un tri´angulo de lado n se pueda dividir en trapecios como los mencionados, necesariamente n2 deber´a ser m´ultiplo de 3, es decir, n debe ser m´ultiplo de 3. Esto tambi´en es suficiente: si n es m´ultiplo de 3, el tri´angulo equil´atero de lado n puede dividirse en tri´angulos equil´ateros de lado 3 y basta probar que uno de ´estos puede dividirse en trapecios de lados 1, 1, 1 y 2. La divisi´on necesaria se muestra en la figura: Soluci´  on del problema 6. Los icosaedros tienen 12 v´ertices, por lo que hay 12 = 66 segmentos que unen pares de v´ertices. Si a esos 66 segmentos les 2 quitamos las aristas, tendremos el n´umero de diagonales. ¿Cu´antas aristas tiene un icosaedro regular? Veamos: tiene 20 caras de 3 aristas cada una y cada arista est´a en dos caras. Por lo tanto, hay 20·3 = 30 aristas. Entonces, hay 66−30 = 36 2 diagonales. Segunda Soluci´ on. Usaremos la f´ormula de Euler que relaciona el n´umero de aristas A, el n´umero de v´ertices V y el n´umero de caras C de un poliedro: V − A + C = 2. 24 Soluciones Problemas de Pr´ actica Como en el icosaedro las caras son tri´angulos tenemos que 2A = 3C ya que cada cara tiene tres aristas y una arista es compartida por dos caras, luego hay 30 aristas. Por la f´ormula de Euler el n´umero de v´ertices es V = A − C + 2 = 30 − 20 + 2 = 12 y entonces se tiene que 5V = 2A, es decir, cada v´ertice tiene 5 v´ertices adyacentes. Al unir cada v´ertice con los 11 v´ertices restantes y al restar sus 5 v´ertices adyacentes, tenemos que de cada v´ertice salen 6 diagonales luego el n´umero de diagonales es 12·6 = 36. 2 Soluci´ on del problema 7. No es dif´ıcil ver que si el n´umero tiene 90 cifras entonces tiene 12 cifras iguales a a y 78 cifras 2. Como un n´umero es divisible entre 9 si y s´olo si la suma de sus cifras es divisible entre 9 y como la suma de las cifras 2 es 2 · 78 que es divisible entre 9, bastar´a ver cuando 12a es divisible entre 9. Como 12a tiene un factor 3, ser´a suficiente que a tenga un factor 3, luego los valores de a son 0, 3, 6 y 9. Soluci´ on del problema 8. Como: a + 1b a + b−1 = = 1 a−1 + b +b a ab+1 b  ab+1 a  a = , b debemos encontrar los a y b tales que ab = 13 y a + b ≤ 80. Como ab = 13, a = 13b, entonces, a + b = 14b ≤ 80. Por lo tanto, b es a lo m´as 5. Las parejas (a, b) que cumplen son (13, 1), (26, 2), (39, 3), (52, 4), (65, 5). Soluci´ on del problema 9. Por el Teorema de la bisectriz: BN MN = AM AB BC PC = QB QP y Los tri´angulos ABN y QBC, son semejantes ya que ∠BAN = ∠BQC y N ∠ABN = ∠QBC = 2θ, por lo tanto, BN = BC . Luego M = PQPC . AB QB AM A Q M P θ θ B θ N C 3.1 Soluciones de los Problemas de Pr´ actica 25 Al sumar 1 de ambos lados de esta ´ultima igualdad obtenemos: QC AN = AM QP Por tanto, AM CP QP CP QC + = + = = 1. AN CQ QC CQ QC Soluci´ on del problema 10. Sea Q la intersecci´on de OP con CD. Probar que O, P y H son colineales es equivalente a probar que P Q⊥CD. Sea α = ∠BAC = ∠BDC. Como O es el circuncentro de ABP , ∠BOP = 2α. A D α P α H O Q 2α B C Como el tri´angulo BOP es is´osceles, ∠OP B = 90o − 21 ∠BOP = 90o − α. Entonces ∠DP Q + ∠P DQ = ∠OP B + α = (90o − α) + α = 90o . Por lo tanto, ∠P QD = 90o , es decir, P Q⊥CD. Soluci´ on del problema 11. Fij´emonos en la forma de llenar el cuadrito central de 2 × 2, cada una de las formas de llenarlo dar´a una forma de llenar toda la cuadr´ıcula. El cuadrado central se puede llenar con dos fichas de dos maneras: y Para cada una de estas formas hay 2 maneras de terminar el llenado de la cuadr´ıcula: 26 Soluciones Problemas de Pr´ actica As´ı hay 4 maneras de llenar la cuadr´ıcula, si iniciamos llenando primero el cuadrado central de 2 × 2. Si en el cuadrado central de 2 × 2 hay solo una ficha completa dentro de ´el, por ejemplo as´ı: Tenemos que los dos cuadrados de 2 × 2 de la derecha tanto el inferior como el superior se pueden llenar cada uno de 2 maneras y el lado izquierdo de la cuadr´ıcula que falta llenar, se llena de manera forzada. Luego, hay 4 maneras de llenar la cuadr´ıcula en este caso y si giramos la cuadr´ıcula para considerar los 4 posibles casos de la ficha de 2×1 que se coloca en el cuadro central, tenemos en total en este caso 16 maneras de llenarla. Finalmente veamos el caso donde no hay una ficha de 2 × 1, dentro del cuadrado central. Dividimos la cuadr´ıcula en 4 cuadros de 2 × 2. Cada uno de estos cuadrados de 2 × 2 se puede llenar de 2 maneras y entonces en este caso hay 16 maneras de llenar la cuadr´ıcula. 3.1 Soluciones de los Problemas de Pr´ actica 27 Resumiendo tenemos que hay en total 36 formas de llenar la cuadr´ıcula. Soluci´ on del problema 12. El reo n´umero 10, que es el primero en hablar, ve 9 sombreros de dos colores posibles. De un color debe haber un n´umero par y del otro uno impar. Los reos se pueden poner de acuerdo para que el reo n´umero 10 diga el color del que ve un n´umero par, de esta forma el reo n´umero 9 sabr´a de que color es su sombrero, ya que cuenta el n´umero de sombreros que ve del color que dijo su compa˜nero: si sigue siendo par, su sombrero es del otro color, y si es impar, su sombrero es de ese color. As´ı cada uno de los siguientes reos cuenta el n´umero de sombreros que ve del color que dijo el reo n´umero 10 y a ese n´umero le suma la cantidad de veces que sus otros compa˜neros han dicho el color, si el n´umero que le da es par, entonces su sombrero es del otro color y si es impar, su sombrero es de ese color. De esta forma el ´unico que quiz´a no se salva es el reo n´umero 10. Soluci´ on del problema 13. Supongamos que 1 es rojo y que n es negro. Para n = 2 es claro el resultado, supongamos el resultado cierto para k < n. Sean a el menor entero coloreado de negro y b el mayor entero coloreado de rojo de entre {1, 2, ..., n}, as´ı 1, 2, ..., a − 1 son rojos y b + 1, b + 2, ..., n son negros. Ahora, fij´emonos en el conjunto {a, a + 1, ..., b} tenemos que inicia con a coloreado de negro y termina en b que es de color rojo, luego, por la hip´otesis de inducci´on el n´umero de parejas consecutivas de diferente color en este conjunto es impar. Sumando los dos cambios de color que hay entre a − 1 y a y entre b y b + 1, el total tambi´en es impar. Soluci´ on del problema 14. Supongamos que el cuadro 1 se llena con una ficha de domin´o vertical (ver figura). 1 2 3 4 5 6 7 8 9 10 11 12 13 14 Si 2 tambi´en se llena con una ficha vertical, terminamos. Supongamos entonces que 2 se cubre con una ficha horizontal. Ahora, si 3 se cubre con una horizontal 28 Soluciones Problemas de Pr´ actica acabamos; supongamos que se cubre con una vertical. Del mismo modo vemos que 1, 3, 5, 7, 9, 11 y 13 se deben llenar con fichas verticales y 2, 4, 6, 8, 10, 12 con fichas horizontales. Esto obliga a cubrir 14 con una ficha vertical formando un cuadrado de 2 × 2 con la ficha que cubre 13. Por lo tanto, siempre hay dos fichas que forman un cuadro de 2 × 2. Segunda Soluci´ on. Supongamos que no hay 2 fichas que formen un cuadrado de 2 × 2. Contaremos el n´umero de parejas (C, D) en el tablero donde C es un cuadro de 2 × 2 y D es una ficha de domin´o contenida completamente en C. Como cada cuadro de 2 × 2 contiene a lo m´as una ficha, el n´umero de parejas es a lo m´as 49, el n´umero de cuadros de 2 × 2 en un tablero de 8 × 8. Por otra parte, cada ficha que no est´e contenida totalmente en el marco de ancho 1, est´a contenida en dos cuadrados de 2 × 2. Por lo tanto, si hay m fichas en el marco, hay m + 2(32 − m) = 64 − m parejas de las que contamos. Entonces, 64 − m es menor o igual que 49 y por lo tanto m ≥ 15. Pero no puede haber 15 fichas en el marco, pues el marco tiene s´olo 28 cuadritos. Esta contradicci´on prueba que debe haber 2 fichas en un cuadro de 2 × 2. Soluci´ on del problema 15. Hay tres tipos de paralelep´ıpedos: (i) paralelep´ıpedos con todas las aristas iguales (cubos), de ´estos hay 10, que correspondende a la manera de elegir la longitud de la arista entre los 10 n´umeros. (ii) paralelep´ıpedos con dos longitudes de aristas iguales y una distinta, hay 10 formas de elegir la longitud de las aristas iguales y 9 formas de elegir la otra longitud, por lo que hay en ´este caso 10 · 9 = 90 paralelep´ıpedos distintos. (iii) paralelep´ıpedos con tres longitudes de aristas distintas, aqu´ı se deben  escoger las tres longitudes distintas 10 entre las 10 posibles estos se hace de 3 = 120 maneras. Por lo que en total hay 10 + 90 + 120 = 220 paralelep´ıpedos distintos. Segunda Soluci´ on. Ordenando los lados del paralelep´ıpedo de menor a mayor evitamos repeticiones. Entonces, el n´umero de paralelep´ıpedos rectangulares distintos con lados enteros entre 1 y 10 es igual al n´umero de tercias de enteros (a, b, c)  que satisfacen 1 ≤ a ≤ b ≤ c ≤ 10. Afirmamos que el n´umero de tercias es 12 , el n´umero de subconjuntos de 3 elementos del conjunto 1, 2, 3, . . . , 12. De 3 la condici´on que deben cumplir las tercias se obtiene 1 ≤ a < b+ 1 < c + 2 ≤ 12, por lo que a, b + 1 y c + 2 son n´umeros del 1 al 12 diferentes entre s´ı. Entonces, a cada tercia (a, b, c) le podemos hacer corresponder el conjunto {a, b + 1, c + 2}. Y viceversa, para obtener una de las tercias a partir de cualquier subconjunto de 3 elementos de {1, 2, 3, . . . , 12}, basta con ordenar los elementos del conjunto, digamos x < y < z y formar la tercia (x, y − 1, z − 2). Como x, y y z est´an entre 3.1 Soluciones de los Problemas de Pr´ actica 29 1 y 12 y se cumple que x < y < z, tenemos que 1 ≤ x ≤ y − 1 ≤ z − 2 ≤ 10, de modo que la tercia propuesta efectivamente cumple la condici´on requerida. Esta correspondencia muestra que el n´umero de tercias que cumplen la condici´on (que es igual al de paralelep´ıpedos buscados) es igual al n´umero de subconjuntos  12 de {1, 2, 3, . . . , 12} de 3 elementos, es decir, 3 . P P P Soluci´ on del problema 16. S´ı es posible. Para ver esto, sean c, v y a las sumas de los n´umeros asignados a las caras, v´ertices y aristas, respectivamente. Como cada cara est´a formada por tres aristas, tenemos que: 2 X a=3 X a=3 X c X v (3.1) y como cada v´ertice es com´un a tres aristas, tenemos que: 2 Adem´as, X c+ X v+ P c= X a = 1 + 2 + · · · + 14 = 105. (3.2) (3.3) Resolviendo el sistema formado por (3.1), (3.2) y (3.3) tenemos que: P v = 30 y P a = 45. Para que el n´umero asignado a una arista sea entero, debemos tener que los n´umeros asignados a las caras adyacentes a la arista deben ser de la misma paridad y los n´umeros asignados a los v´ertices que determina la arista tambi´en deben ser de la misma paridad. Pero si un v´ertice es par (impar) lo ser´an tambi´en los dem´as v´ertices, esto mismo sucede con las caras. Los conjuntos de cuatro n´umeros con la misma paridad, cuya suma sea 30 y sean elementos de {1, 2, 3, ..., 14} , son los siguientes ocho conjuntos: {1, 5,11,13}, {1, 7, 9, 13}, {3, 5, 9, 13}, {3, 7, 9, 11}, {2, 4, 10, 14}, {2, 6, 8, 14}, {2, 6, 10, 12}, {4, 6, 8, 12}. Uno de estos conjuntos debe ser usado para numerar las caras y otro para numerar los v´ertices. Puesto que ning´un promedio debe ser elemento del conjunto, los u ´nicos conjuntos que satisfacen (despu´es de promediar por parejas los n´umeros de cada conjunto de todas las maneras posibles) son: {1, 5, 11, 13} y {2, 4, 10, 14}. Ambos dan el mismo conjunto para numerar las aristas. A saber: {3, 6, 7, 8, 9, 12} . 30 Soluciones Problemas de Pr´ actica As´ı, tenemos dos soluciones dependiendo de qu´e conjunto se utilice para numerar las caras y cual para numerar los v´ertices. Notemos que cada soluci´on es el dual de la otra, ya que en la primera los n´umeros asignados a las caras son los n´umeros asignados a los v´ertices de la segunda y viceversa. Soluci´ on del problema 17. Supongamos que los n´umeros primos son: p, p + 6, p + 12, p + 18, p + 24 al tomar congruencias m´odulo 5 tenemos que estos n´umeros son congruentes a p, p + 1, p + 2, p + 3, p + 4 respectivamente, pero entre cinco n´umeros consecutivos siempre hay uno que es divisible entre 5 y como p debe ser primo no hay otra opci´on m´as que p = 5, y los n´umeros primos son 5, 11, 17, 23 y 29. Soluci´ on del problema 18. El tri´angulo A1 B1 C1 se divide en los tri´angulos AA1 B1 , AA1 C1 y AB1 C1 . Como BB1 es paralela a AA1 , (AA1 B1 ) = (AA1 B) (si pensamos que la base de ambos tri´angulos es AA1 , tiene tambi´en la misma altura: la distancia de AA1 a BB1 ). B1 C1 A B A1 C An´alogamente, (AA1 C1 ) = (AA1 C). Por lo tanto, (AA1 B1 ) + (AA1 C1 ) = (AA1 B) + (AA1 C) = (ABC). Veamos ahora el tri´angulo AB1 C1 : como BB1 es paralela a CC1 , (BB1 C1 ) = (BB1 C). Restando (BB1 A) a cada lado de esa igualdad se obtiene que (AB1 C1 ) = (ABC). Por lo tanto, (A1 B1 C1 ) = (AB1 C1 ) + ((AA1 B1 ) + (AA1 C1 )) = 2(ABC) = 3998. y Soluci´ on del problema 19. (i) Si n es impar, tomamos a = 0, b = n+1 2 n−1 n−4 c = 2 . Si n es m´ultiplo de cuatro, tomamos a = 0, b = n+4 y c = . 4 4 n−2 n Finalmente, si n es de la forma 4k + 2, tomamos a = 1, b = 2 y c = 2 . 3.1 Soluciones de los Problemas de Pr´ actica 31 (ii) Probaremos primero que n = a2 + b2 + c2 no tiene soluciones enteras si n es de la forma 4t (8k + 7). Un cuadrado perfecto es congruente con 0, 1 o´ 4 m´odulo 8. Haciendo todas las posible sumas m´odulo 8 de tres 0′ s, 1′ s y 4′ s, vemos que a2 + b2 + c2 nunca es congruente con 7 m´odulo 8. Tambi´en podemos comprobar que no es m´ultiplo de 4 (o sea, congruente con 0 ´o 4 m´odulo 8) a menos que a, b y c sean los tres pares. Por lo tanto, si a2 + b2 + c2 = 4t (8k + 7), no podemos tener t = 0. Pero si t ≥ 1, a, b y c deben ser pares, digamos 2a1 , 2b1 y 2c1 . Sustituyendo hallamos que a21 + b21 + c21 = 4t−1 (8k + 7), que es la misma ecuaci´on con t reducida en 1. Repetiendo el argumento suficientes veces llegamos a t = 0, lo cual es absurdo. Por lo tanto, n = a2 + b2 + c2 no tiene soluciones si n es de la forma 4t (8k + 7). Contemos cuantas n < 1999 son de esa forma: Con t = 0 hay 249, k = 0, 1, . . . , 248. Con t = 1 hay 62, k = 0, 1, . . . , 61. Con t = 2 hay 15, k = 0, 1, . . . , 14. Con t = 3 hay 4, k = 0, 1, 2, 3. Con t = 4 hay 1, k = 0. En total son 331 n´umeros de esa forma, por lo que hay a lo m´as 1998−331 = 1667 enteros menores que 1999 que son suma de tres cuadrados. √ Soluci´ on del problema 20. Ponemos en cada v´ertice indicado con A, 2 y en cada v´ertice indicado con B, √12 . Ahora, sobre cada lado, el n´umero escrito es el producto de los n´umeros puestos en los v´ertices. El producto de los n´umeros escritos sobre los lados es el cuadrado del producto de los n´umeros asignados a los v´ertices,pues cada v´ertice  pertenece a dos lados. Por lo tanto, el producto √ a  1  b 2 √ = 2a−b . buscado es 2 2 Soluci´ on del problema 21. Como MN y AC son perpendiculares a MB, se tiene que MN k AC. Sea P en MN tal que CP ⊥MN, entonces P MEC es un rect´angulo, P C = ME = AD y ∠MP C = π/2 = ∠ECP. 32 Soluciones Problemas de Pr´ actica M E A B P D N C Como P NkCE se tiene que ∠DCE = ∠CNP , y entonces los tri´angulos P NC y DCA tienen todos sus ´angulos iguales y un lado igual, por tanto son congruentes y entonces NC = CA. Luego el tri´angulo NCA es is´osceles. Soluci´ on del problema 22. Los tri´angulos ALN y ACL son semejantes, por lo que: AL2 = AN · AC. A D N M B L C Tambi´en ALM y ABL son semejantes por lo que: AL2 = AM · AB. Luego AL4 = AB · AC · AN · AM. (3.4) Por otro lado el cuadril´atero AMLN es c´ıclico ya que ∠MAN +∠MLN = 180◦, luego los ´angulos ∠AMN y ∠ALN son iguales y entonces son semejantes los tri´angulos ADM y ANL, por lo que: AD · AL = AN · AM. (3.5) De las ecuaciones (3.4) y (3.5) tenemos que AL3 = AB · AC · AD. Soluci´ on del problema 23. Sea G el centroide, es decir, el punto de intersecci´on de las medianas AL, BM y CN. 3.1 Soluciones de los Problemas de Pr´ actica 33 A N M G L B C Como LN es paralela a CA, tenemos que ∠LAC = ∠ALN. Luego, ∠MBA = ∠LAC = ∠ALN ⇔ BLGN es c´ıclico ⇔ ∠CNA = ∠ALB. Soluci´ on del problema 24. Observemos primero que a(a + b) − ab ab a2 = =a− . a+b a+b a+b  a+b 2 , 2 ab ab tenemos que a+b ≤ a+b , luego − a+b ≥ − a+b . Por lo tanto, 4 4       bc ca b2 c2 ab a2 + b− + c− + + = a− a+b b+c c+a a+b b+c c+a   a+b b+c c+a ≥ (a + b + c) − + + 4 4 4 a+b+c = = 1. 2 Como ab ≤ Soluci´ on del problema 25. Sea ∠ABC el ´angulo recto. Por construcci´on BR = BC y BP = AB, adem´as como ∠ABC = π/2 = ∠RBP, los tri´angulos ABC y P BR son congruentes y de lados paralelos, por lo que AC y P R son iguales y paralelos. Como BQ y AC son perpendiculares, resulta que BQ y RP son perpendiculares. Q A B R D C E P 34 Soluciones Problemas de Pr´ actica Sean D y E las intersecciones de BQ con AC y RP , respectivamente. QE es altura del tri´angulo P QR. Por construcci´on 2 BE = BQ. Calculemos el ´area del tri´angulo P QR: (P QR) = 12 RP · QE = 23 AC · BD = 3 (ABC). Entonces 1 (ABC) = . (P QR) 3 Soluci´ on del problema 26. Consideremos el hex´agono ABCDEF coloreado como indica el problema. H A F G B C E D I Sobre los lados AF y CD construimos tri´angulos equil´ateros AF H y CID respectivamente, como se ve en la figura. En el tri´angulo BGH tenemos que (AHG) = (BAG) por tener la misma base y la misma altura. An´alogamente, (CBG) = (CIG), (DIG) = (EDG) y (F EG) = (HF G). Luego, (AHF G) + (EDG) + (CBG) = (CGDI) + (BAG) + (F EG). Si restamos de esta ecuaci´on las ´areas que no interesan (los tri´angulos equil´ateros que construimos al principio), tenemos que: (AHF G)−(AF H)+(EDG)+(CBG) = (CGDI)−(CID)+(BAG)+(F EG), De donde, (AF G) + (EDG) + (CBG) = (DCG) + (BAG) + (F EG). Soluci´ on del problema 27. La estrategia que deber´a seguir el primer jugador es repetir la ´ultima jugada del segundo jugador. Para explicar esto, s´olo basta colorear el tablero de la siguiente manera: 3.1 Soluciones de los Problemas de Pr´ actica 35 As´ı, con la estrategia descrita, como el primer jugador siempre comienza en una casilla coloreada, (por la paridad del tablero) podemos asegurar que siempre llegar´a en sus turnos a una casilla de las que quedaron coloreadas, y que llegar´a primero a la casilla superior derecha del tablero, puesto que est´a coloreada tambi´en. El segundo jugador solamente podr´a llegar en sus turnos a las casillas blancas, jam´as podr´a llegar a las casillas coloreadas. Soluci´ on del problema 28. Sea n, n + 1, . . . , n + 9 diez enteros consecutivos. Si dos de ellos, n + i, n + j (0 ≤ i < j ≤ 10), tiene divisor primo p en com´un, entonces p|(n + j) − (n + i) = j − i. Ahora bien, j − i est´a entre 1 y 8, de modo que p s´olo puede ser 2, 3, 5 ´o 7. Por lo tanto, si (n + i) no es m´ultiplo de 2, 3, 5 ´o 7, (n+i) ser´a primo relativo con los otros nueve enteros consecutivos. Entonces basta para cada n dar una i, 0 ≤ i < 10, tal que (n + i) no sea m´ultiplo de 2, 3, 5 o´ 7. Para eso s´olo necesitamos tomar en cuenta el n = 0, 1, 2, 3, . . . , 209. Si n es un primo mayor que 7, podemos tomar i = 0. Eso nos da una i para n = 11, 13, 17, 19, 23, 29, 31, 37, 41, 43, 47, 53, 59, 61, 67, 71, 73, 79, 83, 89, 97, 101, 103, 107, 109, 113, 127, 131, 137, 139, 149, 151, 157, 163, 167, 173, 179, 181, 191, 193, 197199. Si i = 0 funciona para n = m, entonces i = k funciona para n = m − k con 0 < k < 10, por eso, donde en la lista anterior los “huecos”son de longitud 10 o menos podemos hallar una i apropiada para el “hueco” n. Hay s´olo 3 “huecos”de longitud mayor que 10: del principio al 11, del 113 al 127, del 199 al final. Los podemos llenar notando que i = 0 funciona tambi´en para n = 1, 121, 209. Esto concluye la prueba. Soluci´ on del problema 29. Cada punto X del plano lo giramos 60◦ alrededor de B (en el sentido de las manecillas del reloj) para obtener X ′ , por ejemplo, C = A′ . 36 Soluciones Problemas de Pr´ actica A D P B P′ E M C D′ M′ Por la construcci´on BDD ′ es equil´atero. Probaremos que E es el punto medio de DD ′, de modo que los ´angulos del tri´angulo BDE son 30◦ , 60◦ y 90◦ . Si probamos que P DCD ′ es un paralelogramo habremos terminado (porque las diagonales de un paralelogramo se bisecan). P ′D ′ forma un ´angulo de 60◦ con P D, as´ı que ∠P ′D ′ C = 120◦ y P DkD ′C. Adem´as, es claro que P D = D ′ C. Por lo tanto, P DCD ′ es un paralelogramo, que es lo que nos faltaba probar. Soluci´ on del problema 30. Si k es el elemento mayor de un subconjunto de A de 7 elementos de {1, 2, 3, . . . , 10}, entonces A \ {k} ⊂ {1, 2, 3, . . . , k − 1}. Por lo tanto, hay k−1 subconjuntos de {1, 2, 3, . . . , 10} cuyo m´aximo elemento es 6  k. Cada uno de esos k−1 subconjuntos contribuye k a la suma, por lo que la 6 suma deseada es:  X    10 10 10 10   X X X (k − 1)! k−1 11 k! k k = k . =7 7 = =7 6 8 7 6!(k − 7)! k=1 7!(k − 7)! k=1 k=7 k=7   P k n+1 (Aqu´ı usamos la f´ormula nk=m m = m+1 . Esto se puede probar viendo que  n+1 es el n´umero de subconjuntos de m + 1 elementos de {1, 2, 3, . . . , n + 1} m+1  k y que m cuenta cuantos de ellos tienen como m´aximo elemento a k + 1). Soluci´ on del problema 31. Hay 2n subconjuntos de N incluyendo el conjunto vac´ıo, 2n−1 de los cuales contienen a n y 2n−1 que no contienen a n. Adem´as se 3.1 Soluciones de los Problemas de Pr´ actica 37 corresponden: si A es un conjunto que tiene a n, el conjunto A\n no lo contiene. La suma S de un conjunto A que contiene a n es de la forma S = n−a+b−c . . . y la suma S ′ = a − b + c . . .. Luego S + S ′ = n y hay 2n−1 de estas sumas. Por lo tanto, la suma total es n · 2n−1 . Soluci´ on del problema 32. Sea a = AB = CD = DE la longitud del lado, b = AC = CE la longitud de la diagonal menor y c = AD = AE la longitud de la diagonal mayor. Por el teorema de Ptolomeo aplicado al cuadril´atero ACDE, tenemos que ab + ac = bc, ecuaci´on que al dividir entre abc, nos da el resultado. Soluci´ on del problema 33. Dividamos el cuadril´atero AF DE por la diagonal AD, formando dos tri´angulos de ´areas a y b que determinaremos. Usando el hecho de que si dos tri´angulos tienen la misma altura, la raz´on de sus ´areas es igual a la raz´on de sus bases, tenemos que: AE a a+b+3 = = EC 3 12 y AF b a+b+3 = = FB 3 12 Estas identidades nos llevan al sistema de ecuaciones: 3a = b + 3 3b = a + 3 que admiten por soluci´on, a = b = 32 , por lo que ´area (AF DE) = 3. Soluci´ on del problema 34. Veamos que no es posible. Los cuadros de las esquinas ser´an pares, cuando un tablero de 2 × 2 se coloque en las esquinas un n´umero impar de veces. El n´umero del cuadro central se modifica con cualquier tablero de 2×2, luego si las esquinas se modifican para llegar a ser n´umeros pares, el central se modificar´a aumentando al n´umero 1, los cuatro n´umeros impares, lo que dar´a al final un impar, no as´ı un n´umero par como desear´ıamos. Segunda Soluci´ on. Inicialmente la suma de todos los n´umeros de la cuadr´ıcula es 5, que es impar. Cada vez que se aplica la regla para modificar la cuadr´ıcula, la suma aumenta en 4 y entonces el nuevo n´umero es
Author:  Elena Fidalgo Rojo

0 downloads 111 Views 420KB Size

Recommend Stories


Problemas para la. 18 a Olimpiada Mexicana de Matemáticas. Luis Miguel García Velázquez Julio César Aguilar Cabrera
Problemas para la 18a Olimpiada Mexicana de Matem´aticas Luis Miguel Garc´ıa Vel´azquez Julio C´esar Aguilar Cabrera Mar´ıa Luisa P´erez Segu´ı Mar´ı

Olimpiada Mexicana de Matemáticas
Olimpiada Mexicana de Matemáticas http://ommenlinea.org/ 2013-2014 ii Marco Antonio Figueroa Ibarra Matemorfosis del CIMAT José Alfredo Cobián Camp

OLIMPIADA MEXICANA DE MATEMÁTICAS
OLIMPIADA MEXICANA DE MATEMÁTICAS DELEGACIÓN QUERÉTARO APUNTES DE GEOMETRÍA PARA OLIMPIADA POR: MARÍA DEL ROSARIO VELÁZQUEZ CAMACHO AGOSTO 2006 1

Problemas de las Olimpiadas de Matemáticas Tornamira. Problemas propuestos en la XIII Olimpiada Tornamira
Problemas de las Olimpiadas de Matemáticas “Tornamira”. Navarra Problemas propuestos en la XIII Olimpiada Tornamira 1.- Daniel y Mikel están sentado

Problemas de las Olimpiadas de Matemáticas Tornamira. Problemas propuestos en la VII Olimpiada Tornamira
Problemas de las Olimpiadas de Matemáticas “Tornamira”. Navarra Problemas propuestos en la VII Olimpiada Tornamira 1.- Recortando en cada vértice de

Meteoritos a la mexicana
Meteorito Bacubirito Imagen: http://upload.wikimedia.org/wikipedia/commons/5/5f/PSM_V69_D026_Bacurbito_side_view.png Meteoritos a la mexicana Octavio

TZALOA Revista de la Olimpiada Mexicana de Matemáticas Año 2013, No. 3
TZALOA Revista de la Olimpiada Mexicana de Matem´aticas ˜ 2013, No. 3 Ano Comit´e Editorial: Anne Alberro Semerena Marco Antonio Figueroa Ibarra Carl

Story Transcript

Problemas para la 18a Olimpiada Mexicana de Matem´ aticas (Avanzado)

Editado por:

Anne Alberro Semerena Radmila Bulajich Manfrino Jos´ e Antonio G´ omez Ortega

2004

Anne Alberro Semerena Facultad de Ciencias, Universidad Aut´onoma del Estado de Morelos. Radmila Bulajich Manfrino Facultad de Ciencias, Universidad Aut´onoma del Estado de Morelos. Jos´ e Antonio G´ omez Ortega Facultad de Ciencias, Universidad Nacional Aut´onoma de M´exico.

Presentaci´ on

La Sociedad Matem´atica Mexicana organiza la 18a Olimpiada Mexicana de Matem´aticas. Los ganadores en ella formar´an la preselecci´on que se preparar´a para las las distintas olimpiadas internacionales del a˜no 2005: la XLVI Olimpiada Internacional, que se llevar´a a cabo en M´exico durante el mes de julio, la XX Olimpiada Iberoamericana a celebrarse en septiembre en Ecuador y la VII Olimpiada de Centroam´erica y el Caribe que se llevar´a a cabo en Honduras en el mes de julio. En la 18a Olimpiada Mexicana de Matem´aticas pueden participar los estudiantes de M´exico nacidos despu´es del 1o de agosto de 1985. Los concursantes deber´an estar inscritos en una instituci´on preuniversitaria durante el primer semestre del c´ıclo escolar 2004 - 2005 y para el 1o de julio del a˜no 2005 no deber´an haber iniciado estudios de nivel universitario. Los problemas que se presentan en este folleto aparecieron en las primeras etapas de las Olimpiadas Estatales de Matem´aticas en M´exico. La intenci´on de este folleto es que sirva como orientaci´on a los alumnos que desean participar en estas Olimpiadas. Como se puede ver, los problemas que aparecen aqu´ı, no son ejercicios rutinarios o problemas en los que se apliquen directamente los conocimientos que se adquieren en la escuela. M´as bien son problemas que requieren de una buena dosis de ingenio y de esfuerzo para ser resueltos. Como en todos los aspectos del aprendizaje de las matem´aticas, el esfuerzo individual y el enfrentamiento solitario con los problemas son importantes, pero tambi´en es muy importante la discusi´on con los compa˜neros y los profesores. Una forma de manifestar creatividad en matem´aticas es resolviendo problemas. Otra forma, que a veces requiere de m´as madurez, es invent´andolos. Invitamos a todos los lectores de este folleto: profesores, estudiantes, ol´ımpicos y exol´ımpicos a que nos env´ıen problemas con soluci´on. Las aportaciones ser´an consideradas

iv

Etapas de la Olimpiada

para su inclusi´on en ex´amenes o en futuros folletos. Esta publicaci´on incluye una seleccci´on de los problemas que formaron parte de los folletos avanzados elaborados como gu´ıas para la 13a y 14a olimpiada mexicana de matam´aticas. El primero editado por: Omar Antol´ın, Radmila Bulajich, Carlos Cabrera y Jos´e Antonio G´omez; el segundo por: Omar Antol´ın, Radmila Bulajich, Jos´e Antonio G´omez y Rita V´azquez.

Etapas de la Olimpiada Como ya es tradici´on, la Olimpiada Mexicana de Matem´aticas consta de tres etapas: Ex´ amenes Estatales. Estos ex´amenes servir´an para formar las selecciones estatales que asistir´an al Concurso Nacional. Concurso Nacional. Este concurso se llevar´a a cabo en el Estado de M´exico en noviembre de 2004 y en ´el se elegir´a a la preselecci´on mexicana. Entrenamientos. A los alumnos de la preselecci´on que surjan del Concurso Nacional se les entrenar´a intensivamente durante el primer semestre del a˜no 2005. Tambi´en se les aplicar´an ex´amenes para determinar a los que representar´an a M´exico en las Olimpiadas Internacionales. La participaci´on en las tres etapas mencionadas es individual.

Resumen de Resultados

v

Resumen de Resultados En el a˜no de 1987 la Sociedad Matem´atica Mexicana organiz´o la Primera Olimpiada Mexicana de Matem´aticas. A partir de esa fecha, los concursos nacionales se han celebrado anualmente en las ciudades de Xalapa, Hermosillo, Metepec, Guanajuato, Oaxtepec, La Trinidad, Acapulco, Guadalajara, Colima, M´erida, Monterrey, Quer´etaro, Oaxaca, Morelia, Oaxtepec, Colima y Guanajuato.

Resultados de M´ exico en las Internacionales Los resultados de las Delegaciones Mexicanas en Olimpiadas Internacionales, Iberoamericanas y de Centroam´erica y el Caribe han sido los siguientes: Olimpiada Interncional de Matem´ aticas A˜no 1988 1989 1990 1991 1992 1993 1994 1995 1996 1997 1998 1999 2000 2001 2002 2003

Pa´ıs sede Australia Rep. Fed. de Alemania Rep. Popular de China Suecia Rusia Turqu´ıa Hong Kong Canad´a India Argentina Taiwan Rumania Corea Estados Unidos Escocia Jap´on

No. de pa´ıses 49 50 54 55 56 73 69 74 75 82 75 81 82 83 84 86

Lugar de M´exico 37 31 36 35 49 63 65 59 53 32 44 52 30 46 46 42

En 2003, la delegaci´on que represent´o a M´exico en la Olimpiada Internacional estuvo integrada por los alumnos: Octavio Arizmendi Echegaray (Morelos), Ana Paula Estrada Vargas (Jalisco), Marco Antonio Figueroa Ibarra (Sonora), Yoalli Mabel Hidalgo Pontet (Jalisco), Antonio Olivas Mart´ınez (Sonora) y Carlos Vargas Obieta (Jalisco). Se obtuvieron 3 medallas de bronce (Marco Antonio Figueroa Ibarra, Antonio Olivas Mart´ınez y Ana Paula Estrada Vargas) y una

Resumen de Resultados

vi

menci´on honor´ıfica (Yoalli Mabel Hidalgo Pontet). En total, en las Olimpiadas Internacionales se han obtenido 3 medallas de plata, 21 medallas de bronce y 17 menciones honor´ıficas. Olimpiada Iberoamericana de Matem´ aticas A˜no 1989 1990 1991 1992 1993 1994 1995 1996 1997 1998 1999 2000 2001 2002 2003

Pa´ıs sede Cuba Espa˜na Argentina Venezuela M´exico Brasil Chile Costa Rica M´exico Rep´ublica Dominicna Cuba Venezuela Uruguay El Salvador Argentina

No. de pa´ıses 13 15 16 16 16 16 18 17 17 18 20 21 21 22 19

Lugar de M´exico 3 3 5 6 9 6 9 2 3 5 3 2 3 3 4

Los cuatro integrantes de la delegaci´on mexicana que participaron en la Olimpiada Iberoamerica de 2003 obtuvieron medalla: una de oro (Marco Antonio Figueroa Ibarra de Sonora), medalla de plata (Antonio Olivas Mart´ınez de Sonora) y dos medallas de bronce (Carlos Vargas Obieta de Jalisco y Adri´an Enrique Chi Centeno de Yucat´an). En total, en las Olimpiadas Iberoamericanas. M´exico ha obtenido 10 medallas de oro, 22 medallas de plata, 21 medallas de bronce y 3 menciones honor´ıficas.

Resumen de Resultados

vii

Olimpiada Matem´ atica de Centroam´ erica y el Caribe A˜no 1999 2000 2001 2002 2003

Pa´ıs sede No. de pa´ıses Costa Rica 10 El Salvador 9 Colombia 10 M´exico 8 Costa Rica 11

Lugar de M´exico 2 2 2 1 1

Los 3 alumnos mexicanos en la IV Olimpiada Matem´atica de Centroam´erica y el Caribe, ganaron 2 medallas de oro, ellos fueron Iv´an Josh´ua Hern´andez M´aynez de Coahuila y Gonzalo Arturo Montalv´an G´amez de Puebla y una medalla de plata para el alumno Rosemberg Toala Enr´ıquez de Chiapas. En total en la Olimpiada Matem´atica de Centroam´erica y el Caribe, M´exico ha obtenido 7 medallas de oro, 6 de plata y 2 de bronce.

Resultados del Concurso Nacional de la 17a Olimpiada Mexicana de Matem´ aticas En noviembre de 2003 se llev´o a cabo en Guanajuato, Gto. el 17a Concurso Nacional, con la participaci´on de todos los estados de la Rep´ublica. Los 16 alumnos ganadores del primer lugar fueron: Marco Antonio Figueroa Ibarra (Sonora) H´ector Daniel Garc´ıa Lara (Chihuahua) Iv´an Joshua Hern´andez M´aynez (Coahuila) Rosemberg Toal´a Enr´ıquez (Chiapas) Gonzalo Arturo Montalv´an G´amez (Puebla) Carlos Vargas Obieta (Jalisco) Federico Bribiesca Argomedo (Michoac´an) Rafael D´ıaz Cruz (Distrito Federal) Guillermo Enrique Carro Prado (Nuevo Le´on) Jos´e Miguel Cisneros Franco (Veracruz) Francisco Javier Ibarra Goycoolea (Baja California) Cristos Alberto Ruiz Toscano (Jalisco) Guevara Manuel Angel Guevara L´opez (Zacatecas) Luis Alberto Mart´ınez Chigo (Veracruz) Arturo Aguirre Escobar (Distrito Federal) David Guadalupe Torres Flores (Guanajuato)

Resumen de Resultados

viii

Los 5 alumnos preseleccionados para la Olimpiada Matem´atica de Centroam´erica y el Caribe fueron: Isaac Buenrostro Morales (Jalisco) Iv´an Joshua Hern´andez M´aynez (Coahuila) Jonathan Eliud Rinc´on Galv´an (Nuevo Le´on) Pablo Sober´on Bravo (Morelos) David Guadalupe Torres Flores (Guanajuato) Aunque la participaci´on en el Concurso Nacional es individual, es importante destacar la labor que han llevado a cabo los estados de la Rep´ublica apoyando a sus concursantes. Con el prop´osito de reconocer este trabajo, presentamos el registro de los estados que ocuparon los primeros 10 lugares en el 17a Concurso Nacional. 1. Jalisco 2. Puebla 3. Chihuahua 4. Distrito Federal 5. Sonora 6. Morelos 7. Nuevo Le´on 8. Guanajuato 9. Quer´etaro 10. Veracruz En esta ocasi´on, el premio a la Superaci´on Acad´emica se llam´o Quanaxhuato y fue ganado por el Distrito Federal. El segundo y tercer lugar de este premio lo ocuparon, respectivamente, Veracruz y Tlaxcala.

Informaci´ on sobre la Olimpiada. Para obtener m´as informaci´on sobre los eventos de la Olimpiada Mexicana de Matem´atica o para consultar m´as material de estudio, visita nuestro sitio de Internet: http://erdos.fciencias.unam.mx/omm COMITE ORGANIZADOR DE LA OLIMPIADA MEXICANA DE MATEMATICAS Mayo 2004

Contenido

Presentaci´ on

III

1. Enunciados de los Problemas 1 1.1. Problemas de Pr´actica . . . . . . . . . . . . . . . . . . . . . . . 1 1.2. Problemas de los ´ultimos tres Concursos Nacionales de la OMM . . . . . . . . . . . . . . . . . . . . . . . 10 2. Olimpiadas Internacionales en las que participa M´ exico 15 2.1. V Olimpiada Matem´atica de Centroam´erica y el Caribe . . . . . . 15 2.2. XVIII Olimpiada Iberoamericana de Matem´aticas . . . . . . . . . . . . . . . . . . . . . . . . . . . . 16 2.3. XLIV Olimpiada Internacional de Matem´aticas . . . . . . . . . . . . . . . . . . . . . . . . . . . . 18 3. Soluciones de los Problemas 21 3.1. Soluciones de los Problemas de Pr´actica . . . . . . . . . . . . . 21 3.2. Soluciones de los ´ultimos tres Concursos Nacionales de la OMM . . . . . . . . . . . . . . . . . . . . . . . 49 Bibliograf´ıa

68

x

Contenido

Cap´ıtulo 1 Enunciados de los Problemas

Presentamos aqu´ı algunos problemas para mostrar el tipo de matem´aticas que se manejan en la fase estatal de la Olimpiada Mexicana de Matem´aticas. Al final encontrar´as las soluciones.

1.1.

Problemas de Pr´ actica

Problema 1. Considere 109 enteros con 0 < a1 < ... < a109 < 1999. Muestre que entre los valores di = ai+1 − ai , i = 1, ..., 108 hay un valor que se repite 4 o m´as veces. Encuentre un ejemplo de enteros 0 < a1 < ... < a109 ≤ 1999 donde ninguna diferencia di = ai+1 − ai se repita m´as de 3 veces. Problema 2. Sean a1 , a2 , ..., an enteros con las propiedades de que: (i) a1 · a2 · ... · an = n (ii) a1 + a2 + · · · + an = 0 Muestre que 4 divide a n. Problema 3. Un n´umero de tres cifras es “equilibrado” si una de sus cifras es . el promedio de las otras dos, por ejemplo el 258 es equilibrado pues 5 = 2+8 2 ¿Cu´antos n´umeros equilibrados de tres cifras hay?

2

Problemas de Pr´ actica

Problema 4. Si intentamos cubrir una cuadr´ıcula de 5 × 5 con piezas de tama˜no 2×1, siempre nos quedar´a un hueco. ¿En qu´e sitios de la cuadr´ıcula puede quedar el hueco? Problema 5. ¿Para qu´e enteros positivos n es posible dividir un tri´angulo equil´atero de lado n en trapecios iguales cuyos lados midan 1, 1, 1 y 2? Problema 6. Un icosaedro es un s´olido regular de 20 caras, cada una de las cuales es un tri´angulo equil´atero. ¿Cu´antas diagonales tiene un icosaedro? Problema 7. Utilizando exclusivamente los d´ıgitos 2 y a se forma el siguiente n´umero de 90 cifras. 2a22a222a2222a...,22..,2a. Si el n´umero es m´ultiplo de 9, ¿qu´e valores son posibles para el d´ıgito a? Problema 8. Encuentre todos los enteros positivos a, b tales que: a + b−1 = 13 a−1 + b

y

a + b ≤ 80.

Problema 9. Considerando la figura siguiente muestre que: AM CP + =1 AN CQ A

Q M P

θ θ B

θ

N C

1.1 Problemas de Pr´ actica

3

Problema 10. Sea ABCD un cuadril´atero inscrito en una circunferencia, las l´ıneas BD y AC se cortan en el punto P . Si O es el circuncentro del tri´angulo AP B y H es el ortocentro del tri´angulo CP D, demuestre que O, P y H est´an alineados. Problema 11. ¿De cu´antas formas distintas se puede llenar una cuadr´ıcula de 4 × 4 con fichas de 2 × 1? Problema 12. En una c´arcel hay 10 reos condenados a muerte a los que se les va a dar una ´ultima oportunidad para salvarse: se pondr´an los 10 en una fila y a cada uno le pondr´an un sombrero, ya sea blanco o negro. Cada reo s´olo podr´a ver el color de los sombreros de sus compa˜neros de adelante (no podra ver el suyo ni ninguno de los de atr´as). Se les ir´a preguntando, de uno en uno, empezando por el u ´ltimo de la fila y en orden hasta terminar con el primero: “¿Cu´al cree que es el color de su sombrero?”. Si un reo atina a su color le salvan la vida, si no lo matan. ¿C´omo le pueden hacer los reos para ponerse de acuerdo de tal forma que se salven al menos 9 reos? Problema 13. El conjunto {1, 2, ..., n} se colorea de rojo y negro, de manera que 1 y n reciban diferente color. Muestre que el n´umero de parejas de enteros consecutivos con diferente color es impar. Problema 14. Una cuadr´ıcula de 8 × 8 se cubre con 32 domin´os de 2 × 1. Demuestre que al menos existen dos domin´os que forman un cuadrado de 2 × 2. Problema 15. ¿Cu´antos paralelep´ıpedos rectangulares distintos se pueden construir, para los cuales la longitud de cada arista es un entero del 1 al 10?. Problema 16. ¿Ser´a posible numerar las 4 caras, los 4 v´ertices y las 6 aristas de un tetraedro, es decir, asignarles un n´umero entero del 1 al 14, de tal manera que el n´umero asignado a una arista sea igual al promedio de los n´umeros asignados a los v´ertices que la determinan, e igual al promedio de los n´umeros asignados a las caras que la comparten? Problema 17. Encuentre una lista de cinco primos diferentes donde la diferencia entre cualesquiera dos t´erminos consecutivos de la lista sea siempre seis. Pruebe que esta lista es ´unica.

4

Problemas de Pr´ actica

Problema 18. Sea ABC un tri´angulo escaleno de ´area 1999. Sea A1 un punto del lado BC y sean B1 y C1 puntos sobre las rectas AC y AB respectivamente, tales que AA1 , BB1 y CC1 son paralelas. Encuentre el ´area del tri´angulo A1 B1 C1 . Problema 19. (i) Pruebe que para todo entero positivo n existen tres enteros a, b, c tales que n = a2 + b2 − c2 . (ii) Pruebe que a lo m´as para 1699 enteros positivos n, n < 1999, existen enteros a, b, c tales que n = a2 + b2 + c2 . Problema 20. En el plano se marcan a + b puntos; a de ellos se designan con la letra A y los otros b puntos se designan con la letra B. Al unir los puntos consecutivos se forma un pol´ıgono de a + b lados. Sobre cada uno de los lados hacemos lo siguiente: si los dos v´ertices del lado est´an denotados por letras A, escribimos el n´umero 2; si los dos v´ertices del lado est´an denotados por letras B, escribimos el n´umero 12 ; si los dos v´ertices del lado est´an denotados por letras diferentes, escribimos el n´umero 1. ¿Cu´al es el producto de todos los n´umeros escritos? Problema 21. Sea ABC un tri´angulo, D y E los pies de las alturas desde A y B repectivamente. Sean M en la prolongaci´on de BE tal que EM = AD y N la intersecci´on de la prolongaci´on BC con la perpendicular a BM por M. Demuestre que el tri´angulo NCA es is´osceles. Problema 22. Sea AL la bisectriz del ´angulo A de un tri´angulo acut´angulo ABC. Sean M y N sobre los lados AB y AC respectivamente, de manera que ∠MLA = ∠B y ∠NLA = ∠C. Si D es el punto de intersecci´on de AL y MN, muestre que AL3 = AB · AC · AD. Problema 23. Sean ABC un tri´angulo y L, M, N los puntos medios de los lados BC, CA, AB respectivamente. Muestre que ∠LAC = ∠MBA si y s´olo si ∠CNA = ∠ALB. Problema 24. Si a, b y c son n´umeros positivos con a + b + c = 2, muestre que: a2 b2 c2 + + ≥ 1. a+b b+c c+a Problema 25. Considere un tri´angulo rect´angulo ABC y llame P , Q, R, a las reflexiones de A, B, C sobre BC, CA, AB respectivamente. Calcule la raz´on del ´area del tri´angulo ABC entre el ´area del tri´angulo P QR.

1.1 Problemas de Pr´ actica

5

Problema 26. Dentro de un hex´agono regular ABCDEF se coloca un punto G arbitrario y se trazan segmentos que lo unen con los v´ertices, formando 6 tri´angulos ABG, BCG, CDG, DEG, EF G, F AG los cuales se colorean en forma alternada de negro y blanco. Demuestre que el ´area de la regi´on negra es igual al ´area blanca. Problema 27. Este es un juego para dos jugadores que se juega en un tablero cuadrado cuadriculado que tiene un n´umero impar de filas y columnas. El juego empieza en la esquina inferior izquierda, donde el primer jugador pone su marca. Los jugadores alternan su jugada. En su turno cada jugador puede poner su marca en una de las casillas contiguas ya sea directamente encima, directamente a la derecha o diagonalmente encima y a la derecha de la ´ultima marca puesta por su oponente. El juego contin´ua de esta forma y gana el jugador que consiga poner su marca en la casilla ubicada en la esquina superior derecha del tablero. Encuentre una estrategia que permita al primer jugador ganar siempre el juego. Problema 28. ¿Por qu´e cada vez que tomamos diez n´umeros consecutivos existe uno que es primo relativo con cada uno de los otros nueve? Problema 29. En el tri´angulo equil´atero ABC, una recta paralela al lado BC intersecta a AC en M y a AB en P . Si D es el incentro del tri´angulo AP M y E es el punto medio de P C, determine los ´angulos del tri´angulo BDE. Problema 30. En cada subconjunto de 7 elementos del conjunto {1,2,...,10} se toma el mayor. ¿Cu´al es la suma de todos esos elementos mayores? Problema 31. Sea A un subconjunto de N = {1, 2, ..., n} y arreglemos sus elementos en orden decreciente de magnitud. Formemos las sumas S sumando y restando alternadamente elementos consecutivos del subconjunto A (el primer elemento de la suma siempre es positivo). ¿Cu´al es la suma de todas las sumas S generadas por los distintos subconjuntos de N? Problema 32. Si ABCDEF G es un hept´agono regular, muestre que 1 1 1 = + . AB AC AD Problema 33. Sea ABC un tri´angulo y sean E, F puntos sobre los lados CA y AB. Sea D la intersecci´on de BE con CF y suponga que las ´areas de los tri´angulos BDF , CDE y BCD son 3, 3 y 9 respectivamente. Encuentre el ´area del cuadril´atero AF DE.

6

Problemas de Pr´ actica

Problema 34. Una cuadr´ıcula de 3 × 3 est´a inicialmente llena con ceros y unos de la siguiente manera: 1 0 1

0 1 0

1 0 1

La cuadr´ıcula se ir´a modificando con la siguiente regla: Cada que se coloque una cuadr´ıcula de 2 × 2 sobre la cuadr´ıcula original, se aumentan en 1 los n´umeros de los cuadros comunes. Por ejemplo si la cuadr´ıcula de 2 × 2 se coloca arriba y a la izquierda del tablero de 3 × 3, ´este queda modificado as´ı: 2 1 1

1 2 0

1 0 1

¿Es posible, repitiendo la regla, llegar a un tablero de 3 × 3 donde todos los n´umeros sean m´ultiplos de 2? Si lo es, d´e una forma de hacerlo, en caso contrario argumente por qu´e no es posible. Problema 35. ¿Es posible que alguna potencia de 2 termine en 1982? Problema 36. Sea E un punto sobre una circunferencia de di´ametro AB. Por los puntos B y E se trazan las rectas tangentes t1 y t2 , respectivamente, que se cortan en el punto M. Sea C el punto de intersecci´on de las rectas AE y t1 . Demuestre que M es el punto medio de BC. Problema 37. En la siguiente cuadr´ıcula: 2 1 2 1 2

1 2 1 2 1

2 1 2 1 2

1 2 1 2 1

2 1 2 1 2

una “sustituci´on”consiste en tomar el n´umero en un cuadro y sumarle o restarle un m´ultiplo par de alguno de sus vecinos. ¿Es posible que despu´es de varias sustituciones sucesivas queden los n´umeros del 1 al 25 en la cuadr´ıcula?

1.1 Problemas de Pr´ actica

7

Problema 38. ¿Ser´a posible dividir la superficie de una esfera en un n´umero impar de regiones, todas triangulares? Nota: Cada dos regiones triangulares son ajenas, o comparten un v´ertice, o bien tienen un lado completo en com´un. Problema 39. Demuestre que no es posible construir un tri´angulo cuyos lados sean n´umeros primos y que su ´area sea entera. Problema 40. Se tienen los n´umeros del 1 al 100 ({1, 2, 3, 4, ..., 100}) y cada uno es pintado de rojo, azul, amarillo o verde. Demuestre que hay dos n´umeros del mismo color cuya diferencia tambi´en es del mismo color. Problema 41. Sea C1 una circunferencia fija y AB una cuerda en ella. Sean P un punto en uno de los arcos AB y C2 una circunferencia tangente a C1 en P y tangente a la cuerda AB. Si Q es el punto de tangencia de AB a C2 , demuestre que el ´angulo AP Q es constante, para todo punto P sobre el arco AB. Problema 42. Consideremos un c´ırculo con centro O, A y B puntos en la circunferencia tales que ∠AOB = 60◦ . Sean M un punto cualquiera en el arco AB y P , Q, R, S, los puntos medios de los segmentos AM, OB, OA, BM, respectivamente. Demuestre que P R es perpendicular a P S. Problema 43. Sean x, y, z n´umeros enteros positivos tales que 7 divide a x3 + y 3 − z 3 . Pruebe que alguno de ellos es m´ultiplo de 7. Problema 44. Pruebe que si un n´umero primo p puede escribirse de la forma p = p21 + p22 + p23 con p1 < p2 < p3 n´umeros primos entonces p1 = 3. Problema 45. Un rect´angulo grande est´a dividido en 9 rect´angulos m´as peque˜nos, como muestra la figura. En la parte interior de algunos rect´angulos peque˜nos est´a escrito su per´ımetro. ¿Cu´al es el per´ımetro del rect´angulo grande? 6 12

4 8

6

8

Problemas de Pr´ actica

Problema 46. Se quieren pintar las caras de un dodecaedro de tal forma que si dos caras tienen una arista en com´un entonces quedan pintadas con colores diferentes. (i) Muestre que no es posible hacer esto con s´olo tres colores, (ii) Muestre que es posible lograr esto con cuatro colores. Problema 47. Si x1 · x2 · x3 · x4 + x2 · x3 · x4 · x5 + ... + xn · x1 · x2 · x3 = 0 donde cada xi es 1 ´o −1, demuestre que n es m´ultiplo de 4. Problema 48. Encuentre el entero positivo m´as peque˜no que se puede expresar como suma de 9 enteros positivos consecutivos, como suma de 10 enteros positivos consecutivos y como suma de 11 enteros positivos consecutivos (las tres cosas a la vez). Problema 49. ¿Es posible agrupar los n´umeros 1, 2, ..., 100 en 3 grupos A1 , A2 y A3 tales que 102 divida a la suma de los elementos de A1 , 203 divida a la suma de los elementos de A2 y 304 divida a la suma de los elementos de A3 ? Problema 50. Paula numera un cuaderno con 96 hojas y coloca en las p´aginas la secuencia del 1 al 192. Juan quita 24 hojas al azar y suma los 48 n´umeros escritos en las p´aginas. ¿Puede ser esta suma igual a 1998? Problema 51. Dada una cuadr´ıcula de 5×5 se escribe en cada uno de los cuadros un 1 ´o un −1. El producto de los n´umeros en cada columna es escrito debajo de cada una de ellas. El producto de los n´umeros en cada rengl´on es escrito a la derecha de cada uno de estos. Pruebe que la suma de esos 10 productos no puede ser cero. Problema 52. Demuestre que si p y q son n´umeros primos tales que entero entonces p = q.

p2 +q 2 p+q

es

Problema 53. Pruebe que el n´umero de cuatro cifras N = abcd es divisible entre 3 si (a − 2b + c + 4d) es divisible entre 3. Problema 54. Pablo ha dibujado un cuadrado ABCD con tinta negra y debe colorear con rojo todos los puntos P del interior del cuadrado tales que el ´area del cuadril´atero BCP A es igual al triple del ´area del cuadril´atero AP CD. Describir cu´al es la parte roja del dibujo y justificar.

1.1 Problemas de Pr´ actica

9

Problema 55. Supongamos que queremos formar 5 pilas de cajas con las siguientes condiciones: cada pila debe tener entre una y cinco cajas, adem´as, cada pila no puede tener m´as cajas que la pila de su izquierda. ¿De cu´antas formas podemos hacer esto? Problema 56. Tenemos 16 focos acomodados en un tablero de 4 × 4, todos apagados. Cada vez que alguien toca un foco, este cambia de estado junto con todos los focos de su fila y su columna (Las filas son horizontales y las columnas verticales). (i) Demuestre que es posible, despu´es de tocar los focos adecuados, que todos los focos queden encendidos. (ii) Si el tama˜no del tablero fuera de 5 × 5. ¿Ser´ıa posible terminar con todos los focos prendidos? Problema 57. Los n´umeros del 1 al 12 se colocan (sin repetir) en los c´ırculos del siguiente arreglo triangular:

(i) Demuestre que no existe una forma de acomodarlos que cumpla que las sumas de los n´umeros que est´an en cada uno de los lados del tri´angulo sea 27. (ii) Muestre que si existe un acomodo en que la suma en cada uno de los lados del tri´angulo sea 28. Problema 58. Sea ABCDEF GH un oct´agono regular de radio 1. Demuestra que AB · AD = AC. Problema 59. Demuestre que para 4 puntos no alineados, los cuatro tri´angulos que estos puntos determinan no pueden ser todos acut´angulos.

10

Problemas Concursos Nacionales de la OMM

Problema 60. Sea ABCD un cuadril´atero convexo, donde M es el punto medio de BC, N es el punto medio de CD y O es la intersecci´on de las diagonales AC y BD. Demuestre que O es el gravicentro del tri´angulo AMN si y s´olo si ABCD es un paralelogramo. Problema 61. La colecci´on infinita de n´umeros 1, 2, 4, 5, 7, 9, 10, 12,14,16, ... se ha formado de la manera siguiente: se coloca el primer impar (1), luego se colocan los siguientes dos pares (2, 4), despu´es los tres impares siguientes al u ´ltimo par colocado (5, 7, 9), luego los cuatro pares siguientes al ´ultimo impar que se coloc´o y as´ı sucesivamente. ¿Cu´al es el n´umero par m´as cercano a 1998 que aparece en esta colecci´on? Problema 62. (i) Demuestre que el producto de tres enteros positivos consecutivos no puede ser un cubo. (ii) Demuestre que s´olo existe un primo p tal que el n´umero 2p + 1 es un cubo. Problema 63. Sean a, b y c tres enteros positivos tales que a > b > c. Pruebe que si a + b es m´ultiplo de c, b + c es m´ultiplo de a y a + c es m´ultiplo de b, abc entonces el cociente a+b+c es un cuadrado. Problema 64. ¿Cu´ales son las posibles ´areas de un hex´agono con todos los ´angulos iguales y cuyos lados miden 1, 2, 3, 4, 5, 6, en alg´un orden? Problema 65. En una cuadr´ıcula de 3 × 3 se acomodan los d´ıgitos del 1 al 9 sin repetir. Considere cada rengl´on como un n´umero de 3 cifras, y ll´amele A a la suma de estos tres n´umeros. Ahora, considere cada columna como un n´umero de tres cifras, y ll´amele B a la suma de estos tres n´umeros. ¿Puede encontrar alguna forma de acomodar los d´ıgitos del 1 al 9 de manera que A + B = 1997?

1.2.

Problemas de los u ´ltimos tres Concursos Nacionales de la OMM

Problema 1. (15a OMM) Encuentra todos los n´umeros de 7 d´ıgitos que son m´ultiplos de 3 y de 7, y cada uno de cuyos d´ıgitos es 3 o´ 7.

Enunciados de los Problemas

11

Problema 2. (15a OMM) Se tienen algunas pelotas de colores (son por lo menos tres colores), y por lo menos tres cajas. Las pelotas se ponen en las cajas de manera que no quede vac´ıa ninguna caja y que no haya tres pelotas de colores distintos que est´en en tres cajas distintas. Prueba que hay una caja tal que todas las pelotas que est´an fuera de ella son del mismo color. Problema 3. (15a OMM) En un cuadril´atero ABCD inscrito en una circunferencia llamemos P al punto de intersecci´on de las diagonales AC y BD, y sea M el punto medio de CD. La circunferencia que pasa por P y que es tangente a CD en M corta a BD y a AC en los puntos Q y R, respectivamente. Se toma un punto S sobre el segmento BD de tal manera que BS = DQ. Por S se traza una paralela a AB que corta a AC en un punto T . Prueba que AT = RC. Problema 4. (15a OMM) Dados dos enteros positivos n y a se forma una lista de 2001 n´umeros como sigue: El primer n´umero es a; a partir del segundo, cada n´umero es el residuo que se obtiene al dividir el cuadrado del anterior entre n. A los n´umeros de la lista se les ponen los signos + y − alternadamente empezando con +. Los n´umeros con signo as´ı obtenidos se suman y a esa suma se le llama suma final para n y a. ¿Para qu´e enteros n ≥ 5 existe alguna a tal que 2 ≤ a < n2 y la suma final para n y a es positiva? Problema 5. (15a OMM) Sea ABC un tri´angulo tal que AB < AC y el ´angulo BAC es el doble del ´angulo BCA. Sobre el lado AC se toma un punto D tal que CD = AB. Por el punto B se traza una recta l paralela a AC. La bisectriz exterior del ´angulo en A intersecta a l en el punto M, y la paralela a AB por el punto C intersecta a l en el punto N. Prueba que MD = ND. Problema 6. (15a OMM) Un coleccionista de monedas raras tiene monedas de denominaciones 1, 2, ..., n (tiene muchas monedas de cada denominaci´on). Desea poner algunas de sus monedas en 5 cajas de manera que se cumplan las siguientes condiciones: (a) En cada caja hay a lo m´as una moneda de cada denominaci´on. (b) Todas las cajas tienen el mismo n´umero de monedas y la misma cantidad de dinero. (c) Para cualesquiera dos cajas sucede que entre los dos tienen por lo menos una moneda de cada denominaci´on. (d) No existe una denominaci´on tal que todas las cajas tengan una moneda de esa denominaci´on. ¿Para qu´e valores de n puede el coleccionista hacer lo que se propone?

12

Problemas Concursos Nacionales de la OMM

Problema 7. (16a OMM) En una cuadr´ıcula de 32 × 32 se escriben los n´umeros del 1 al 1024 de izquierda a derecha, con los n´umeros del 1 al 32 en el primer rengl´on, los del 33 al 64 en el segundo, etc. La cuadr´ıcula se divide en cuatro cuadr´ıculas de 16 × 16 que se cambian de lugar entre ellas como sigue: Despu´es, cada cuadr´ıcula de 16 × 16 se divide en cuatro cuadr´ıculas de 8 × 8 que se cambian de lugar del mismo modo; a su vez cada una de esas se divide y as´ı sucesivamente hasta llegar a cuadr´ıculas de 2 × 2 que se dividen en cuadros de 1 × 1, los cuales se cambian de lugar del mismo modo. Al terminar estas operaciones, ¿qu´e n´umeros quedan en la diagonal que va de la esquina superior izquierda a la inferior derecha en la cuadr´ıcula de 32 × 32? Problema 8. (16a OMM) Sean ABCD un paralelogramo y K la circunferencia circunscrita al tri´angulo ABD. Sean E y F las intersecciones de K con los lados (o sus prolongaciones) BC y CD respectivamente (E distinto de B y F distinto de D). Demuestra que el circuncentro del tri´angulo CEF est´a sobre K. Problema 9. (16a OMM) Sean n un entero positivo. ¿Tiene n2 m´as divisores positivos de la forma 4k + 1 o de la forma 4k − 1? Problema 10. (16a OMM) Una ficha de domin´o tiene dos n´umeros (no necesariamente diferentes) entre 0 y 6. Las fichas se pueden voltear, es decir, 4 5 es la misma ficha que 5 4 . Se quiere formar una hilera de fichas de domin´o distintas de manera que en cada momento de la construcci´on de la hilera, la suma de todos los n´umeros de las fichas puestas hasta ese momento sea impar. Las fichas se pueden agregar de la manera usual a ambos extremos de la hilera, es decir, de manera que en cualesquiera dos fichas consecutivas aparezca el mismo n´umero en los extremos que se juntan. Por ejemplo, se podr´ıa hacer la hilera: 1 3 3 4 4 4 , en la que se coloc´o primero la ficha del centro y luego la de la izquierda. Despu´es de poner la primera ficha, la suma de todos los n´umeros es 7; despu´es de poner la segunda, 11; despu´es de la tercera, 19. ¿Cu´al es la mayor cantidad de fichas que se pueden colocar en una hilera? ¿Cu´antas hileras de esa longitud m´axima se pueden construir? Problema 11. (16a OMM) Tres enteros distintos forman una terna compatible si alguno de ellos, digamos n, cumple que cada uno de los otros dos es, o bien divisor, o bien m´ultiplo de n. Para cada terna compatible de n´umeros entre 1 y 2002 se calcula la suma de los tres n´umeros de la terna. ¿Cu´al es la mayor suma obtenida? ¿Cu´ales son las ternas en las que se obtiene la suma m´axima?

Enunciados de los Problemas

13

Problema 12. (16a OMM) Sea ABCD un cuadril´atero con AD paralelo a BC, los ´angulos en A y B rectos y tal que el ´angulo CMD es recto, donde M es el punto medio de AB. Sean K el pie de la perpendicular a CD que pasa por M, P el punto de intersecci´on de AK con BD y Q el punto de intersecci´on de BK con AC. Demuestra que el ´angulo AKB es recto y que KQ KP + = 1. PA QB Problema 13. (17a OMM) Dado un n´umero k de dos o m´as cifras, se forma otro entero m insertando un cero entre la cifra de las unidades y la de las decenas de k. Encuentra todos los n´umeros k para los cuales m resulta ser un m´ultiplo de k. Problema 14. (17a OMM) Sean A, B y C tres puntos colineales con B entre A y C. Sea Y una circunferencia tangente a AC en B, y sean X y Z las circunferencias de di´ametros AB y BC, respectivamente. Sea P el otro punto (adem´as de B) en el que se cortan las circunferencias X y Y; sea Q el otro punto (adem´as de B) en el que se cortan las circunferencias Y y Z. Sup´on que la recta P Q corta a X en un punto R distinto de P , y que esa misma recta P Q corta a Z en un punto S ditinto de Q. Demuestra que concurren AR, CS y la tangente com´un a X y Z por B. Problema 15. (17a OMM) En una fiesta hay el mismo n´umero n de muchachos que de muchachas. Sup´on que a cada muchacha le gustan a muchachos y que a cada muchacho le gustan b muchachas. ¿Para qu´e valores de a y b es correcto afirmar que forzosamente hay un muchacho y una muchacha que se gustan mutuamente? Problema 16. (17a OMM) Sea ABCD un trapecio con AB paralelo a DC. Se toman puntos P y Q sobre AB y CD respectivamente, tales que PAPB = DQ . QC Sea M la intersecci´on de AQ con DP y sea N la intersecci´on de P C con QB. Pruebe que la longitud de MN depende s´olo de las longitudes de AB y DC, y calcula su valor.

14

Problemas Concursos Nacionales de la OMM

Problema 17. (17a OMM) Se escriben en tarjetas todas las parejas de enteros (a, b) con 1 ≤ a < b ≤ 2003. Dos personas juegan con las tarjetas como sigue: cada jugador en su turno elige (a, b) (que se retira del juego) y escribe el producto a · b en un pizarr´on (ambos jugadores usan el mismo pizarr´on). Pierde el jugador que ocasione que el m´aximo com´un divisor de los n´umeros escritos hasta ese momento sea 1. ¿Qui´en tiene estrategia ganadora? (Es decir, ¿cu´al de los dos jugadores puede inventar un m´etodo con el cual asegura su triunfo?) Problema 18. (17a OMM) Dado un entero n un cambio sensato consiste en sustituir n por 2n + 1 ´o 3n + 2. Dos enteros positivos a y b se llaman compatible si existe un entero que se puede obtener haciendo uno o m´as cambios sensatos, tanto a partir de a, como a partir de b. Encuentra todos los enteros positivos compatibles con 2003 menores que 2003.

Cap´ıtulo 2 Olimpiadas Internacionales en las que participa M´ exico

2.1.

V Olimpiada Matem´ atica de Centroam´ erica y el Caribe

Problema 1. Dos jugadores A y B, juegan por turnos el siguiente juego: Se tiene un mont´on de 2003 piedras. En su primer turno, A escoge un divisor de 2003, y retira ese n´umero de piedras del mont´on inicial. Posteriormente, B escoge un divisor del n´umero de piedras restantes, y retira ese n´umero de piedras del nuevo mont´on, y siguen as´ı sucesivamente. Pierde el jugador que retira la u ´ltima piedra. Demostrar que uno de los dos jugadores tiene una estrategia ganadora y describir dicha estrategia. Nota: Se entiende por estrategia ganadora un m´etodo de juego que le garantiza la victoria al que lo aplica sin importar lo que haga su oponente. Problema 2. Sea S una circunferencia y AB un di´ametro de ella. Sea t la recta tangente a S en B y considere dos puntos C, D en t tales que B est´e entre C y D. Sean E y F las intersecciones de S en AC y AD, y sean G y H las intersecciones de S con CF y DE. Demostrar que AH = AG.

16

Problemas de las Olimpiadas Internacionales

Problema 3. Sean a, b enteros positivos, con a > 1 y b > 2. Demostrar que ab + 1 ≥ b(a + 1) y determinar cu´ando se tiene la igualdad. Problema 4. Sean S1 y S2 dos circunferencias que se intersectan en dos puntos distintos P y Q. Sean l1 y l2 dos rectas paralelas, tales que: (i) l1 pasa por el punto P e intersecta a S1 en un punto A1 distinto de P y a S1 en un punto A2 distinto de P . (ii) l2 pasa por el punto Q e intersecta a S1 en un punto B1 distinto de Q y a S2 en un punto B2 distinto de Q. Demostrar que los tri´angulos A1 QA2 y B1 QB2 tienen igual per´ımetro. Problema 5. Un tablero cuadrado de 8 cm de lado se divide en 64 casillas cuadradas de 1 cm de lado cada una. Cada casilla se puede pintar de blanco o de negro. Encontrar el n´umero total de maneras de colorear el tablero de modo tal que cada cuadrado de 2 cm de lado formado por cuatro casillas con un v´ertice com´un, contenga dos casillas blancas y dos negras. Problema 6. Digamos que un entero positivo es tico si la suma de sus d´ıgitos (en base 10) es m´ultiplo de 2003. paralelas, tales que: (i) Demostrar que existe un entero positivo N tal que sus primeros 2003 m´ultiplos N, 2N, 3N, . . . , 2003N, son todos ticos. (ii) ¿Existe alg´un entero positivo N tal que todos sus m´ultiplos sean ticos?

2.2.

XVIII Olimpiada Iberoamericana de Matem´ aticas

Problema 1. (a) Se tienen dos sucesiones, cada una de 2003 enteros consecutivos, y un tablero de 2 filas y 2003 columnas ... ...

... ...

Decida si siempre es posible distribuir los n´umeros de la primera sucesi´on en la primera fila y los de la segunda sucesi´on en la segunda fila, de tal manera que los resultados obtenidos al sumar los dos n´umeros de cada columna formen una nueva sucesi´on de 2003 n´umeros consecutivos.

Enunciados de los Problemas

17

(b) ¿Y si se reemplaza 2003 por 2004? Tanto en a) como en b), si la respuesta es afirmativa, explique c´omo ditribuir´ıa los n´umeros, y si es negativa, justifique el porqu´e. Problema 2. Sean C y D dos puntos de la semicircunferencia de di´ametro AB tales que B y C est´an en semiplanos distintos respecto de la recta AD. Denotemos M, N y P los puntos medios de AC, DB y CD, respectivamente. Sean OA y OB los circuncentros de los tri´angulos ACP y BDP . Demuestre que las rectas OA OB y MN son papalelas. Problema 3. Pablo estaba copiando el siguiente problema: Considere todas las sucesiones de 2004 n´umeros reales (x0 , x1 , x2 , . . . , x2003 ), tales que x0 0≤ 0≤

= x1 x2 .. .

1, ≤ 2x0 , ≤ 2x1 ,

0 ≤ x2003 ≤ 2x2002 . Entre todas estas sucesiones, determine aquella para la cual la siguiente expresi´on toma su mayor valor: S = . . . . Cuando Pablo iba a copiar la expresi´on de S le borraron la pizarra. Lo u ´nico que pudo recordar es que S era de la forma S = ±x1 ± x2 ± . . . ± x2002 + x2003 , donde el ´ultimo t´ermino, x2003 , ten´ıa coeficiente +1, y los anteriores ten´ıan coeficientes +1 ´o −1. Demuestre que Pablo, a pesar de no tener el enunciado completo, puede determinar con certeza la soluci´on del problema. Problema 4. Sea M = {1, 2, . . . , 49} el conjunto de los primeros 49 enteros positivos. Determine el m´aximo entero k tal que el conjunto M tiene un subconjunto de k elementos en el que no hay 6 n´umeros consecutivos. Para ese valor m´aximo de k, halle la cantidad de subconjuntos de M, de k elementos, que tienen la propiedad mencionada.

18

Problemas de las Olimpiadas Internacionales

Problema 5. En el cuadrado ABCD, sean P y Q puntos pertenecientes a los lados BC y CD respectivamente, distintos de los extremos, tales que BP = CQ. Se consideran puntos X e Y , X 6= Y , pertenecientes a los segmentos AP y AQ, respectivamente. Demuestre que, cualesquiera sean X e Y , existe un tri´angulo cuyos lados tienen las longitudes de los segmentos BX, XY y DY . Problema 6. Se definen las suceisones (an )n≥0 , (bn )n≥0 por: an+1

a0 = 1 = a2001 + bn , n

b0 = 4 y bn+1 = b2001 + an , n

para n ≥ 0.

Demuestre que 2003 no divide a ninguno de los t´erminos de estas sucesiones.

2.3.

XLIV Olimpiada Internacional de Matem´ aticas

Problema 1. Sea A un subconjunto del conjunto S = {1, 2, . . . , 1000000} que contiene exactamente 101 elementos. Demuestra que existen n´umeros t1 , t2 , . . . , t100 en S tal que los conjuntos Ai = {x + tj / x ∈ A} para j = 1, 2, . . . , 100

son ajenos dos a dos.

Problema 2. Encuentra todas las parejas de enteros positivos (a, b) tales que

sea un entero positivo.

a2 2ab2 − b2 + 1

Problema 3. Considera un hex´agono convexo en el cual cualesquiera dos lados √ opuestos tienen la siguiente propiedad: la distancia entre sus puntos medios es 23 veces la suma de sus longitudes. Demuestra que todos los ´angulos del hex´agono son iguales. Un hex´agono convexo ABCDEF tiene tres pares de lados opuestos: AB y DE, BC y EF , CD y F A. Problema 4. Sea ABCD un cuadril´atero c´ıclico. Sean P , Q y R los pies de las perpendiculares desde D a las rectas BC, CA y AB, respectivamente. Demuestra que P Q = QR si y s´olo si la bisectriz del ∠ABC y ∠ADC se intersectan en AC.

Enunciados de los Problemas

19

Problema 5. Sea n un entero positivo y x1 , x2 , . . . , xn n´umeros reales tales que x1 ≤ x2 ≤ · · · ≤ xn . (a) Demuestra que n X n X i=1 j=1

|xi − xj |

!2

n



n

2(n2 − 1) X X (xi − xj )2 . 3 i=1 j=1

(b) Demuestra que la igualdad es v´alida si y s´olo si x1 , x2 , . . . , xn es una progresi´on aritm´etica. Problema 6. Sea p un n´umero primo. Demuestra que existe un n´umero primo q tal que para todo entero n, el n´umero np − p no es divisible entre q.

20

Problemas de las Olimpiadas Internacionales

Cap´ıtulo 3 Soluciones de los Problemas

3.1.

Soluciones de los Problemas de Pr´ actica

Soluci´ on del problema 1. (i) Sean di = ai+1 −ai las diferencias, i = 1, ..., 108. Por un lado tenemos que: d1 + d2 + · · · + d108 = a109 − a1 ≤ 1998 − 1 = 1997. Ahora, si d′1 ≤ d′2 ≤ · · · ≤ d′108 son las diferencias ordenadas, como d′1 , d′2 , d′3 son al menos 1; d′4 , d′5 , d′6 son al menos 2;...; d′106 , d′107 , d′108 son al menos 36, tenemos que: d1 + d2 + · · · + d108 = d′1 + d′2 + · · · + d′108 ≥ 3 (1 + 2 + · · · + 36) = 1998, lo que da una contradicci´on. (ii) Considere: a1 = 1 y ai = a1 + d1 + · · · + di−1 , donde d1 = d2 = d3 = 1, d4 = d5 = d6 = 2, ..., d106 = d107 = d108 = 36. Soluci´ on del problema 2. Primero veamos que n es par. Si n fuera impar entonces cada ai deber´a ser impar, pero la suma de un n´umero impar de impares no puede ser igual a cero. Luego n es par. Ahora veamos que n es m´ultiplo de 4. Si n fuera de la forma n = 4m + 2 = 2(2m + 1), entonces alguna ai es 2 y las restantes n − 1 son impares, luego, la suma de las ai impares, es impar (pues n − 1 es impar) y si agregamos el 2 tenemos que a1 + a2 + · · · + an es impar, luego, no podr´a ser 0. Por tanto n debe ser de la forma 4m.

22

Soluciones Problemas de Pr´ actica

Soluci´ on del problema 3. Hay 105. Primero observemos que: 111, 222, 333, ..., 999 son equilibrados. Notemos tambi´en que si abc es un n´umero equilibrado entonces tambi´en lo son: acb, bac, bca, cab y cba, adem´as si dos cifras de un n´umero equilibrado son iguales la tercera cifra es igual a estas, por lo que los n´umeros equilibrados de cifras diferentes aparecen de 6 en 6. Finalmente, hay que tener en cuenta que si abc es equilibrado con c = a+b entonces a y b deben 2 tener la misma paridad. Construimos ahora los n´umeros equilibrados siguiendo un orden, primero los que tienen la primera cifra igual a 1, despu´es cuando es 2, etc. 132, 153, 174, 195 243, 264, 285, 354, 375, 396, 465, 486, 576, 597, 687, 798. Por la observaci´on anterior estos 16 n´umeros equilibrados, generan a 16 · 6 = 96 n´umeros y con los 9 primeros, dan los 105 n´umeros equilibrados.

Soluci´ on del problema 4. Coloreamos el tablero con dos colores: blanco y negro en forma alternada, de manera que queden 13 negros y 12 blancos. Como una pieza de 2 × 1 cubre un cuadro blanco y uno negro, al cubrir el tablero con 12 de estas piezas nos quedar´a sin cubrir un cuadro negro. Veamos que cualquiera de ellos puede quedar sin cubrir. Dos renglones contiguos siempre se pueden cubrir, as´ı el cuadrado de 4 renglones y 5 columnas que se obtiene al quitar el primer rengl´on se puede cubrir y en el primer rengl´on se pueden dejar de cubrir cualquiera de los 3 cuadros negros. Este m´etodo funciona tambi´en para el tercer y quinto rengl´on.

Fij´emonos en el cuadro de 3 × 3 que est´a en la esquina superior izquierda, ´este se puede cubrir dejando el centro descubierto de la siguiente manera:

3.1 Soluciones de los Problemas de Pr´ actica

23

Como el resto del cuadrado de 5×5 se cubre, entonces tenemos manera de cubrir el de 5 × 5 dejando el cuadrado negro central del subcuadrado de 3 × 3 que seleccionamos. En forma an´aloga, se procede para los cuadros negros restantes. Soluci´ on del problema 5. Los trapecios de lados 1, 1, 1 y 2 son el resultado de “pegar”3 tri´angulos equil´ateros de lado 1 formando la mitad de un hex´agono regular. Un tri´angulo equil´atero de lado n se divide en n2 triangulitos de lado 1. Por lo tanto, para que un tri´angulo de lado n se pueda dividir en trapecios como los mencionados, necesariamente n2 deber´a ser m´ultiplo de 3, es decir, n debe ser m´ultiplo de 3. Esto tambi´en es suficiente: si n es m´ultiplo de 3, el tri´angulo equil´atero de lado n puede dividirse en tri´angulos equil´ateros de lado 3 y basta probar que uno de ´estos puede dividirse en trapecios de lados 1, 1, 1 y 2. La divisi´on necesaria se muestra en la figura:

Soluci´  on del problema 6. Los icosaedros tienen 12 v´ertices, por lo que hay 12 = 66 segmentos que unen pares de v´ertices. Si a esos 66 segmentos les 2 quitamos las aristas, tendremos el n´umero de diagonales. ¿Cu´antas aristas tiene un icosaedro regular? Veamos: tiene 20 caras de 3 aristas cada una y cada arista est´a en dos caras. Por lo tanto, hay 20·3 = 30 aristas. Entonces, hay 66−30 = 36 2 diagonales. Segunda Soluci´ on. Usaremos la f´ormula de Euler que relaciona el n´umero de aristas A, el n´umero de v´ertices V y el n´umero de caras C de un poliedro: V − A + C = 2.

24

Soluciones Problemas de Pr´ actica

Como en el icosaedro las caras son tri´angulos tenemos que 2A = 3C ya que cada cara tiene tres aristas y una arista es compartida por dos caras, luego hay 30 aristas. Por la f´ormula de Euler el n´umero de v´ertices es V = A − C + 2 = 30 − 20 + 2 = 12 y entonces se tiene que 5V = 2A, es decir, cada v´ertice tiene 5 v´ertices adyacentes. Al unir cada v´ertice con los 11 v´ertices restantes y al restar sus 5 v´ertices adyacentes, tenemos que de cada v´ertice salen 6 diagonales luego el n´umero de diagonales es 12·6 = 36. 2 Soluci´ on del problema 7. No es dif´ıcil ver que si el n´umero tiene 90 cifras entonces tiene 12 cifras iguales a a y 78 cifras 2. Como un n´umero es divisible entre 9 si y s´olo si la suma de sus cifras es divisible entre 9 y como la suma de las cifras 2 es 2 · 78 que es divisible entre 9, bastar´a ver cuando 12a es divisible entre 9. Como 12a tiene un factor 3, ser´a suficiente que a tenga un factor 3, luego los valores de a son 0, 3, 6 y 9. Soluci´ on del problema 8. Como: a + 1b a + b−1 = = 1 a−1 + b +b a

ab+1 b  ab+1 a



a = , b

debemos encontrar los a y b tales que ab = 13 y a + b ≤ 80. Como ab = 13, a = 13b, entonces, a + b = 14b ≤ 80. Por lo tanto, b es a lo m´as 5. Las parejas (a, b) que cumplen son (13, 1), (26, 2), (39, 3), (52, 4), (65, 5). Soluci´ on del problema 9. Por el Teorema de la bisectriz: BN MN = AM AB

BC PC = QB QP

y

Los tri´angulos ABN y QBC, son semejantes ya que ∠BAN = ∠BQC y N ∠ABN = ∠QBC = 2θ, por lo tanto, BN = BC . Luego M = PQPC . AB QB AM A

Q M P

θ θ B

θ

N C

3.1 Soluciones de los Problemas de Pr´ actica

25

Al sumar 1 de ambos lados de esta ´ultima igualdad obtenemos: QC AN = AM QP Por tanto, AM CP QP CP QC + = + = = 1. AN CQ QC CQ QC Soluci´ on del problema 10. Sea Q la intersecci´on de OP con CD. Probar que O, P y H son colineales es equivalente a probar que P Q⊥CD. Sea α = ∠BAC = ∠BDC. Como O es el circuncentro de ABP , ∠BOP = 2α. A D α P α H O Q 2α

B

C

Como el tri´angulo BOP es is´osceles, ∠OP B = 90o − 21 ∠BOP = 90o − α. Entonces ∠DP Q + ∠P DQ = ∠OP B + α = (90o − α) + α = 90o . Por lo tanto, ∠P QD = 90o , es decir, P Q⊥CD. Soluci´ on del problema 11. Fij´emonos en la forma de llenar el cuadrito central de 2 × 2, cada una de las formas de llenarlo dar´a una forma de llenar toda la cuadr´ıcula. El cuadrado central se puede llenar con dos fichas de dos maneras:

y

Para cada una de estas formas hay 2 maneras de terminar el llenado de la cuadr´ıcula:

26

Soluciones Problemas de Pr´ actica

As´ı hay 4 maneras de llenar la cuadr´ıcula, si iniciamos llenando primero el cuadrado central de 2 × 2. Si en el cuadrado central de 2 × 2 hay solo una ficha completa dentro de ´el, por ejemplo as´ı:

Tenemos que los dos cuadrados de 2 × 2 de la derecha tanto el inferior como el superior se pueden llenar cada uno de 2 maneras y el lado izquierdo de la cuadr´ıcula que falta llenar, se llena de manera forzada. Luego, hay 4 maneras de llenar la cuadr´ıcula en este caso y si giramos la cuadr´ıcula para considerar los 4 posibles casos de la ficha de 2×1 que se coloca en el cuadro central, tenemos en total en este caso 16 maneras de llenarla. Finalmente veamos el caso donde no hay una ficha de 2 × 1, dentro del cuadrado central. Dividimos la cuadr´ıcula en 4 cuadros de 2 × 2.

Cada uno de estos cuadrados de 2 × 2 se puede llenar de 2 maneras y entonces en este caso hay 16 maneras de llenar la cuadr´ıcula.

3.1 Soluciones de los Problemas de Pr´ actica

27

Resumiendo tenemos que hay en total 36 formas de llenar la cuadr´ıcula. Soluci´ on del problema 12. El reo n´umero 10, que es el primero en hablar, ve 9 sombreros de dos colores posibles. De un color debe haber un n´umero par y del otro uno impar. Los reos se pueden poner de acuerdo para que el reo n´umero 10 diga el color del que ve un n´umero par, de esta forma el reo n´umero 9 sabr´a de que color es su sombrero, ya que cuenta el n´umero de sombreros que ve del color que dijo su compa˜nero: si sigue siendo par, su sombrero es del otro color, y si es impar, su sombrero es de ese color. As´ı cada uno de los siguientes reos cuenta el n´umero de sombreros que ve del color que dijo el reo n´umero 10 y a ese n´umero le suma la cantidad de veces que sus otros compa˜neros han dicho el color, si el n´umero que le da es par, entonces su sombrero es del otro color y si es impar, su sombrero es de ese color. De esta forma el ´unico que quiz´a no se salva es el reo n´umero 10. Soluci´ on del problema 13. Supongamos que 1 es rojo y que n es negro. Para n = 2 es claro el resultado, supongamos el resultado cierto para k < n. Sean a el menor entero coloreado de negro y b el mayor entero coloreado de rojo de entre {1, 2, ..., n}, as´ı 1, 2, ..., a − 1 son rojos y b + 1, b + 2, ..., n son negros. Ahora, fij´emonos en el conjunto {a, a + 1, ..., b} tenemos que inicia con a coloreado de negro y termina en b que es de color rojo, luego, por la hip´otesis de inducci´on el n´umero de parejas consecutivas de diferente color en este conjunto es impar. Sumando los dos cambios de color que hay entre a − 1 y a y entre b y b + 1, el total tambi´en es impar. Soluci´ on del problema 14. Supongamos que el cuadro 1 se llena con una ficha de domin´o vertical (ver figura). 1 2 3 4 5 6 7 8 9 10 11 12 13 14

Si 2 tambi´en se llena con una ficha vertical, terminamos. Supongamos entonces que 2 se cubre con una ficha horizontal. Ahora, si 3 se cubre con una horizontal

28

Soluciones Problemas de Pr´ actica

acabamos; supongamos que se cubre con una vertical. Del mismo modo vemos que 1, 3, 5, 7, 9, 11 y 13 se deben llenar con fichas verticales y 2, 4, 6, 8, 10, 12 con fichas horizontales. Esto obliga a cubrir 14 con una ficha vertical formando un cuadrado de 2 × 2 con la ficha que cubre 13. Por lo tanto, siempre hay dos fichas que forman un cuadro de 2 × 2. Segunda Soluci´ on. Supongamos que no hay 2 fichas que formen un cuadrado de 2 × 2. Contaremos el n´umero de parejas (C, D) en el tablero donde C es un cuadro de 2 × 2 y D es una ficha de domin´o contenida completamente en C. Como cada cuadro de 2 × 2 contiene a lo m´as una ficha, el n´umero de parejas es a lo m´as 49, el n´umero de cuadros de 2 × 2 en un tablero de 8 × 8. Por otra parte, cada ficha que no est´e contenida totalmente en el marco de ancho 1, est´a contenida en dos cuadrados de 2 × 2. Por lo tanto, si hay m fichas en el marco, hay m + 2(32 − m) = 64 − m parejas de las que contamos. Entonces, 64 − m es menor o igual que 49 y por lo tanto m ≥ 15. Pero no puede haber 15 fichas en el marco, pues el marco tiene s´olo 28 cuadritos. Esta contradicci´on prueba que debe haber 2 fichas en un cuadro de 2 × 2. Soluci´ on del problema 15. Hay tres tipos de paralelep´ıpedos: (i) paralelep´ıpedos con todas las aristas iguales (cubos), de ´estos hay 10, que correspondende a la manera de elegir la longitud de la arista entre los 10 n´umeros. (ii) paralelep´ıpedos con dos longitudes de aristas iguales y una distinta, hay 10 formas de elegir la longitud de las aristas iguales y 9 formas de elegir la otra longitud, por lo que hay en ´este caso 10 · 9 = 90 paralelep´ıpedos distintos. (iii) paralelep´ıpedos con tres longitudes de aristas distintas, aqu´ı se deben  escoger las tres longitudes distintas 10 entre las 10 posibles estos se hace de 3 = 120 maneras. Por lo que en total hay 10 + 90 + 120 = 220 paralelep´ıpedos distintos. Segunda Soluci´ on. Ordenando los lados del paralelep´ıpedo de menor a mayor evitamos repeticiones. Entonces, el n´umero de paralelep´ıpedos rectangulares distintos con lados enteros entre 1 y 10 es igual al n´umero de tercias de enteros (a, b, c)  que satisfacen 1 ≤ a ≤ b ≤ c ≤ 10. Afirmamos que el n´umero de tercias es 12 , el n´umero de subconjuntos de 3 elementos del conjunto 1, 2, 3, . . . , 12. De 3 la condici´on que deben cumplir las tercias se obtiene 1 ≤ a < b+ 1 < c + 2 ≤ 12, por lo que a, b + 1 y c + 2 son n´umeros del 1 al 12 diferentes entre s´ı. Entonces, a cada tercia (a, b, c) le podemos hacer corresponder el conjunto {a, b + 1, c + 2}. Y viceversa, para obtener una de las tercias a partir de cualquier subconjunto de 3 elementos de {1, 2, 3, . . . , 12}, basta con ordenar los elementos del conjunto, digamos x < y < z y formar la tercia (x, y − 1, z − 2). Como x, y y z est´an entre

3.1 Soluciones de los Problemas de Pr´ actica

29

1 y 12 y se cumple que x < y < z, tenemos que 1 ≤ x ≤ y − 1 ≤ z − 2 ≤ 10, de modo que la tercia propuesta efectivamente cumple la condici´on requerida. Esta correspondencia muestra que el n´umero de tercias que cumplen la condici´on (que es igual al de paralelep´ıpedos buscados) es igual al n´umero de subconjuntos  12 de {1, 2, 3, . . . , 12} de 3 elementos, es decir, 3 . P P P Soluci´ on del problema 16. S´ı es posible. Para ver esto, sean c, v y a las sumas de los n´umeros asignados a las caras, v´ertices y aristas, respectivamente. Como cada cara est´a formada por tres aristas, tenemos que: 2

X

a=3

X

a=3

X

c

X

v

(3.1)

y como cada v´ertice es com´un a tres aristas, tenemos que: 2 Adem´as, X

c+

X

v+

P

c=

X

a = 1 + 2 + · · · + 14 = 105.

(3.2) (3.3)

Resolviendo el sistema formado por (3.1), (3.2) y (3.3) tenemos que: P

v = 30 y

P

a = 45.

Para que el n´umero asignado a una arista sea entero, debemos tener que los n´umeros asignados a las caras adyacentes a la arista deben ser de la misma paridad y los n´umeros asignados a los v´ertices que determina la arista tambi´en deben ser de la misma paridad. Pero si un v´ertice es par (impar) lo ser´an tambi´en los dem´as v´ertices, esto mismo sucede con las caras. Los conjuntos de cuatro n´umeros con la misma paridad, cuya suma sea 30 y sean elementos de {1, 2, 3, ..., 14} , son los siguientes ocho conjuntos: {1, 5,11,13}, {1, 7, 9, 13}, {3, 5, 9, 13}, {3, 7, 9, 11}, {2, 4, 10, 14}, {2, 6, 8, 14}, {2, 6, 10, 12}, {4, 6, 8, 12}. Uno de estos conjuntos debe ser usado para numerar las caras y otro para numerar los v´ertices. Puesto que ning´un promedio debe ser elemento del conjunto, los u ´nicos conjuntos que satisfacen (despu´es de promediar por parejas los n´umeros de cada conjunto de todas las maneras posibles) son: {1, 5, 11, 13} y {2, 4, 10, 14}. Ambos dan el mismo conjunto para numerar las aristas. A saber: {3, 6, 7, 8, 9, 12} .

30

Soluciones Problemas de Pr´ actica

As´ı, tenemos dos soluciones dependiendo de qu´e conjunto se utilice para numerar las caras y cual para numerar los v´ertices. Notemos que cada soluci´on es el dual de la otra, ya que en la primera los n´umeros asignados a las caras son los n´umeros asignados a los v´ertices de la segunda y viceversa. Soluci´ on del problema 17. Supongamos que los n´umeros primos son: p, p + 6, p + 12, p + 18, p + 24 al tomar congruencias m´odulo 5 tenemos que estos n´umeros son congruentes a p, p + 1, p + 2, p + 3, p + 4 respectivamente, pero entre cinco n´umeros consecutivos siempre hay uno que es divisible entre 5 y como p debe ser primo no hay otra opci´on m´as que p = 5, y los n´umeros primos son 5, 11, 17, 23 y 29. Soluci´ on del problema 18. El tri´angulo A1 B1 C1 se divide en los tri´angulos AA1 B1 , AA1 C1 y AB1 C1 . Como BB1 es paralela a AA1 , (AA1 B1 ) = (AA1 B) (si pensamos que la base de ambos tri´angulos es AA1 , tiene tambi´en la misma altura: la distancia de AA1 a BB1 ). B1 C1

A

B

A1

C

An´alogamente, (AA1 C1 ) = (AA1 C). Por lo tanto, (AA1 B1 ) + (AA1 C1 ) = (AA1 B) + (AA1 C) = (ABC). Veamos ahora el tri´angulo AB1 C1 : como BB1 es paralela a CC1 , (BB1 C1 ) = (BB1 C). Restando (BB1 A) a cada lado de esa igualdad se obtiene que (AB1 C1 ) = (ABC). Por lo tanto, (A1 B1 C1 ) = (AB1 C1 ) + ((AA1 B1 ) + (AA1 C1 )) = 2(ABC) = 3998. y Soluci´ on del problema 19. (i) Si n es impar, tomamos a = 0, b = n+1 2 n−1 n−4 c = 2 . Si n es m´ultiplo de cuatro, tomamos a = 0, b = n+4 y c = . 4 4 n−2 n Finalmente, si n es de la forma 4k + 2, tomamos a = 1, b = 2 y c = 2 .

3.1 Soluciones de los Problemas de Pr´ actica

31

(ii) Probaremos primero que n = a2 + b2 + c2 no tiene soluciones enteras si n es de la forma 4t (8k + 7). Un cuadrado perfecto es congruente con 0, 1 o´ 4 m´odulo 8. Haciendo todas las posible sumas m´odulo 8 de tres 0′ s, 1′ s y 4′ s, vemos que a2 + b2 + c2 nunca es congruente con 7 m´odulo 8. Tambi´en podemos comprobar que no es m´ultiplo de 4 (o sea, congruente con 0 ´o 4 m´odulo 8) a menos que a, b y c sean los tres pares. Por lo tanto, si a2 + b2 + c2 = 4t (8k + 7), no podemos tener t = 0. Pero si t ≥ 1, a, b y c deben ser pares, digamos 2a1 , 2b1 y 2c1 . Sustituyendo hallamos que a21 + b21 + c21 = 4t−1 (8k + 7), que es la misma ecuaci´on con t reducida en 1. Repetiendo el argumento suficientes veces llegamos a t = 0, lo cual es absurdo. Por lo tanto, n = a2 + b2 + c2 no tiene soluciones si n es de la forma 4t (8k + 7). Contemos cuantas n < 1999 son de esa forma: Con t = 0 hay 249, k = 0, 1, . . . , 248. Con t = 1 hay 62, k = 0, 1, . . . , 61. Con t = 2 hay 15, k = 0, 1, . . . , 14. Con t = 3 hay 4, k = 0, 1, 2, 3. Con t = 4 hay 1, k = 0. En total son 331 n´umeros de esa forma, por lo que hay a lo m´as 1998−331 = 1667 enteros menores que 1999 que son suma de tres cuadrados.

√ Soluci´ on del problema 20. Ponemos en cada v´ertice indicado con A, 2 y en cada v´ertice indicado con B, √12 . Ahora, sobre cada lado, el n´umero escrito es el producto de los n´umeros puestos en los v´ertices. El producto de los n´umeros escritos sobre los lados es el cuadrado del producto de los n´umeros asignados a los v´ertices,pues cada v´ertice  pertenece a dos lados. Por lo tanto, el producto √ a  1  b 2 √ = 2a−b . buscado es 2 2 Soluci´ on del problema 21. Como MN y AC son perpendiculares a MB, se tiene que MN k AC. Sea P en MN tal que CP ⊥MN, entonces P MEC es un rect´angulo, P C = ME = AD y ∠MP C = π/2 = ∠ECP.

32

Soluciones Problemas de Pr´ actica M E

A

B

P

D

N

C

Como P NkCE se tiene que ∠DCE = ∠CNP , y entonces los tri´angulos P NC y DCA tienen todos sus ´angulos iguales y un lado igual, por tanto son congruentes y entonces NC = CA. Luego el tri´angulo NCA es is´osceles. Soluci´ on del problema 22. Los tri´angulos ALN y ACL son semejantes, por lo que: AL2 = AN · AC. A

D

N

M B

L

C

Tambi´en ALM y ABL son semejantes por lo que: AL2 = AM · AB. Luego AL4 = AB · AC · AN · AM.

(3.4)

Por otro lado el cuadril´atero AMLN es c´ıclico ya que ∠MAN +∠MLN = 180◦, luego los ´angulos ∠AMN y ∠ALN son iguales y entonces son semejantes los tri´angulos ADM y ANL, por lo que: AD · AL = AN · AM.

(3.5)

De las ecuaciones (3.4) y (3.5) tenemos que AL3 = AB · AC · AD. Soluci´ on del problema 23. Sea G el centroide, es decir, el punto de intersecci´on de las medianas AL, BM y CN.

3.1 Soluciones de los Problemas de Pr´ actica

33

A N

M

G

L

B

C

Como LN es paralela a CA, tenemos que ∠LAC = ∠ALN. Luego, ∠MBA = ∠LAC = ∠ALN ⇔ BLGN es c´ıclico ⇔ ∠CNA = ∠ALB. Soluci´ on del problema 24. Observemos primero que a(a + b) − ab ab a2 = =a− . a+b a+b a+b  a+b 2 , 2

ab ab tenemos que a+b ≤ a+b , luego − a+b ≥ − a+b . Por lo tanto, 4 4       bc ca b2 c2 ab a2 + b− + c− + + = a− a+b b+c c+a a+b b+c c+a   a+b b+c c+a ≥ (a + b + c) − + + 4 4 4 a+b+c = = 1. 2

Como ab ≤

Soluci´ on del problema 25. Sea ∠ABC el ´angulo recto. Por construcci´on BR = BC y BP = AB, adem´as como ∠ABC = π/2 = ∠RBP, los tri´angulos ABC y P BR son congruentes y de lados paralelos, por lo que AC y P R son iguales y paralelos. Como BQ y AC son perpendiculares, resulta que BQ y RP son perpendiculares. Q A

B

R

D

C

E P

34

Soluciones Problemas de Pr´ actica

Sean D y E las intersecciones de BQ con AC y RP , respectivamente. QE es altura del tri´angulo P QR. Por construcci´on 2 BE = BQ. Calculemos el ´area del tri´angulo P QR: (P QR) = 12 RP · QE = 23 AC · BD = 3 (ABC). Entonces 1 (ABC) = . (P QR) 3

Soluci´ on del problema 26. Consideremos el hex´agono ABCDEF coloreado como indica el problema. H A

F G

B C

E D

I Sobre los lados AF y CD construimos tri´angulos equil´ateros AF H y CID respectivamente, como se ve en la figura. En el tri´angulo BGH tenemos que (AHG) = (BAG) por tener la misma base y la misma altura. An´alogamente, (CBG) = (CIG), (DIG) = (EDG) y (F EG) = (HF G). Luego, (AHF G) + (EDG) + (CBG) = (CGDI) + (BAG) + (F EG). Si restamos de esta ecuaci´on las ´areas que no interesan (los tri´angulos equil´ateros que construimos al principio), tenemos que: (AHF G)−(AF H)+(EDG)+(CBG) = (CGDI)−(CID)+(BAG)+(F EG), De donde, (AF G) + (EDG) + (CBG) = (DCG) + (BAG) + (F EG). Soluci´ on del problema 27. La estrategia que deber´a seguir el primer jugador es repetir la ´ultima jugada del segundo jugador. Para explicar esto, s´olo basta colorear el tablero de la siguiente manera:

3.1 Soluciones de los Problemas de Pr´ actica

35

As´ı, con la estrategia descrita, como el primer jugador siempre comienza en una casilla coloreada, (por la paridad del tablero) podemos asegurar que siempre llegar´a en sus turnos a una casilla de las que quedaron coloreadas, y que llegar´a primero a la casilla superior derecha del tablero, puesto que est´a coloreada tambi´en. El segundo jugador solamente podr´a llegar en sus turnos a las casillas blancas, jam´as podr´a llegar a las casillas coloreadas.

Soluci´ on del problema 28. Sea n, n + 1, . . . , n + 9 diez enteros consecutivos. Si dos de ellos, n + i, n + j (0 ≤ i < j ≤ 10), tiene divisor primo p en com´un, entonces p|(n + j) − (n + i) = j − i. Ahora bien, j − i est´a entre 1 y 8, de modo que p s´olo puede ser 2, 3, 5 ´o 7. Por lo tanto, si (n + i) no es m´ultiplo de 2, 3, 5 ´o 7, (n+i) ser´a primo relativo con los otros nueve enteros consecutivos. Entonces basta para cada n dar una i, 0 ≤ i < 10, tal que (n + i) no sea m´ultiplo de 2, 3, 5 o´ 7. Para eso s´olo necesitamos tomar en cuenta el n = 0, 1, 2, 3, . . . , 209. Si n es un primo mayor que 7, podemos tomar i = 0. Eso nos da una i para n = 11, 13, 17, 19, 23, 29, 31, 37, 41, 43, 47, 53, 59, 61, 67, 71, 73, 79, 83, 89, 97, 101, 103, 107, 109, 113, 127, 131, 137, 139, 149, 151, 157, 163, 167, 173, 179, 181, 191, 193, 197199. Si i = 0 funciona para n = m, entonces i = k funciona para n = m − k con 0 < k < 10, por eso, donde en la lista anterior los “huecos”son de longitud 10 o menos podemos hallar una i apropiada para el “hueco” n. Hay s´olo 3 “huecos”de longitud mayor que 10: del principio al 11, del 113 al 127, del 199 al final. Los podemos llenar notando que i = 0 funciona tambi´en para n = 1, 121, 209. Esto concluye la prueba.

Soluci´ on del problema 29. Cada punto X del plano lo giramos 60◦ alrededor de B (en el sentido de las manecillas del reloj) para obtener X ′ , por ejemplo, C = A′ .

36

Soluciones Problemas de Pr´ actica

A D P B

P′

E

M C

D′ M′

Por la construcci´on BDD ′ es equil´atero. Probaremos que E es el punto medio de DD ′, de modo que los ´angulos del tri´angulo BDE son 30◦ , 60◦ y 90◦ . Si probamos que P DCD ′ es un paralelogramo habremos terminado (porque las diagonales de un paralelogramo se bisecan). P ′D ′ forma un ´angulo de 60◦ con P D, as´ı que ∠P ′D ′ C = 120◦ y P DkD ′C. Adem´as, es claro que P D = D ′ C. Por lo tanto, P DCD ′ es un paralelogramo, que es lo que nos faltaba probar. Soluci´ on del problema 30. Si k es el elemento mayor de un subconjunto de A de 7 elementos de {1, 2, 3, . . . , 10}, entonces A \ {k} ⊂ {1, 2, 3, . . . , k − 1}. Por lo tanto, hay k−1 subconjuntos de {1, 2, 3, . . . , 10} cuyo m´aximo elemento es 6  k. Cada uno de esos k−1 subconjuntos contribuye k a la suma, por lo que la 6 suma deseada es:  X    10 10 10 10   X X X (k − 1)! k−1 11 k! k k = k . =7 7 = =7 6 8 7 6!(k − 7)! k=1 7!(k − 7)! k=1 k=7 k=7   P k n+1 (Aqu´ı usamos la f´ormula nk=m m = m+1 . Esto se puede probar viendo que  n+1 es el n´umero de subconjuntos de m + 1 elementos de {1, 2, 3, . . . , n + 1} m+1  k y que m cuenta cuantos de ellos tienen como m´aximo elemento a k + 1).

Soluci´ on del problema 31. Hay 2n subconjuntos de N incluyendo el conjunto vac´ıo, 2n−1 de los cuales contienen a n y 2n−1 que no contienen a n. Adem´as se

3.1 Soluciones de los Problemas de Pr´ actica

37

corresponden: si A es un conjunto que tiene a n, el conjunto A\n no lo contiene. La suma S de un conjunto A que contiene a n es de la forma S = n−a+b−c . . . y la suma S ′ = a − b + c . . .. Luego S + S ′ = n y hay 2n−1 de estas sumas. Por lo tanto, la suma total es n · 2n−1 . Soluci´ on del problema 32. Sea a = AB = CD = DE la longitud del lado, b = AC = CE la longitud de la diagonal menor y c = AD = AE la longitud de la diagonal mayor. Por el teorema de Ptolomeo aplicado al cuadril´atero ACDE, tenemos que ab + ac = bc, ecuaci´on que al dividir entre abc, nos da el resultado. Soluci´ on del problema 33. Dividamos el cuadril´atero AF DE por la diagonal AD, formando dos tri´angulos de ´areas a y b que determinaremos. Usando el hecho de que si dos tri´angulos tienen la misma altura, la raz´on de sus ´areas es igual a la raz´on de sus bases, tenemos que:

AE a a+b+3 = = EC 3 12

y

AF b a+b+3 = = FB 3 12

Estas identidades nos llevan al sistema de ecuaciones: 3a = b + 3 3b = a + 3 que admiten por soluci´on, a = b = 32 , por lo que ´area (AF DE) = 3. Soluci´ on del problema 34. Veamos que no es posible. Los cuadros de las esquinas ser´an pares, cuando un tablero de 2 × 2 se coloque en las esquinas un n´umero impar de veces. El n´umero del cuadro central se modifica con cualquier tablero de 2×2, luego si las esquinas se modifican para llegar a ser n´umeros pares, el central se modificar´a aumentando al n´umero 1, los cuatro n´umeros impares, lo que dar´a al final un impar, no as´ı un n´umero par como desear´ıamos. Segunda Soluci´ on. Inicialmente la suma de todos los n´umeros de la cuadr´ıcula es 5, que es impar. Cada vez que se aplica la regla para modificar la cuadr´ıcula, la suma aumenta en 4 y entonces el nuevo n´umero es tambi´en impar. Una posici´on en que todos los n´umeros son pares tendr´a una suma tambi´en par, luego no es posible llegar a la posici´on que se indica. Soluci´ on del problema 35. Supongamos que para alguna n, 2n termina en ˙ Como 2n acaba en 2, 2n−1 deber´a terminar en 1 ´o 6. Si 2n−1 acaba en 1982.

38

Soluciones Problemas de Pr´ actica

1, entonces 2n−1 = 1 y 2n = 2 lo cual no puede ser. Luego 2n−1 acaba en 6 y entonces 2n−1 = 10a + 6 para alg´un entero positivo a, por lo que 2n = 20a + 12, como en 20a hay un n´umero par de decenas (hay 2a) y en 12 hay una, el d´ıgito de las decenas de 2n es impar, luego no puede ser 8, por lo que ninguna potencia de 2 termina en 82. Soluci´ on del problema 36. Sea O el punto medio de AB y centro de la circunferencia. Como MB = ME por ser longitudes de las tangentes a una circunferencia desde el punto M y como OB = OE por ser radios, tenemos que los tri´angulos rect´angulos OBM y OEM son congruentes, luego ∠BOM = ∠MOE = α. t1 t2 C E M A

α α

α

O

B

Tambi´en tenemos que ∠BAE = 21 ∠BOE = α, luego ACkOM y como O es punto medio de AB, tenemos que M es punto medio de BC. Soluci´ on del problema 37. Las “sustituciones”no alteran la paridad de los n´umeros de la cuadr´ıcula. En la cuadr´ıcula original hay 13 pares y 12 impares, por lo anterior despu´es de cualquier n´umero de “sustituciones” habr´an 13 n´umeros pares y 12 impares por lo que nunca se podr´a llegar a los 12 pares y 13 impares que tienen los n´umeros del 1 al 25. Soluci´ on del problema 38. No es posible. Sean r el n´umero de regiones en una divisi´on de la esfera en tri´angulos y a el n´umero de aristas que tiene esta divisi´on. Como cada tri´angulo tiene tres aristas y como cada arista delimita a dos regiones, se tiene que 3r = 2a. As´ı una divisi´on de la esfera en r tri´angulos nos lleva a que 3r es par, por lo que r deber´a ser par. Soluci´ on del problema 39. Supongamos que es posible y sean p, q, r n´umeros primos que son las longitudes de los lados de un tri´angulo cuya ´area A es entera. De la f´ormula de Her´on, obtenemos:

3.1 Soluciones de los Problemas de Pr´ actica

39

16A2 = (p + q + r) (p + q − r) (p − q + r) (−p + q + r) Como los factores del lado derecho son todos de la misma paridad y como el lado izquierdo es par, tenemos que cada factor es par. Luego p, q y r no pueden ser los tres impares, as´ı alguno es 2, digamos que r = 2. Por la desigualdad del tri´angulo, q < p + 2 y p < q + 2, por tanto |p − q| < 2. Si |p − q| = 1, entonces como √ p y q son primos estos son 2 y 3 y el ´area A del 3 7 tri´angulo de lados 2, 2 y 3 es 4 que no es entero. Si |p − q| = 0, entonces p = q y tenemos que 16A2 = (2p + 2) (2p − 2) (2)(2) luego A2 = p2 − 1 y como p y A son enteros, se deber´a tener que p = 1 y A = 0 lo cual no es posible. As´ı en cualquier caso llegamos a una contradicci´on. Soluci´ on del problema 40. Si tenemos 4 colores para pintar los 100 n´umeros, por el principio de las casillas, alg´un color se debe usar 25 o m´as veces, digamos que el color es el rojo y que los 25 n´umeros pintados de rojo son: r1 < r2 < ... < r25 . Consideremos las 24 diferencias: r25 − r24 < r25 − r23 < ... < r25 − r1 , si alguna de estas diferencias es roja ya terminamos, por lo que supondremos ahora que no hay rojas, estos 24 n´umeros est´an entonces coloreados con 3 colores por lo que un color, digamos verde, se debe usar al menos 8 veces, sean v1 = r25 − rj1 < ... < v8 = r25 − rj8 las diferencias coloreadas con verde. Las 7 diferencias v8 − v1 , ..., v8 − v7 , son distintas, positivas y menores que 100, si alguna es verde ya terminamos. Como cada una de estas diferencias (v8 − vl ) = (r25 − rj8 ) −(r25 − rjl ) = rjl −rj8 , es una diferencia de rojos, ninguna de estas 7 diferencias es rojo, luego los 7 n´umeros son azules o amarillos. Estos 7 n´umeros al colorearse de amarillo o azul, usan un color al menos 4 veces, digamos el amarillo para: a1 = v8 − vm1 < a2 = v8 − vm2 < a3 = v8 − vm3 < a4 = v8 − vm4 . Si una de las diferencias, a4 − a1 , a4 − a2 , a4 − a3 es amarilla acabamos, como a4 − ai = (v8 − vm4 ) − (v8 − vmi ) = vmi − vm4 , es diferencia de verdes. Tenemos que si a4 − ai es verde,  tambi´en acabamos. Y como a4 − ai = vmi − vm4 = r25 − rji − r25 − rjm4 = rjm4 − rji es tambi´en diferencia de rojos, tenemos que si a4 − ai es roja tambi´en acabamos. Luego b4 = a4 − a1 , b2 = a4 − a2 y b3 = a4 − a3 son azules, ahora: b3 − b1 = a1 − a3 = vm3 − vm1 = rjm1 − rjm3 , es un n´umero que se puede ver como diferencia de dos n´umeros pintados del mismo color y de cada color, luego tambi´en terminamos.

40

Soluciones Problemas de Pr´ actica

Soluci´ on del problema 41. Sea t la tangente por P , a C1 ( y C2 ) y T el punto de intersecci´on de t con AB. Tenemos que T P = T Q, por lo que el tri´angulo T P Q es is´osceles. C1 B

Q A T

α

γ

β

δ β

t

α

P Como t es tangente se tiene que el ´angulo α que forma la tangente t con la cuerda BP es igual al ∠BAP . An´alogamente β = ∠ABP = ∠AP T . Luego si γ = ∠QT P y δ = ∠T QP = ∠QP T , tenemos que: α = γ + β por ser ´angulo externo del tri´angulo AT P y γ + 2δ = 180◦ por ser la suma de los ´angulos interiores del tri´angulo T P Q. Por tanto, 2γ = 180◦ − γ = 180◦ − (α − β) .   180◦ (α+β) α+β ◦ − β = 90 − = . As´ı, ∠AP Q = δ − β = 90◦ − α−β 2 2 2 ◦ Pero ∠AP B = 180 − (α + β) es constante para cualquier P dentro del arco AB, luego ∠AP Q es constante. Soluci´ on del problema 42. El que ∠AOB = 60◦ , implica que el tri´angulo AOB es equil´atero. Si R, Q son puntos medios, entonces RQkAB y RQ = 12 AB. Si P , S son puntos medios, tambi´en P SkAB y P S = 12 AB. A P R

M S

b

O

Q

B

3.1 Soluciones de los Problemas de Pr´ actica

41

Adem´as SQ = P R = 21 MO = 21 AB. An´alogamente SQkMOkP R, luego P RQS es un paralelogramo de lados iguales, es decir un rombo. Pero en un rombo las diagonales son perpendiculares. Soluci´ on del problema 43. Observemos que m´odulo 7, se tiene que: (7n ± 1)3 (7n ± 2)3 (7n ± 3)3 (7n)3

≡ ≡ ≡ ≡

(±1)3 ≡ ±1 (±1)3 ≡ ±1 (±1)3 ≡ ±1 0

Luego x3 + y 3 − z 3 ≡ 0 m´odulo 7, implica que uno de x3 , y 2, z 3 es congruente a 0 m´odulo 7 y entonces x, y ´o z ≡ 0 (mod 7). Soluci´ on del problema 44. Supongamos que ninguno de p1 , p2 , p3 es divisible entre 3 entonces pj ≡ ±1 (mod 3), por lo que p = p21 +p22 +p23 ≡ 0 (mod 3). Pero si p es primo, debe ser p = 3, lo que es una contradicci´on pues p = p21 +p22 +p23 > 3. Por tanto p1 = 3. Soluci´ on del problema 45. Sean a, b, c, d, e y f las longitudes de los segmentos en que se han dividido los lados del rect´angulo, como muestra la figura. 6 12

4

6

d

8

a

f e

c

b

De los per´ımetros conocidos tenemos: a+e b+e c+e d+b f +b

= = = = =

6 2 3 4 3

luego: a + b + c + d + e + f + 2b + 2e = 18. As´ı a + b + c + d + e + f = 18 − 2(b + e) = 18 − 4 = 14. Por lo que el per´ımetro del rect´angulo es 28.

42

Soluciones Problemas de Pr´ actica

Soluci´ on del problema 46. (i) No es posible por lo siguiente, en un pent´agono debemos poner un color, digamos el color A, alrededor de ´el hay 5 pent´agonos que se deben colorear con los otros dos colores B y C, pero como se debe de ir coloreando en forma alternada B, C, B, C en el quinto pent´agono no importa que color pongamos tendremos dos colores iguales juntos. (ii) Una coloraci´on con 4 colores es la siguiente 4 2

3

3

1 1

4 1

2

4

as 2 atr´

3

Soluci´ on del problema 47. Sea xi xi+1 xi+2 xi+3 , uno de los sumandos, si de los factores hay a n´umeros que son −1 y 4 − a que son 1, tenemos que: xi xi+1 xi+2 xi+3 = (−1)a Pero (−1)a ≡ 1 − 2a ≡ 1 + (4 − a) − a ≡ 1 + xi + xi+1 + xi+2 + xi+3 (mod 4). Luego:

0=

n X i=1

xi xi+1 xi+2 xi+3 ≡

n X i=1

(1 + xi + xi+1 + xi+2 + xi+3 ) ≡ n (mod 4)

la ´ultima congruencia se sigue de que cada xi aparece 4 veces, luego n es divisible entre 4. Soluci´ on del problema 48. Supongamos que n es el n´umero buscado, debemos tener que: n = (a + 1) + (a + 2) + ... + (a + 9) = 9a + 45 = 9 (a + 5) n = (b + 1) + (b + 2) + ... + (b + 10) = 10b + 55 = 5 (2b + 11) n = (c + 1) + (c + 2) + ... + (c + 11) = 11c + 66 = 11(c + 6) luego n deber´a ser m´ultiplo de 9, 5 y 11, como el n´umero m´as peque˜no que es m´ultiplo de estos tres es 9 · 5 · 11 = 495, este es el n´umero buscado, bastar´a tomar a = 50, b = 44 y c = 39.

3.1 Soluciones de los Problemas de Pr´ actica

43

Soluci´ on del problema 49. Supongamos que es posible y que 102a es la suma de los elementos de A1 , 203b la suma de los elementos de A2 y 304c la de los de A3 . Como entre A1 , A2 y A3 se tienen todos los n´umeros entre 1 y 100. 102a + 203b + 304c = 1 + 2 + ... + 100 = 50 · 101, la ecuaci´on anterior se puede reescribir como: a + b + c + 101 (a + 2b + 3b) = 50 · 101. Luego, 101 | a + b + c y como a + b + c > 0, tenemos que a + b + c ≥ 101. Por otra parte: 101 (a + b + c) < 102a + 203b + 304c = 50 · 101, de donde a + b + c < 50, lo que es una contradicci´on. Luego no es posible. Soluci´ on del problema 50. La primera hoja tiene las p´aginas 1 y 2, la segunda hoja los p´aginas 3 y 4, luego la siguiente las 5 y 6,... en general una hoja tiene las p´aginas 4n + 1 y 4n + 2 o las p´aginas 4n + 3 y 4n + 4. En cualquier caso la suma de los n´umeros de cada hoja es congruente a 3 m´odulo 4. Luego la suma de los n´umeros en 24 hojas es 24 · 3 ≡ 0 (mod 4) y como 1998 ≡ 2 (mod 4), nunca podr´a ser la suma 1998. Soluci´ on del problema 51. Veamos como cambia la suma σ de los 10 n´umeros, cuando un cuadrito cambia de valor, es decir cuando el cuadrito se multiplica por −1. Cuando hacemos este cambio el producto del rengl´on y de la columna correspondiente tambi´en cambian de signo, luego la suma se altera por −4, 0 o´ 4 dependiendo si los productos del rengl´on y de la columna eran los dos +1, uno +1 y el otro −1 o los dos eran −1. As´ı la operaci´on de cambiar el valor de un cuadrito, altera la suma σ en m´ultiplos de 4. Si al principio hay puros 1, tenemos que σ = 10 y entonces los ´unicos valores posibles son 10, 6, 2, −2, −6 y −10, por lo que nunca se obtendr´a el valor 0. 2

2

2

2

2

−2pq +q Soluci´ on del problema 52. pp+q es entero si y s´olo si p +q +2pq−2pq = (p+q) p+q p+q 2pq es entero, si y s´olo si p+q es entero. Luego p + q divide a 2pq, pero los divisores de 2pq, son 1, 2, p, q, 2p, 2q, pq y 2pq. p + q = 1 ´o 2 es imposible por ser p y q mayores que 1. p + q = p ´o p + q = q nos lleva a que uno de p ´o q es cero lo cual es falso. p + q = 2p ´o p + q = 2q nos lleva a que p = q. p + q = pq ⇒ p = q(p − 1) como p y q son primos, tenemos que p = q = 2 p + q = 2pq ⇒ p = q(2p − 1) y como q es primo, llegamos a que p = 1, que es una contradicci´on.

Soluci´ on del problema 53. 3 | abcd ⇔ 3 | a + b + c + d ⇔ 3 | a + b + c + d − 3b + 3d = a − 2b + c + 4d.

44

Soluciones Problemas de Pr´ actica

Soluci´ on del problema 54. Supongamos que el cuadrado ABCD tiene ´area igual a 1, luego el ´area del cuadril´atero BCP A es 43 y el ´area del cuadril´atero AP CD es 41 . Como el ´area del cuadril´atero BCP A es mayor que 12 , P est´a del mismo lado de AC que D. Ya que: 34 = ´area (BCP A) = ´area (ABC) + ´area (AP C) = 21 + ´area (AP C), tenemos que ´area (AP C) = 14 = 21 ´area (ADC). Pero los tri´angulos AP C y ADC tienen la misma base, la altura del tri´angulo AP C es la mitad de la altura del tri´angulo ADC, luego la parte roja es el segmento paralelo a AC que est´a a la mitad de la altura de D a AC, es decir, es el segmento que une los puntos medios de AD y DC. Soluci´ on del problema 55. Sean a, b, c, d y e los n´umeros de cajas en las pilas de izquierda a derecha, por las condiciones del problema tenemos que: 1 ≤ e ≤ d ≤ c ≤ b ≤ a ≤ 5 o equivalentemente: 1 ≤ e < d + 1 < c + 2 < b + 3 < a + 4 ≤ 9. Es f´acil contar el n´umero de soluciones de esta desigualdad. Cada subconjunto de {1, 2, ..., 9} con 5 elementos corresponde a una ´unica soluci´on (pues podemos  ordenar s´olo de una manera los elementos del subconjunto). Luego hay 95 = 126 maneras de apilar las cajas. Soluci´ on del problema 56. Si tocamos una vez cada uno de los n2 focos de una cuadr´ıcula de n × n focos en principio apagados, todos quedar´an prendidos. En efecto cada foco cambia de estado cada vez que un foco del rengl´on o columna, donde se encuentra es tocado, es decir, cambia de estado 2n − 1 veces. Como 2n − 1 es impar queda prendido el foco. Soluci´ on del problema 57. Observemos que 1 + 2 + ... + 12 = 13×12 = 78. 2 (i) Supongamos que s´ı es posible lograr que las sumas sean 27, y sean a, b y c los n´umeros que quedan colocados en los v´ertices del tri´angulo. Entonces en los lados del tri´angulo sin contar los extremos las sumas son 27 − a − b, 27 − b − c y 27 − c − a. Entonces a + b + c + (27 − a − b) + (27 − b − c) + (27 − c − a) = 78, por tanto 3 × 27 − (a + b + c) = 78, de donde a + b + c = 3, lo cual es imposible. (ii) Para ver que s´ı es posible con 28 consideremos la suma como arriba; en este caso 3 × 28 − (a + b + c) = 78, por lo que a + b + c = 6. Pongamos a = 1, b = 2 y c = 3. Entonces las sumas en los lados sin considerar los extremos deben ser 25, 24 y 23. Pongamos 12 en el lado con suma 25, 11 en el que tiene suma 24 y 10 en el de suma 23; entonces en las casillas que sobran en cada lado la suma debe ser 13, lo cual se logra con 4 + 9, 5 + 8 y 6 + 7 como se muestra en el esquema.

3.1 Soluciones de los Problemas de Pr´ actica

45

1 9

11

4

6

9 2

11 10

5

8

3

Soluci´ on del problema 58. Sea O el centro del oct´agono. De la ley de cosenos en el tri´angulo ABO, √ √ √ AB = OA2 + OB 2 − 2OA · OB cos 45◦ = 2 − 2 cos 45◦ = 2 − 2. Usando la ley de cosenos en los tri´angulos ACO y ADO obtenemos: q √ √ √ √ ◦ ◦ AC = 2 − 2 cos 90 = 2, AD = 2 − 2 cos 135 = 2 + 2 y finalmente, AB · AD =

p

2−

√ √ p √ √ 2 2 + 2 = 4 − 2 = 2 = AC.

Soluci´ on del problema 59. Si los cuatro puntos A, B C y D forman un cuadril´atero convexo, tenemos que los ´angulos interiores del cuadril´atero suman 360◦ , luego no es posible que los cuatro ´angulos sean menores de 90◦ , digamos que ∠A ≥ 90◦ entonces A junto con los dos v´ertices adyacentes forman un tri´angulo que no es acut´angulo. Si los cuatro puntos no forman un cuadril´atero convexo, hay tres que forman un tri´angulo, digamos A, B y C dejando al cuarto punto D dentro del tri´angulo ABC, si nos fijamos en los ´angulos ∠ADB, ∠BDC y ∠CDA al menos uno de ellos es mayor de 90◦ si por ejemplo es el primero, tendremos que ADB es el tri´angulo no acut´angulo. Soluci´ on del problema 60. Supongamos que O es el gravicentro del tri´angulo AMN. Sea T = AC · MN. Como O es el gravicentro de AMN, T es punto medio de MN y AO = 2OT .

46

Soluciones Problemas de Pr´ actica D A

N O B

T M

C

Como M y N son los puntos medios de los lados CB y CD del tri´angulo BCD, el que T sea punto medio de MN implica que O es el punto medio de BD, tambi´en como M y N son puntos medios, OT = T C. Entonces AO = 2OT = OT + T C = OC. As´ı que AC y BD se bisectan, de modo que ABCD es paralelogramo. Rec´ıprocamente, si ABCD es paralelogramo, AO = OC = 2OT y T es el punto medio de MN. De modo que AT es mediana del tri´angulo AMN y O la divide en raz´on 2 a 1 por lo tanto O es el gravicentro del tri´angulo AMN. Soluci´ on del problema 61. Si arreglamos los n´umeros en forma triangular 1 2 5 10 17 . . . 1850 1937 2026

4 7 9 12 14 16 19 21 23 25

. .

. .

. .

. .

1936 = 442 2023 2025 = 452

Vemos que el par m´as cercano a 1998 es el 2026. Soluci´ on del problema 62. (i) Tomemos tres enteros positivos consecutivos n−1, n y n+1. Entonces su producto es n3 −n. Ahora observemos que (n − 1)3 < n3 − n < n3 (la segunda desigualdad es clara; para ver la primera, notemos que (n − 1)3 = n3 − 3n2 + 3n − 1, as´ı que basta probar que −3n2 + 3n − 1 < −n, esto es, 3n2 − 4n + 1 > 0, es decir, (n − 1)(3n − 1) > 0; pero esta u ´ltima es clara 3 pues ambos factores son positivos ya que n ≥ 2). Entonces n − n no puede ser

3.1 Soluciones de los Problemas de Pr´ actica

47

el cubo de un entero pues se encuentra entre dos cubos consecutivos que son (n − 1)3 y n3 .

(ii) Supongamos que 2p + 1 = A3 , con A entero. que entonces A es  Notemos 3 2 impar. Despejando p tenemos p = A 2−1 = A−1 (A + A + 1). Pero p es primo 2 A−1 A−1 2 y 2 es entero (pues A es impar), as´ı 2 = 1 ´o A + A + 1 = 1; como la segunda ecuaci´on es imposible (A > 0), entonces A − 1 = 2, as´ı 2p + 1 = 27 y p = 13.

Soluci´ on del problema 63. Observemos que la ´unica manera en que se pueden cumplir simult´aneamente las dos condiciones a > b > c y que a sea factor de b + c, es que b + c = a. Entonces a + c = b + 2c y como b es divisor de a + c, entonces b debe ser divisor tambi´en de 2c; otra vez, ´esta ´ultima condici´on junto con la hip´otesis b > c nos dice que b = 2c. Por tanto a = b + c = 3c. Entonces 3 abc = 6c6c = c2 . a+b+c

Soluci´ on del problema 64. Como los ´angulos del hex´agono son iguales, este ´angulo vale 120o , luego si prolongamos lados alternados del hex´agono al intersectarse obtenemos un tri´angulo equil´atero . e

e

f a

d 120◦

a

b

60◦ c

c

Sean a, b, c, d, e, f los lados del hex´agono como se muestra en la figura entonces a + b + c + d + e + f = 21 y si l es el lado del tri´angulo equil´atero formado, entonces l = a + b + c = c + d + e = e + f + a de donde obtenemos que 3l = 21 + a + c + e, as´ı l = 7 + a+c+e . Pero el valor m´as chico de a + c + e es 3 2 cuando la terna (a, c, e) toma los valores 1, 2, 3 y el m´as grande es 5 cuando toma los valores 4, 5, 6. Los valores 3 y 4 los toma ´unicamente en las ternas que se indica en la siguiente tabla:

48

Soluciones Problemas de Pr´ actica

a+c+e 3

2

3

4 5

(a, c, e) (1, 2, 3) (1, 2, 6) (1, 3, 5) (2, 3, 4) (3, 4, 5) (1, 5, 6) (2, 4, 6) (4, 5, 6)

En el caso de a+c+e = 2, obtenemos el hex´agono tomando a = 1, c = 2 y e = 3, 3 entonces b = 6, d = 4 y f = 5. (Observe que no importa el orden en que se tomen los n´umeros 1, 2, 3 para la terna (a, c, e) pues el hex´agono es el mismo). Para el siguiente caso donde l = 10 la ´unica terna posible es (1, 3, 5), pues por ejemplo la terna (1, 2, 6) es imposible ya que para formar el tri´angulo de lado 10, en el lado donde aparece 1 y 2 necesitar´ıamos un segmento de entre b, d, f de medida 7 lo cual es imposible. De la misma forma para el caso l = 11 s´olo es posible formar un tri´angulo con la terna (2, 4, 6) y finalmente para l = 12 obtenemos otro tri´angulo posible. As´ı hemos encontrado cuatro hex´agonos posibles con esas caracter´ısticas. Las ´areas posibles son dos como se ve en el siguiente esquema, 2 o recuerde que el ´area de un tri´angulo equil´atero de lado l es l sen2 60 , luego el ´area o del hex´agono es sen260 (l2 − (a2 + c2 + e2 )): a+c+e 3

2 3 4 5

l 9 10 11 12

´area 3 67 √4 3 65 √4 3 65 √4 3 67 4 √

Soluci´ on del problema 65. Supongamos que s´ı es posible y que el acomodo es el siguiente: a d g

b e h

c f i

Entonces la suma que se pide que sea igual a 1997 es:

Soluciones

49

abc + def + ghi + adg + beh + cf i = 1997 Pero al tomar congruencia m´odulo 9, tenemos en el lado izquierdo que: abc + def + ghi + adg + beh + cf i ≡ 2 (a + b + c + d + e + f + g + h + i) ≡ 2 (1 + 2 + 3 + 4 + 5 + 6 + 7 + 8 + 9) ≡ 2 · 45 ≡ 0 (mod 9) Por otro lado 1997 ≡ 8 (mod 9). Luego, no es posible el acomodo.

3.2.

Soluciones de los u ´ltimos tres Concursos Nacionales de la OMM

Soluci´ on del problema 1. Para que el n´umero sea m´ultiplo de 3 la suma de sus d´ıgitos debe ser m´ultiplo de 3. Por lo tanto, el n´umero de sietes deber´a ser m´ultiplo de 3. Si hay 0 ´o 6 sietes el n´umero no resulta m´ultiplo de 7 (esto puede verse de varias maneras: directamente dividiendo entre 7; usando congruencias; o, en caso de un n´umero con 6 sietes, diciendo que por ser suma de un n´umero de 6 sietes y un cero, con un n´umero que es un tres seguido de ceros, no puede ser m´ultiplo de 7; etc.) Si el n´umero tiene 3 sietes, es suma de un n´umero de 3 sietes y algunos ceros con un n´umero de 4 treses y algunos ceros. Para que sea m´ultiplo de 7 el n´umero original, debe serlo el de 4 treses y algunos ceros. Los n´umeros 3, 30, 300, ..., 3000000 dejas residuos 3, 2, 6, 4, 5, 1, 3 al dividir entre 7, respectivamente. S´olo resta encontrar todas las maneras de elegir cuatro de ´estos que sumen un m´ultiplo de 7. (Se simplifica el trabajo si ponemos siempre los cuatro residuos en orden creciente). Son las siguientes: 1, 2, 5, 6; 1, 3, 4, 6 (son 2 pues hay dos 3′ s distintos); 2, 3, 4, 6; 2, 3, 4, 5 (son 2, pues hay dos 3′ s distintos). Estas 6 combinaciones corresponden a los n´umeros 7337337, 3373377, 7373373, 3777333, 3733737, 7733733, que son los deseados. Segunda Soluci´ on. Hay muchas maneras de resolver este ue involucran m´as o menos trabajo. Se puede decir desde el principio que el n´umero se escribe como 3a + 7b, donde a y b son n´umeros de ceros y unos que suman 1111111. Entonces, el n´umero es

50

Soluciones de los ´ ultimos tres Concursos

3(111111−b)+7b = 333333+4b. Aqu´ı, b debe ser m´ultiplo de 3, y como 333333 deja residuo 3 al dividir entre 7, b debe dejar residuo 1. Por lo tanto, buscamos n´umeros de ceros y unos con 0, 3 ´o 6 unos que dejen residuo 1 al dividir entre 7. Esto es parecido a lo de arriba, pero un poco m´as f´acil (porque ahora s´olo son 3 residuos en lugar de 4). Se pueden encontrar las maneras de escoger 4 de 3, 2, 6, 4, 5, 1, 3′ de manera que sumen un m´ultiplo de 7 con un argumento combinatorio. Los agrupamos en parejas que sumen 7: 1 − 6, 2 − 5, 3 − 4 y 3′ − 4. Si usamos los dos n´umeros de una pareja, los otros dos n´umeros deben formar una pareja tambi´en. Si usamos parejas de ´estas, no podemos usar la de 3 y la de 3′ porque repiten el 4. Esto nos da 42 − 1 = 5 combinaciones. Ahora, si no usamos parejas, es f´acil ver que necesitamos usar tanto el 3 como el 3′ . Los otros dos n´umeros deben sumar 8, la ´unica posibilidad es 2, 6. Para encontrar los m´ultiplos de 7 entre los n´umeros con 3 sietes, podemos hacer las divisiones: son 35. Cambiar los sietes por ceros ayuda). Despu´es de hacer eso, podemos dividir entre 3, es decir cambiar los treses por unos, que puede simplificar un poco el trabajo). Aqu´ı conviene observar que los ceros del final no contribuyen a que el n´umero sea m´ultiplo de 7; eliminarlos reduce las 35 divisiones a 20. Soluci´ on del problema 2. Si se toman tres pelotas de colores diferentes, ´estas se encuentran en una o dos cajas s´olamente. Veamos que hay tres pelotas de colores distintos que est´an en dos cajas distintas. De lo contrario cualesquiera tres pelotas de colores distintos est´an siempre en una misma caja. Sean A, B, C pelotas de colores diferentes (hay por lo menos 3 colores). Deben estar en la misma caja. Sea P cualquier otra pelota. P difiere en color con por lo menos dos de A, B y C, por lo que tambi´en est´a en la caja de A, B y C. Entonces, todas las pelotas est´an en una caja y como hay por lo menos tres cajas, hay cajas vac´ıas, lo cual es una contradicci´on. Por lo tanto, existen tres pelotas A, B y C de colores distintos tales que A y B est´an en la misma caja, y C est´a en otra. Ahora, consideremos una pelota P que no est´a ni en la caja de A y B, ni en la de C (hay por lo menos tres cajas). Entonces, A, C y P est´an en cajas distintas y por lo tanto, P es del mismo color que A o del mismo color que C. An´alogamente, considerando B, C y P , vemos que P es del color de B o del de C. Por lo tanto, es del color de C. En resumen, cualquier pelota de una caja diferente a la de A (y B) y a la de C es del mismo color que C. Escogiendo ahora una pelota C ′ de

Soluciones

51

una de estas cajas (es decir de una caja que no es ni la de A y B, ni la de C) y repitiendo el argumento, vemos que todas las pelotas de la caja de C son del mismo color que C ′ y, por lo tanto, del mismo color que C. As´ı todas las pelotas que no est´an en la caja de A y B son del mismo color. Soluci´ on del problema 3. Por el teorema de la potencia de un punto, tenemos que AP · P C = BP · P D y que DQ · DP = DM 2 = CM 2 = CR · CP . D Q

A P

T S B

M R C

Ahora, por el teorema de Tales aplicado a las paralelas AB y ST , AT BS DQ DQ · DP CR · CP CR = = = = = AP BP BP BP · DP AP · P C AP que es lo que quer´ıamos probar. Soluci´ on del problema 4. Son todas las n ≥ 6. Sea a1 , a2 , ..., a2001 la lista (sin signos) que se obtiene para a = 2 (donde a1 = 2) y sea S su suma alternada. Si S ≤ 0 nos fijamos en a = a2 , su lista asociada es a2 , a3 , ..., a2001 , a2002 . La suma alternada de esta lista es −(S − 2) + a2002 = −S + 2 + a2002 > 0. De modo que si 2 no cumple la condici´on, a2 s´ı, siempre y cuando est´e entre 2 y n2 . Si n ≥ 9, a2 = 4 y est´a en el rango deseado, de modo que para n ≥ 9 existe a que cumpla las condiciones. Veamos n = 5, 6, 7, 8: Para n = 5 la ´unica a en el rango es 2 y no cumple. Su lista asociada es 2, 4, 1, 1, 1, ... cuya suma alternada es −1. Para n = 6 elegimos a = 2. Su lista es 2, 4, 4, 4, ... cuya suma alternada es 2. Para n = 7 elegimos a = 3. Su lista es 3, 2, 4, 2, 4, 2, 4, ... cuya suma alternada es 3 + 2 · 1000 = 2003. Para n = 8 tambi´en elegimos a = 3. Su lista es 3, 1, 1, 1, ... cuya suma alternada es 3.

52

Soluciones de los ´ ultimos tres Concursos

Soluci´ on del problema 5. Notemos que ∠MAB = 90−∠A/2 y que ∠MBA = ∠A, por lo que ∠AMB = 180 − (∠A + 90 − ∠A/2) = 90 − ∠A/2 y as´ı AMB es is´osceles. Entonces, MB = AB = CD, de modo que MB y DC son iguales y paralelos, por lo que MBCD es un paralelogramo. Por lo tanto, ∠DMB = ∠C. Por otra parte, ABNC es un paralelogramo, de modo que ∠BNC = ∠A y ∠DCN = 180 − ∠A. Tambi´en, CD = AB = NC, por lo que DCN es is´osceles. A M

D

C

B

N Entonces, ∠NDC = ∠DNC = ∠A/2. Por lo tanto, ∠BND = ∠A/2. Como ∠C = ∠A/2, tenemos que ∠BND = ∠DMB, por lo que MDN es is´osceles y MD = ND. Soluci´ on del problema 6. Sean C1 , ..., C5 las cajas. Introducimos cajas auxiliares D1 , ..., D5 . En la caja Di ponemos una moneda de cada denominaci´on que falta en Ci . Notemos que (a) y (b) se cumplen para las cajas Di . En cambio, (c) y (d) se transforman en: (c′ ) Cualesquiera dos Di′ s distintas no pueden tener denominaciones en com´un. (d′ ) Para cualquier denominaci´on, ha una moneda de esa denominaci´on en alguna Di . As´ı (c′ ) y (d′ ) nos dicen que las denominaciones est´an repartidas en las cajas Di . Resolvemos mejor el problema para estas cajas. Por (a), (b) y (c′ ), si cada Di tiene t monedas, n = 5t. Aqu´ı, t no puede ser 1, pues no se cumplir´ıa (b). Probaremos que n puede ser cualquier 5t con t ≥ 2. Para t = 2 acomodamos las monedas como sigue:

Soluciones

53

D1 1 10

D2 2 9

D3 3 8

D4 4 7

D5 5 6

D2 2 9 13

D3 3 10 11

D4 4 6 14

D5 5 7 12

Para t = 3 las acomodamos as´ı: D1 1 8 15

Observemos ahora que dada una distribuci´on de monedas en cajas que cumpla (a), (b) y (c′ ) podemos agregarle el mismo valor a cada moneda en esas cajas, y la distribuci´on sigue cumpliendo las tres condiciones. Tambi´en, si tenemos dos distribuciones que cumplan esas tres condiciones y que no tengan denominaciones en com´un podemos unirlas en una sola que tambi´en las cumpla. Simplemente ponemos en la primer caja de una nueva distribuci´on lo que estaba en las primeras cajas de las otras dos distribuciones, en la segunda caja lo que hab´ıa en las segundas cajas de las otras dos, etc. Entonces, podemos hacer una distribuci´on para t = 5 como sigue: hacemos la de t = 3 (´esta usa las denominciones 1, 2, ..., 15), hacemos la de t = 2 agregando a cada moneda 15 (´esta usa las denominaciones 16, 17, ..., 25); finalmente, juntamos esas dos. Ahora, con la distribuci´on para t = 5 podemos hacer una para t = 7, 9, 11, .... Tambi´en en lugar de usar distribuciones para t = 3 y t = 2, podemos usar varias veces la de t = 2; as´ı obtenemos distribuciones para t = 4, 6, 8, .... Soluci´ on del problema 7. Si uno hace las operaciones del problema en una cuadr´ıcula de 4 × 4 es f´acil conjeturar que al terminar todas las operaciones, la cuadr´ıcula queda girada 90◦ en el sentido de las manecillas del reloj. Prob´emoslo: Si se empezara con una cuadr´ıcula de 2 × 2, obviamente quedar´ıa girada. Supongamos que si se empieza con una cuadr´ıcula de n × n termina girada, donde n es alguna potencia de 2. Para una cuadr´ıcula de 2n × 2n la primer operaci´on nos lleva de: a →

b →

d →

c →

a

d →

a →

c →

b →

54

Soluciones de los ´ ultimos tres Concursos

(donde las flechas indican que los n´umeros est´an en orden creciente de izquierda a derecha en cada rengl´on). El resto de los movimientos se hacen dentro de esas cuatro subcuadr´ıculas de n × n y ya sabemos que el resultado de esas es girar cada cuadro 90◦ , de modo que obtenemos: d ↓

a ↓

c ↓

b ↓

(donde las flechas indican que los n´umeros est´an en orden creciente de arriba a abajo en cada columna). Esto es la cuadr´ıcula de 2n × 2n girada 90◦ . Esto prueba (por inducci´on) que el resultado de aplicar las operaciones del problema en cualquier cuadr´ıcula cuadrada cuyo lado es potencia de 2 es girar la cuadr´ıcula 90◦ en el sentido de las manecillas del reloj. Por lo tanto, los n´umeros que quedan en la diagonal son 32, 63, 94, 125, ..., 993 (aumentan de 31 en 31). Soluci´ on del problema 8. Como AD es paralela a BE, los arcos AB y DE son iguales y por lo tanto, DE = AB = DC y CDE es is´osceles. An´alogamente, CBF es is´osceles. Sean M y N los puntos medios de CE y CF respectivamente. Por los tri´angulos is´osceles, las rectas DM y BN son las mediatrices de los segmentos CE y CF , y entonces, su intersecci´on K es el circuncentro. El cuadril´atero MCNK es c´ıclico (por tener dos ´angulos opuestos rectos) y por lo tanto, ∠BKD = 180◦ − ∠ECF = 180◦ − ∠BCD = ∠ABC = ∠BED. Por lo tanto el cuadril´atero BKED es c´ıclico, es decir, K est´a sobre K. Segunda Soluci´ on. Sea K el circuncentro de CEF . Entonces, ∠EKF = 2∠ECF . Como AD es paralela a BE, los arcos AB y DE son iguales y por lo tanto, DE = AB = DC y CDE es is´osceles, de donde ∠EDC = 180◦ − 2∠ECD. Por lo tanto, ∠EKF + ∠EDC = 180◦, as´ı que EKF D es c´ıclico y K est´a sobre K. Soluci´ on del problema 9. Tiene m´as de la forma 4k + 1. En efecto, si n no es divisible por primos de la forma 4k − 1, todos sus divisores impares son de la forma 4k + 1. ¿Qu´e pasa cuando se “agrega” un primo de la forma 4k − 1?. Supongamos que n cumple que r > s donde r y s son los n´umeros de divisores de n2 de las formas 4k + 1 y 4k − 1 respectivamente. Sea q un primo de la forma 4k − 1 que no divide a n. Probaremos que m = q α n tambi´en cumple la condici´on.

Soluciones

55

Cualquier divisor de m2 es de la forma q β d donde d es divisor de n2 y 0 ≤ β ≤ 2α. El divisor q β d es de la forma 4k + 1 si d lo es y β es par, o si d es de la forma 4k − 1 y β es impar. Por lo tanto, m2 tiene (α + 1)r + αs = α(r + s) + r divisores de la forma 4k + 1. Un razonamiento similar nos dice que m2 tiene α(r + s) + s divisores de la forma 4k − 1. Entonces, m2 tiene m´as divisores de la forma 4k + 1 que de la forma 4k − 1. Esto prueba (por inducci´on) que n2 siempre tiene m´as divisores de la forma 4k+1. Segunda Soluci´ on. Sea n = pα1 1 pα2 2 ...pαr r q1β1 q2β2 ...qsβs la factorizaci´on de n como producto de potencias de primos distintos, donde los qi son los primos de la forma 4k − 1 que dividen a n, y los pi son los de la forma 4k + 1 y un pi es 2 si n es par). Separamos todos los divisores de n2 en dos clases: los que son divisibles por alg´un qi y los que no son divisibles por ninguno. Dado un divisior d = pλ1 1 pλ2 2 ...pλr r q1µ1 q1µ1 ...qsµs de la primera clase, buscamos la m´ınima i tal que µ′ µi 6= 0 y a d le asociamos el divisor d′ = pλ1 1 pλ2 2 ...pλr r q1µ1 q1µ1 ...qi i ...pµs r donde ( µi + 1 µi < 2αi µ′i = 1 µi < 2αi . Si d es par, d′ tambi´en lo es. Si d es de la forma 4k + 1, d′ es de la forma 4k − 1 y viceversa. Esto nos da una biyecci´on, en los divisores de la primera clase, entre los divisores de la forma 4k − 1 y los de la forma 4k + 1. Los divisores de la segunda clase son todos pares o de la forma 4k + 1 (y hay al menos uno de la forma 4k + 1, a saber: 1), por lo que n2 tiene m´as divisores de la forma 4k + 1 que de la forma 4k − 1. Soluci´ on del problema 10. Para que la suma de los n´umeros en la primer ficha sea impar, uno de ellos debe ser par y el otro impar. Para que la suma siga impar, las siguientes fichas deben tener suma par, es decir, tener ambos n´umeros pares o ambos impares. Todas las fichas par-par deben quedar del lado par de la primera ficha y las impar-impar del lado impar. Las mulas (fichas dobles) siempre pueden colocarse en una hilera que cumpla las condiciones de manera que se sigan cumpliendo, por lo tanto, en una hilera de longitud m´axima se usan las siete. Encontremos la longitud m´axima de una hilera sin mulas: Del lado impar se pueden usar todas las fichas: 1 3 3 5 5 1 , por ejemplo. en cambio del lado par s´olo se pueden usar 5 de las 6 fichas. En efecto, cuando

56

Soluciones de los ´ ultimos tres Concursos

las fichas se acomodan en hilera, cada n´umero que no est´e en un extremo aparece un n´umero par de veces. Pero cada n´umero par (0, 2, 4, 6) aparece 3 veces en las fichas par-par y no pueden estar todos en los extremos. Por lo tanto debemos dejar sin usar al menos una de las fichas. Entonces la longitud m´axima de la parte par es a lo m´as 5 y la longitud m´axima de una hilera es a lo m´as 7+3+1+5 = 16: 7 mulas, 3 impar-impar, 1 impar-par y 5 par-par. Que de hecho se pueden formar hileras de esa longitud se ver´a cuando las contemos. ¿De cu´antas formas se puede formar el lado impar (sin mulas)? Una vez que se elige con que ficha empezar el resto tiene dos opciones, por ejemplo si empezamos con 1 3 , debe quedar 1 3 3 5 5 1 o 3 1 1 5 5 3 . As´ı que hay 6 formas. Ahora con mulas: una vez puestas las otras fichas las mulas se pueden insertar de dos formas: la mula del n´umero en los extremos se puede poner en cualquiera de los dos extremos y las otras dos est´an obligadas. Por lo tanto con todo y mulas hay 6 · 2 formas de hacer el lado impar. ¿Y el lado par (sin mulas)? Ya vimos que debe faltar una ficha, digamos 0 6 . En las dem´as fichas par-par, los n´umeros de la ficha faltante aparecen 2 veces, y los otros dos aparecen 3 veces. Estos ´ultimos deben ser los extremos del lado par. Entonces los dos n´umeros de la ficha faltante obligan a poner algunas juntas, en el ejemplo, debemos ir juntas 2 0 0 4 y tambi´en 2 6 6 4 . Junto con la ficha 2 4, tenemos 3 bloques con extremos 2 y 4. Para formar el lado par basta escoger el orden de los 3 bloques y si queremos empezar con 2 o con 4. Por lo tanto, sin mulas, hay 6 · 3!·2 (el primer factor es el n´umero de formas de elegir la ficha faltante) formas de hacer el lado par. Las mulas de los n´umeros de los extremos pueden estar en dos posiciones y las mulas de los otros dos n´umeros s´olo en una. Por lo tanto con todo y mulas hay (6 · 3! · 2(2 · 2) formas de hacer el lado par. El n´umero total de formas de hacer la hilera de longitud m´axima es el producto de las formas de hacer los dos lados (puesto que la ficha par-impar queda determinada). As´ı que el n´umero de formas buscado es (6 · 2)(6 · 3! · 2)(2 · 2) = 27 33 . Si una hilera y la que se obtiene al girarla 180◦ se consideran distintas el n´umero es el doble. Comentario. Una soluci´on m´as directa se obtiene haciendo el ´arbol completo de los casos aprovechando la simetr´ıa para reducir el trabajo. Soluci´ on del problema 11. Sean a, b y c tres enteros distintos que forman una terna compatible, y supongamos que a es el mayor de los tres. Notemos que a debe ser m´ultiplo o divisor de al menos uno de b y c, digamos de b. Como a > b, a debe ser m´ultiplo de b.

Soluciones

57

Si c tambi´en es divisor de a (o de b, lo cual lo vuelve divisor tambi´en de a) entonces b y c son a lo m´as a/2 y a/3 en alg´un orden, de modo que a + b + c ≤ 1 + 12 + 31 a = 11 a. 6 Si c no es divisor de a, debe ser m´ultiplo de b. Si a = kb, entonces c ≤ (k − 1)b, porque a > c. entonces, a + b + c ≤ a + b + (k − 1)b = a + kb = 2a. Entonces, en cualquier caso la suma es a lo m´as 2a y s´olo llega a eso si la terna es de la forma kb, b, (k − 1)b. Para las ternas formadas con n´umeros del 1 al 2002 la suma m´axima es, entonces, 4004 y s´olo ocurre para ternas de la forma antes mencionada cuando kb = 2002. Factorizando 2002 (es 2 · 7 · 11 · 13) encontramos sus divisores, los posibles valores de b. Las ternas son: {2002, 1, 2001}, {2002, 2, 2000}, {2002, 7, 1995}, {2002, 11, 1991}, {2002, 13, 1989}, {2002, 14, 1988} , {2002, 22, 1980}, {2002, 26, 1976}, {2002, 77, 1925}, {2002, 91, 1911}, {2002, 143, 1859}, {2002, 154, 1848}, {2002, 182, 1820}, {2002, 286, 1716}. (N´otese que tomar b = 1001 no resulta en una terna v´alida porque los tres n´umeros no son distintos). Soluci´ on del problema 12. El cuadril´atero BCKM es c´ıclico por tener ´angulos opuestos rectos. Entonces, ∠ABK = ∠MBK = ∠MCK = ∠MCD. An´alogamente, ∠BAK = ∠MDC, por lo que ∠ABK+∠BAK = ∠MCD+∠MDC = 180◦ − ∠CMD = 90◦ , de donde ∠AKB es recto. Como el tri´angulo AKB es rect´angulo, el punto medio de AB, M, es el circuncentro de AKB y por lo tanto, AM = MK. Entonces, los tri´angulos rect´angulos AMD y KMD tiene un cateto correspondiente igual y comparten la hipotenusa MD, lo cual los hace congruentes. an´alogamente MKC y MBC son congruentes. En particular, AD = DK y KC = BC. Usando esto hay muchas formas de terminar, aqu´ı presentamos algunas: Primera Forma. Como los tri´angulos AKC y ABC comparten el lado AC la raz´on de sus ´areas es la raz´on de sus alturas sobre el lado AC. La raz´on de estas alturas es KQ (trazando las alturas se forman tri´angulos semejantes). QB Por otra parte, los dos tri´angulos tienen un lado igual, a saber BC = KC, por lo que la raz´on de sus ´areas es la raz´on de las alturas a esos lados. As´ı que AR llamando R al pie de la perpendicular desde A a CD, KQ = AB . An´alogamente, QB KP BS = AB , donde S es el pie de la perpendicular desde B a CD. como AR, MK PA y BS son perpendiculares a CD y M es el punto medio de AB, tenemos que AR + BS = 2MK = AB, por lo que las fracciones suman uno, como deb´ıamos probar. Segunda Forma. Sean E el punto de intersecci´on de AC con BD y L el punto sobre la recta CD tal que AL es paralela a BD. Como AD y BC son paralelas,

58 CE EA

Soluciones de los ´ ultimos tres Concursos =

BC . AD

Por el teorema de Tales,

CE EA

=

CD . DL

Entonces,

AD BC BC = = . DL CD AD + BC Otra vez por el teorema de Tales, KD AD KP = = , PA DL DL de donde

KP AD = . PA AD + BC

An´alogamente

KQ BC = QB AD + BC y las fos fracciones suman uno, como se ped´ıa probar. Tercera Forma. Sea E el punto de intersecci´on de AC con BD. Por el teorema de Menelao aplicado al tri´angulo BKD con la recta EQC obtenemos: KQ BE DC · · = 1. QB ED KC BE Como AD y BC son paralelas, ED = CD = AD + BC, obtenemos que

BC . AD

Usando esto, que KC = BC y que

BC KQ = . QB AD + BC Ahora terminamos como en la forma anterior. Otra forma de empezar Finalmente, otra demostraci´on de que AKB es recto y de que AD = DK y KC = BC: Como ∠AMD + ∠BMC = 180◦ − ∠CMD = 90◦ , los tri´angulos rect´angulos AMD y BCM son semejantes. Ahora, tenemos que MC BC BC = = , MD AM MB por lo que el tri´angulo rect´angulo MCD tambi´en es semejante a AMD y BCM. Como MK es altura de MCD, entonces KMD y KCM tambi´en son semejantes a estos tri´angulos. Como AMD y KMD comparten la hipotenusa, de hecho son congruentes y por lo tanto, AD = DK. An´alogamente KC = BC.

Soluciones

59

Tambi´en, como AMD y KMD son congruentes, por simetr´ıa, AK es perpendicular a MD y an´alogamente BK es perpendicular a MC. Si llamamos R y S a las intersecciones de AK con MD y de MC con BK, tres de los ´angulos del cuadril´atero KRMS son rectos y por lo tanto el cuarto tambi´en. Soluci´ on del problema 13. Supongamos que m = kt. Notemos que 10k y m coinciden salvo por los d´ıgitos de las decenas y de las unidades. Espec´ıficamente, si el d´ıgitos de las unidades de k es a, entonces m termina en 0a y 10k termina en a0. Si a = 0, entonces 10k = m, de modo que todos los k que acaban en 0 cumplen la condici´on. Supongamos ahora que a no es cero. En este caso, 10k (que termina en a0) es mayor que m (que acaba en 0a). Por lo tanto, t es cuando mucho 9. Por otra parte t debe ser mayor que 1 porque m > k (tiene un d´ıgito m´as, de hecho). Para cada valor de t encontramos todas las posibilidades para k como sigue: a, el d´ıgito de las unidades de k, debe ser tal que kt tambi´en tenga d´ıgitos de las unidades a. Despu´es, encontramos el d´ıgito de las decenas usando que el d´ıgito de las decenas de m = kt es 0, de modo que t por el d´ıgito de las decenas de k m´as lo que se lleva de calcular at debe terminar en 0. A partir de este momento, cada d´ıgito de k debe ser tal que si se multiplica por t y al producto se le suma lo que se lleve de la posici´on anterior, el resultado termine en el d´ıgito anterior de k. Para t = 2, 4 y 8 no hay nin´un valor de a tal que at termine en a. Para t = 6, a puede ser 2, 4, 6 u 8. Con el procedimiento ya descrito obtenemos los n´umeros X2, X34, X84, X18 y X68, donde las X denotan d´ıgitos para los cuales no queda ninguna posibilidad (o sea que para t = 6 la ´unica soluci´on es k = 18). Notemos que si despu´es de 18 continuamos con el m´etodo, se obtiene . . . 00018, por lo que k = 18 realmente es la ´unica soluci´on en este caso. Para t = 5 se obtiene X5. Para t = 3 se obtiene . . . 28571428571435 donde el grupo de d´ıgitos 285714 se repite una infinidad de veces. Por lo tanto, no hay soluci´on con t = 3. Para t = 7 se obtiene . . . 0015 con el 0 repiti´endose, por lo que la u ´nica soluci´on con t = 7 es k = 15. Para t = 9 se obtiene . . . 0045 con el 0 repiti´endose, por lo que la u ´nica soluci´on con t = 9 es k = 45. En resumen las soluciones son 15, 18, 45 y los n´umeros que terminen en 0. Segunda Soluci´ on. Si b es el d´ıgito de las unidades de k y a es el n´umero formado por los dem´as d´ıgitos, entonces k = 10a+b y m = 100a+b. Supongamos que m = kt. Entonces, 100a + b = t(10a + b), o sea, 10(10 − t)a = (t − 1)b.

60

Soluciones de los ´ ultimos tres Concursos

Como t ≥ 1, el lado derecho es no negativo. Luego, el lado izquierdo tambi´en lo es, de donde t ≥ 10. Si t fuera 1, tendr´ıamos a = 0, lo cual es absurdo pues k tiene dos o m´as d´ıgitos. Si t = 10, entonces b = 0 y todos estos valores de k cumplen. Ahora (t − 1)b debe ser m´ultiplo de 10 y t − 1 no puede ser m´ultiplo de 10 (t est´a entre 2 y 9). Por lo tanto, b debe ser par, m´ultiplo de 5, o ambos. Si es ambos, b = 0 y esos valores ya los conoc´ıamos. Si b = 5, la ecuaci´on se simplifica a 2(10−t)a = t−1. Entonces, 2(10−t) ≤ t−1, o sea, t ≥ 7. Como t − 1 es par, t debe ser 7 ´o 9. Para t = 7 obtenemos a = 1 y la soluci´on k = 15. Para t = 9 obtenemos a = 4 y la soluci´on k = 45. Si b es para (distinto de 0), t − 1 debe ser m´ultiplo de 5. Por lo tanto, t = 6. La ecuaci´on se simplifica a 8a = b, como b es un d´ıgito, la u ´nica soluci´on es a = 1 y b = 8, o sea, k = 18. En resumen, las soluciones son los m´ultiplos de 10, y tambi´en los n´umeros 15, 45 y 18.

Soluci´ on del problema 14. Sea T la interseccion de AR con CS. Probaremos que T est´a sobre la tangente com´un a X y Z probando que el ´angulo ABT es recto. Como AB es di´ametro de X , ∠BRT es recto. COmo BC es di´ametro de Z, ∠BST es recto. Por lo tanto el cuadril´atero BRST es c´ıclico. Z

X B

A P R

C

S

Q

Y Si R y S est´an en P y Q, tenemos que ∠RBT = ∠RST = ∠CSQ = ∠CBQ = ∠BP Q = ∠BAR (la primer igualdad ocurre porque BRST es c´ıclico, la segunda porque los ´angulos son opuestos por el v´ertice, la tercera porque BSQC es c´ıclico,

Soluciones

61

la cuarta porque BC es tangente a Y, y la ´ultima porque AP BR es c´ıclico). Si P y Q est´an en R y S es muy parecido: ∠RBT = ∠RST = 180◦ − ∠CSQ = ∠CBQ = ∠BP Q = 180◦ − ∠BP R = ∠BAR. En cualquier caso, ∠RBT = ∠BAR. Como ∠BAR + ∠ABR = 90◦ , entonces ∠RBT + ∠ABR = 90◦ , o sea ∠ABT = 90◦ . Z

X B

A P

R

C

Y S

Q

Segunda Soluci´ on. Sea M el punto diametralmente opuesto a B en Y. Como AB es di´ametro de X , el ´angulo AP B es recto. Como BM es di´ametro de Y, el ´angulo BP M es recto. Por lo tanto, A, P y M son colineales. An´alogamente, C, Q y M son colineales. Probaremos que AR, CS y la tangente com´un concurren viendo que son alturas del tri´angulo ACX. Z

X B

A P R

C

S

Y

M

Q

62

Soluciones de los ´ ultimos tres Concursos

Para ver que AR es perpendicular a CM bastar´ıa ver que BR y CM son paralelas. Tenemos que ∠ABR = ∠QP M = ∠QBM = ∠BCQ (la primera igualdad es porque AP RB es c´ıclico, la segunda porque BP MQ lo es y la u ´ltima porque BM es tangente a Z). Por lo tanto, AR es altura de ACM. An´alogamente, CS es altura del tri´angulo y claramente BM es altura. Por lo tanto, AR, CS y BM concurren. Soluci´ on del problema 15. Son los valores de a y b que cumplen que a+b > n. Probemos primero que a + b > n, entonces forzosamente hay un muchacho y una muchacha que se gustan mutuamente: Primera Forma. Supondremos que no hay dos que se gustan mutuamente y demostraremos que a + b ≤ n. A cada quien le preguntamos quienes le gustan y hacemos una lista de parejas en als que alguno de los dos le gusta al otro. Como no hay dos que se gusten mutuamente, al hacer as´ı la lista no obtenemos parejas repetidas. Por lo tanto, obtenemos a lo m´as n2 parejas en la lista. Por otra parte, por cada muchacha obtenemos a parejas y por cada muchacho b, de modo que obtenemos exactamente an + bn = (a + b)n parejas. Entonces, (a + b)n ≤ n2 de donde a + b ≤ n. Segunda Forma. Este es realmente el mismo argumento que el anterior dicho de otro modo. Numeramos los muchachos y las muchachas con los n´umeros del 1 al n, y hacemos una tabla n × n en la que el cuadro en el rengl´on i y la columna j, lo pintamos de rojo si a la muchacha i le gusta el muchacho j, de azul si la muchacha i le gusta al muchacho j, y lo dejamos sin pintar en otro caso. Si no hay dos que se gusten mutuamente, ninguna casilla se pinta tanto de rojo como de azul. En cada rengl´on hay a casillas rojas y en cada columna hay b casillas azules, de modo que en total hay (a + b)n casillas pintadas. Por lo tanto, (a + b)n ≤ n2 , o sea, a + b ≤ n. Tercera forma. Supongamos que a + b > n. Como a cada muchacha le gustan a muchachos, en total hay an “gustos”de parte de las muchachas. Entonces, a alg´un muchacho le tocan al menos a “gustos”, es decir, hay alg´un muchacho popular que le gusta a al menos a muchachas. Como al muchacho popular le gustan b muchachcas y a+b > n, no pueden ser todas distointas las a muchachas a las que le gusta y las b que le gustan a ´el. Por lo tanto, hay una muchacha que le gusta a y a quien le gusta el muchacho popular. Ahora probaremos que si a + b ≤ n puede suceder que no haya una muchacha y un muchacho que se gusten mutuamente.

Soluciones

63

Primera Forma. Numeramos los muchachos y las muchachas con los n´umeros del 1 al n. Imaginemos que a la muchacha i le gustan los muchachos i, i + 1,. . ., i + a − 1 (los n´umeros se toman m´odulo n) y que al muchacho j le gustan las muchachas j + 1, j + 2, . . . , j + b. En este caso no hay dos que se gusten mutuamente. En efecto, si a la muchacha i le gusta el muchacho j, j debe ser uno de i, i + 1, . . . , i + a − 1, digamos i + t. Entonces, al muchacho j le gustan las muchachas i + t + 1, i + t + 2, . . . , i + t + b. El primero de eso n´umeros es al menos i + 1 y el ´ultimo es a lo m´as i + a + b − 1 ≤ i + n − 1, o sea que ninguno es i (o i + n). Segunda Forma. Por lo que se vi´o en la segunda forma de la primera parte, basta pintar una cuadr´ıcula de manera que haya a cuadros rojos en cada rengl´on y b azules en cada columna. Empezamos pintando los primeros a cuadros del primer rengl´on de rojo. El segundo rengl´on lo hacemos como el primero con los cuadros rojos recorridos un lugar a la derecha. El tercero es el segundo recorrido, etc. Cuando a la hora de recorres un cuadro rojo se salga por el borde de la cuadr´ıcula lo pasamos al principio de su rengl´on. Obtenemos una cuadr´ıcula con a cuadros rojos en cada rengl´on y n − a cuadros vac´ıos en cada columna. Como n − a ≥ b podemos simplemente elegir b cuadros de cada columna y pintarlos de azul. Tercera Forma. Elegimos a+b n´umeros distintos entre 1 y n, digamos r1 , r2 , . . . , ra , s1 , s2 , . . . , sb . Imaginemos una fiesta donde a la muchacha i le gusta el muchacho j si i − j es congruente m´odulo n con alg´un rk , y al muchacho j le gusta la muchacha i si i − j es congruente m´odulo n con un sk . As´ı es f´acil ver que a cada muchacha le gustan a muchachos: a la muchacha i le gustan los muchachos i − r1 , i − r2 , . . . , i − ra (m´odulo n). An´alogamente, a cada muchacho le gustan b muchachas. Adem´as, no hay dos que se gusten mutuamente pues si i − j es congruente con a lo m´as uno de los a + b n´umeros r1 , r2 , . . . , ra , s1 , s2 , . . . , sb . Soluci´ on del problema 16. Primero probaremos que MN es paralela a AB PM AP y a DC. Los tri´angulos AP M y QDM son semejantes, de donde M = DQ . D PN PB Tambi´en son semejantes P BN y CQN, de donde N C = QC . Pero por hip´otesis, AP PM PN = DQ , as´ı que M = N y por el teorema de Thales, MN es paralela a PB QC D C DC. Hay muchas formas de concluir, presentamos dos. Entonces, los tri´angulos P MN y P DC son semejantes, de donde DP DM DQ DC DC = =1+ =1+ =1+ . MN MP MP AP AB

64

Soluciones de los ´ ultimos tres Concursos

Aqu´ı, la ´ultima igualdad ocurre porque P divide al segmento AB en la misma raz´on que Q divide a DC. De aq´ui, despejamos MN =

D

AB · DC . AB + DC

D P

B

M

N

Q

C

Alternativamente, de la semejanza de los tri´angulos P MN y P DC obtenemos MN = PP M y de la semejanza de los tri´angulos QMN y QAB obtenemos que DC D MQ Q MN D N N = . Por el teorema de Thales, M = M . Entonces, M +M = AB AQ AQ PD DC AB MP DM + P D = 1, de donde podemos despejar otra vez MN. PD Soluci´ on del problema 17. Si el primer jugador empieza eligiendo la tarjeta (1, p) con p primo, pierde el jugador que no tome una tarjeta que incluya un m´ultiplo de p (pues en ese momento el m´aximo com´un divisor pasa de p a 1). Si la cantidad de tarjetas que incluyen un m´ultiplo de p es impar (incluyendo la tarjeta (1, p)), al primer jugador le toca la ´ultima de ellas y gana. Desafortunadamente, p = 2 no cumple: el n´umero de tarjetas con al menos un n´umero par es igual al total parejas menos el n´umero de parejas con ambos impares, esto es,  de  2003 1002 − 2 = 2003 · 1001 − 501 · 1001 que es par. Pero p = 3 si cumple: como 2  1336  2003 hay 1336 n´umeros entre 1 y 2003 que no son m´ultiplos de 3, hay 2 − 2 = 2003 · 1001 − 668 · 1335 parejas, donde al menos uno de los n´umeros es m´ultiplo de 3, y ese n´umero es impar. Se puede verificar que esta estrategia funciona para los siguientes valores de p: 3, 7, 11, 13, 23, 37, 43, 47, 73, 89, 101, 103, 107, 109, 127, 131, 137, 139, 167, 173, 179, 181, 191, 193, 197, 199, 251, 257, 263, 269, 271, 277, 281, 283, 293, 307, 311, 313, 317, 331, 503, 509, 521, 523, 541, 547, 557, 563, 569, 571, 577, 587, 593, 599, 601, 607, 613, 617, 619, 631, 641, 643, 647, 653, 659, 661, 673, 677, 683, 691, 701, 709, 719, 727, 733, 739, 743, 751, 757, 761, 769, 773, 787, 797, 809, 811, 821, 823, 827, 829, 839, 853, 857, 859, 863, 877, 881, 883, 887, 907, 911, 919, 929, 937, 941, 947, 953, 967, 971, 977, 983, 991, 997.

Soluciones

65

Soluci´ on del problema 18. Primero veamos que no importa el orden en el que se efectuen los dos tipos de cambio sensato: en un orden tenemos n → 2n + 1 → 3(2n + 1) + 2 = 6n + 5, en el otro n → 3n + 2 → 2(3n + 2) + 1 = 6n + 5. Entonces, al hacer una serie de cambios sensatos, podemos hacer primero todos los del tipo n → 2n + 1 y despu´es todos los del tipo n → 3n + 2. ¿Qu´e se obtiene si a n se le aplican k cambios sensatos del tipo n → 2n + 1? Calculemos los primeros pasos: n → 2n + 1 → 4n + 3 → 8n + 7 → 16n + 15 → . . .. Despu´es de k cambios se obiene 2k n + 2k − 1. An´alogamente, si a n se le aplican k cambios sensatos del tipo n → 3n + 2 se obtiene 3k n + 3k − 1. Ahora, podemos determinar cu´ando dos n´umeros a y b son compatibles: seg´un lo anterior, si c se obtiene con cambios sensatos a partir de a, usando j del primer tipo y k del segundo, entonces c = 3k (2j a + 2j − 1) + 3k − 1 = 2j 3k (a + 1) − 1. An´alogamente, si c se obtiene a partir de b, c se puede escribir como c = 2r 3s (b+ 1) − 1. Igualando, 2j 3k (a + 1) = 2r 3s (b + 1). Rec´ıprocamente, si existen n´umeros j, k, r y s que cumplan la ´ultima igualdad, a y b son compatibles (pues si a a le hacemos j cambios del tipo n → 2n + 1 y k del tipo n → 3n + 2 obtenemos el mismo resultado que si a b le hacemos r cambios del tipo n → 2n + 1 y s del tipo n → 3n + 2). As´ı que para que a se compatible con 2003, debemos tener, para algunos n´umeros, j, k, r y s, que 2j 3k (a+ 1) = 2r+2 3s+1 · 167, es decir, a = 2r+2−j 3s+1−k · 167 −1. Por lo tanto, los n´umeros buscados son los de la forma a = 2l 3m · 167 − 1 que son menores que 2003 = 12 · 167 − 1. Los n´umeros de la forma 2l 3m menores que 12 son 1, 2, 3, 4, 6, 8 y 9, por lo que los n´umeros buscados son 166, 333, 500, 667, 1001, 1335 y 1502. Segunda Soluci´ on. Supongamos que c se obtiene a partir de a con cambios sensatos y escribamos la lista de los n´umeros intermedios que se obtienen con los cambios sensatos: a → x → y → . . . → c, donde x = 2a+1 ´o 3a+2, y = 2x+1 ´o 3x + 2, etc. Si le sumamos uno a cada n´umero de la lista, obtenemos otra listas a + 1 → x + 1 → y + 1 → . . . → c + 1, en la que x + 1 = (2a + 1) + 1 = 2(a + 1) ´o (3a + 2) + 1 = 3(a + 1), es decir, el segundo n´umero es el doble o el triple del anterior. Consideremos ahora la factorizaci´on de a+1 como producto de potencias de primoes distintos. La factorizaci´on de c + 1 tiene las mismas potencias de los primos que no son ni 2, ni 3; y del 2 y el 3 puede tener cualquier potencia que sea mayor o igual que la que aparece en la factorizaci´on de a + 1. Por lo tanto, a y b con compatibles si y s´olo si las factorizaciones de a + 1 y b + 1 tienen las mismas potencias de todos los primos que no son ni 2, ni 3. Como 2003+1 = 22 ·3·167, los compatibles con 2003 son los de la forma 2l 3m ·167−1.

66

Soluciones de los ´ ultimos tres Concursos

De ´estos, los menores que 2003 son los que tienen 2l 3m 12. Los valores posibles son 2l 3m = 1, 2, 3, 4, 6, 8, 9, por lo que los n´umeros buscados son 166, 333, 500, 667, 1001, 1335 y 1502.

Bibliograf´ıa

[1] Comit´e Organizador de la Olimpiada Matem´atica Mexicana, Olimpiada de Matem´aticas, 140 problemas. Academia de la Investigaci´on Cient´ıfica, M´exico 1993. [2] Bulajich R.& G´omez Ortega J.A., Geometr´ıa, Cuadernos de Olimpiadas de Matem´aticas, Instituto de Matem´aticas, UNAM, 2002. [3] E. Gentile, Aritm´etica Elemental, Monograf´ıa No. 25 de la Serie de Matem´aticas del Programa Regional de Desarrollo Cient´ıfico y Tecnol´ogico de la OEA. Ediciones de la OEA, 1988. [4] R. Grimaldi, Matem´aticas Discreta y Combinatoria. Addison-Wesley Iberoamericana, M´exico 1989. [5] V. Gusiev, V. Litvinenko, A. Mordkovich, Pr´acticas para Resolver Problemas de Matem´aticas, (Geometr´ıa). Editorial Mir, Mosc´u 1969. [6] Illanes A., Principios de olimpiada, Cuadernos de Olimpiadas de Matem´aticas, Instituto de Matem´aticas de la UNAM, 2001. [7] V. Litvinenko, A. Mordkovich, Pr´acticas para Resolver Problemas de Matem´aticas, (Algebra y Trigonometr´ıa). Editorial Mir, Mosc´u 1989. [8] P´erez Segu´ı M.L., Combinatoria, Cuadernos de Olimpiadas de Matem´aticas, Instituto de Matem´aticas de la UNAM, 2000. [9] P´erez Segu´ı M.L., Teor´ıa de N´umeros, Cuadernos de Olimpiadas de Matem´aticas, Instituto de Matem´aticas de la UNAM, 2003.

68

Bibliograf´ıa

[10] I. Niven, H. Zuckerman, Introducci´on a la Teor´ıa de los N´umeros. LimusaWiley, M´exico 1972. [11] H. Shariguin, Problemas de Geometr´ıa, Colecci´on Ciencia Popular. Editorial Mir, Mosc´u 1989. [12] N. Vilenkin, ¿De Cu´antas Formas? (Combinatoria). Editorial Mir, Mosc´u 1972.

69

´ ORGANIZADOR DE LA OLIMPIADA MEXICANA DE COMITE MATEMATICAS Radmila Bulajich (Presidenta) Ana Alberro Semerena Ignacio Barradas Bribiesca Jes´us Jer´onimo Castro Mart´ın Eduardo Fr´ıas Armenta Jos´e Antonio G´omez Ortega Alejandro Illanes Mej´ıa Carmen Sosa Garza

Get in touch

Social

© Copyright 2013 - 2024 MYDOKUMENT.COM - All rights reserved.